You are on page 1of 143

CHAPTER 1: INTRODUCTION

Multiple Choice
a 1. The controller of a company or other organization is
a. a staff manager.
b. an operating manager.
c. an accountant, not a manager.
d. a natural manager.
c 2. Which item is NOT an IMA Standard for Ethical Conduct?
a. Integrity.
b. Competence.
c. Loyalty.
d. Objectivity.
d 3. Which statement about the degree of detail in a report is true?
a. It depends on the level of the manager receiving the report.
b. It may depend on the frequency of the report.
c. It depends on the type of manager receiving the report.
d. All of the above.
b 4. Managerial accounting is similar to financial accounting in that
a. both are governed by generally accepted accounting principles.
b. both deal with economic events.
c. both concentrate on historical costs.
d. both classify reported information in the same way.
d 5. Managerial accounting differs from financial accounting in that it is
a. more concerned with the future.
b. more concerned with segments of a company.
c. less constrained by rules and regulations.
d. all of the above.
b 6. One of the ways managerial accounting differs from financial accounting is that managerial
accounting
a. is bound by generally accepted accounting principles.
b. classifies information in different ways.
c. does not use financial statements.
d. deals only with economic events.
d 7. Which activity is NOT normally performed by managerial accountants?
a. Assisting managers to interpret data in managerial accounting reports.
b. Designing systems to provide information for internal and external reports.
c. Gathering data from sources other than the accounting system.
d. Deciding the best level of inventory to be maintained.
c 8. Conventional and just-in-time manufacturers both
a. Maintain large inventories of their products.
b. Sell only to other manufacturing companies.
c. Desire to meet customers' deadlines.
d. Require about the same amount of space to operate.
d 9. Classifying costs by behavior is
a. associated primarily with financial accounting.
b. not relevant to a company that has only selling expenses.
c. common in reports prepared for external readers.
d. none of the above.
a 10. Which is NOT a common accounting classification of costs?
a. By the method of payment for the expenditure.
b. By the objective of expenditure.
c. By behavior.
d. By the function incurring the expenditure.
a 11. Which classification of costs is most relevant for income statements to be used internally?
a. Behavior.
b. Function.
c. Method of payment.
1

d. Object.
d 12. The set of processes that transform raw materials into finished products is known as a
a. differentiation strategy.
b. flexible manufacturing system.
c. lowest cost strategy.
d. value chain.
a 13. Income statements classifying costs by object show such items as
a. tax expense, wages expense, depreciation expense.
b. cost of goods sold, selling expenses, administrative expenses.
c. assets, liabilities, owners' equity.
d. all of the above.
a 14. The period that begins with the arrival of materials and ends with the shipment of a completed
good is the
a. cycle time.
b. manufacturing cell.
c. computer-integrated manufacturing.
d. performance period.
a 15. Which function is most directly related to management by objectives?
a. Planning.
b. Control.
c. Decision making.
d. Reporting.
d 16. Which consideration influences the frequency of an internal report?
a. The wishes of the managers receiving the report.
b. The frequency with which decisions are made that require the information in the report.
c. The cost of preparing the report.
d. All of the above.
a 17. A just-in-time manufacturer is more likely than a conventional manufacturer to
a. receive more frequent deliveries of materials.
b. spend less money on advertising.
c. need workers with fewer skills.
d. all of the above.
c 18. A conventional manufacturer is more likely than a just-in-time manufacturer to
a. have a short production cycle.
b. produce goods in small batches.
c. hold large inventories to serve as buffers.
d. none of the above.
a 19. The professional certification most relevant for managerial accountants is the
a. CMA.
b. CPA.
c. CSA.
d. MAS.
d 20. A firm that is competing using a _______________________ strategy is attempting to create a
perception of uniqueness that will permit a higher selling price.
a. value chain
b. lowest cost
c. lead time
d. differentiation
b 21. Planning and control are
a. different names for the same thing.
b. the basic functions of management.
c. described equally well by the terms "decision making" and "performance evaluation."
d. exemplified by, respectively, financial statements and budgeting.
a 22. In contrast to a balance sheet, an income statement
a. is for a period of time, a balance sheet is at a point in time.
b. gives information about cash and a balance sheet does not.
c. is prepared after the statement of retained earnings.
2

d. has two columns, while a balance sheet has more than two.
c 23. One characteristic of the conventional manufacturing environment is
a. flexible manufacturing systems.
b. manufacturing cells.
c. a just-in-case philosophy.
d. a high degree of quality control.
D 24. A characteristic of the just-in-time manufacturing environment is
a. frequent deliveries of materials.
b. manufacturing cells.
c. little or no inventory of finished product.
d. all of the above.
d 25. Conventional and just-in-time manufacturers differ in that the conventional manufacturer is
likely to
a. be a new entrant into its industry.
b. need less storage space than its JIT competitors.
c. give less credibility to management accounting reports.
d. have a longer production cycle than its JIT competitors.
True-False
F 1. Published financial statements show costs classified by behavior.
T 2. Generally accepted accounting principles govern financial accounting but not managerial
accounting.
T 3. Economic events are the raw data for both financial and managerial accounting.
F 4. Internal financial statements must be prepared using generally accepted accounting principles.
T 5. The form and content of reports can influence decisions made by managers.
F 6. Management-by-objectives and management-by-exception are two names for the same general
management principle.
F 7. "Pro forma" is the name given to an income statement that classifies costs by function.
T 8. Some managerial accounting reports contain costs not incorporated in the basic accounting
system.
F 9. A professional examination exists to test the competence of financial accountants, but not of
managerial accountants.
F 10. Managerial accountants should, but have no obligation to, maintain their professional skills.
Problems
1. Based on the following, compute total owners' equity.
Cash
$ 8,000 Total noncurrent liabilities $
0
Total assets
$64,000 Retained earnings
$16,000
Total contributed
capital
$12,000 Current liabilities
$36,000
SOLUTION:
$28,000

($16,000 RE + $12,000 contributed capital)

2. Based on the following, compute total noncurrent assets.


Total current assets
Total current liabilities
Retained earnings
Total noncurrent liabilities
Total contributed capital

$600,000
$240,000
$162,000
$710,000
$360,000
3

SOLUTION:
$872,000 ($240,000 + $710,000 + $360,000 + $162,000 - $600,000)
3. Based on the information provided, compute accrued wages payable at the end of 20X4.
Accrued wages payable, beginning of 20X4
$ 14,000
Wages expense for 20X4
$ 45,000
Cash paid in 20X4 for wages
$ 42,500
SOLUTION:
16,500

($14,000 + $45,000 - $42,500)

4. Using the following data, find the missing items.


Total assets, beginning of year . . . . . . . $160
Total assets, end of year . . . . . . . . . . $240
Total liabilities, beginning of year . . . . $120
Total owners' equity, end of year . . . . . . $ 60
Net income for year . . . . . . . . . . . . $ 24
Additional investment during year . . . . . . $ 16
Contributed capital, beginning of the year . $ 30
a. Total owners' equity at the beginning of the year.
b. Total liabilities at the end of the year.
c. Retained earnings at the beginning of the year.
d. End-of-the-year retained earnings.
e. Dividends declared during the year.
SOLUTION:
a. Total OE, $40 ($160 - $120)
b. Total liabilities, $180 ($240 - $60 )
c. Beginning RE, $10 ($40 beginning owners' equity - $30 )
d. Ending RE, $14 [$60 - ($30 + $16)]
e. Dividends declared, $20 ($10 of beg RE + $24 - $14 ending RE)
5. Based on the following, compute total owners' equity.
Cash
Total assets

$ 30,000 Total noncurrent liabilities $18,000


$126,000 Retained earnings
$24,000
Current liabilities
$56,000

SOLUTION:
$52,000

(126,000 Total assets - $56,000 current liabilities - $18,000 total noncurrent liabilities)

6. Based on the following, compute total noncurrent assets.


Total current assets
Total current liabilities
Retained earnings
Total noncurrent liabilities
Total owners' equity

$135,000
$ 70,000
$ 47,500
$182,500
$125,000

SOLUTION:
$242,500 ($70,000 + $182,500 + $125,000 - $135,000)
7. Based on the information provided, compute accrued taxes payable at the end of 20X9.
Accrued taxes, beginning of 20X9
Taxes expense for 20X9

$244,000
$760,000
4

Cash paid in 20X9 for taxes

$750,000

SOLUTION:
$254,000

($244,000 + $760,000 - $750,000)

8. Based on the information provided, compute prepaid supplies at the end of 20X9.
Prepaid supplies, beginning of 20X9
Supplies expense for 20X9
Cash paid in 20X9 for supplies

$ 10,000
$210,000
$216,000

SOLUTION:
$16,000

($10,000 + $216,000 - $210,000)

9. Using the following data, find the missing items.


Total assets, beginning of year . . . . . . . $1,050
Total assets, end of year . . . . . . . . . . $1,200
Total liabilities, beginning of year . . . . $ 630
Total owners' equity, end of year . . . . . . $ 480
Net income for year . . . . . . . . . . . . $ 164
Additional capital invested during year . . . $ 50
Contributed capital, beginning of the year . $ 200
a. Total owners' equity at the beginning of the year.
b. Total liabilities at the end of the year.
c. Retained earnings at the beginning of the year.
d. End-of-the-year retained earnings.
e. Dividends declared during the year.
SOLUTION:
a. Total OE, $420 ($1,050 - $630)
b. Total liabilities, $720 ($1,200 - $480)
c. Beginning RE, $220 ($420 beginning owners' equity - $200)
d. Ending RE, $230 [$480 - ($200 + $50)]
e. Dividends declared, $154 ($220 of beg RE + $164 - $230 ending RE)
10. Using the following data, find the missing items.
Total liabilities, beginning of year . . . . . $440
Total liabilities, end of year . . . . . . . . $490
Total owners' equity, beginning of year . . . . $340
Total assets, end of year . . . . . . . . . . . $950
Dividends declared during the year. . . . . . . $ 74
Additional capital invested during year . . . . $ 90
Contributed capital, beginning of the year . . $240
a. Total assets at the beginning of the year.
b. Total owners' equity at the end of the year.
c. Retained earnings at the beginning of the year.
d. End-of-the-year retained earnings
e. Net income for year.
SOLUTION:
5

a. Total assets, $780 ($440 + $340)


b. Total owners' equity, $460 ($950 - $490)
c. Beginning RE, $100 ($340 beginning owners' equity - $240)
d. Ending RE, $130 [$460 - ($240 + $90)]
e. Net income, $104 ($130 of ending RE + $74 - $100 beginning RE)
CHAPTER 2: PROFIT PLANNING
Multiple Choice
c 1. Which formula gives unit sales required to earn a target profit? (P = selling price, V = variable
cost per unit, F = total fixed costs, T = target profit)
a. F/(P - V)
b. (F + T)/P
c. (F + T)/(P - V)
d. (F + T)/V
c 2. Which formula gives the sales dollars required to earn a target profit? (P = selling price, V =
variable cost per unit, F = total fixed costs, T = target profit)
a. F/[(P - V)/P]
b. (F + T)/(P)
c. (F + T)/[(P - V)/P]
d. F + T/V
d 3. Over the relevant range, total revenues and total costs
a. increase, but at a decreasing rate.
b. decrease.
c. remain constant.
d. can be graphed as straight lines.
b 4. At the break-even point, total contribution margin is
a. zero.
b. equal to total fixed costs.
c. equal to total costs.
d. equal to total variable costs.
c 5. If a company is operating at a loss,
a. fixed costs are greater than sales.
b. selling price is lower than variable cost per unit.
c. selling price is less than average total cost per unit.
d. fixed cost per unit is greater than variable cost per unit.
b 6. As volume increases, average cost per unit
a. increases.
b. decreases.
c. remains constant.
d. increases in proportion to the change in volume.
d 7. All else constant, if the selling price falls,
a. total variable costs will be lower than expected.
b. contribution margin percentage will be higher than expected.
c. total contribution margin will be higher than expected.
d. per-unit contribution margin will be lower than expected.
c 8. If all goes according to plan except that unit variable cost falls,
a. total contribution margin will be lower than expected.
b. the contribution margin percentage will be lower than expected.
c. profit will be higher than expected.
d. per-unit contribution margin will be lower than expected.
a 9. If all goes according to plan except that total fixed costs rise,
a. income will be lower than expected.
b. total contribution margin will be lower than expected.
c. total sales will be lower than expected.
d. income will be higher than expected.
a 10. Which of the following decreases per-unit contribution margin the most for a company currently
earning a profit?
a. A 10% decrease in selling price.
6

b. A 10% increase in variable cost per unit.


c. A 10% increase in fixed costs.
d. A 10% increase in fixed cost per unit.
c 11. If variable cost as a percentage of sales increases, the
a. contribution margin percentage increases.
b. selling price increases.
c. break-even point in dollars increases.
d. fixed costs decrease.
b 12. Which cost is most likely to be variable for a retailer?
a. Advertising.
b. Cost of goods sold.
c. Sales salaries.
d. Rent.
a 13. A cost-volume-profit graph reflects relationships
a. expected to hold over the relevant range.
b. of results over the past few years.
c. that the company's managers would like to have happen.
d. likely to prevail for the industry.
d 14. A multiproduct company
a. cannot use CVP analysis.
b. must use a separate CVP graph for each of its products.
c. can use CVP analysis only if the contribution margin percentages on each product are the
same.
d. could earn a higher-than-expected profit even though the total number of units sold was less
than expected.
a 15. If selling price, per-unit variable cost, and total fixed costs are constant,
a. the break-even point in units remains constant.
b. profit per unit remains constant for all levels of volume within the relevant range.
c. total variable costs equal total fixed costs.
d. total contribution margin equals total fixed costs.
b 16. XYZ Company desires a profit of $120,000 and expects to sell 20,000 units. Variable cost per
unit is $16 and total fixed costs are $160,000. The selling price must be
a. $40.
b. $30.
c. $26.
d. $20.
a 17. Contribution margin percentage is 30% and contribution margin per unit is $12. Which of the
following is true?
a. Variable cost per unit is $28.
b. Return on sales is 12%.
c. Selling price is $48.
d. Variable cost percentage is 12%.
b 18. Contribution margin is 30% of sales. Profit is $80,000. Sales are $600,000. Fixed costs are
a. $ 90,000.
b. $100,000.
c. $160,000.
d. $180,000.
a 19. TRS Company changed production methods, increasing fixed costs and decreasing its per-unit
variable costs. The change
a. increases risk and increases potential profit.
b. increases risk and decreases potential profit.
c. decreases risk and decreases potential profit.
d. decreases risk and increases potential profit.
c 20. Introducing income taxes into cost-volume-profit analysis
a. raises the break-even point.
b. lowers the break-even point.
c. increases unit sales needed to earn a particular target profit.
d. decreases the contribution margin percentage.
7

d 21. Selling price is $100, unit variable cost is $68, and fixed costs are $400,000. Unit sales required
to earn a $120,000 profit are
a. 5,200
b. 7,647
c. 13,700
d. 16,250
c 22. The tax rate is 40%. A company that wants a profit of $120,000 after taxes must earn how
much before taxes?
a. $ 48,000.
b. $ 72,000.
c. $200,000.
d. $300,000.
a 23. Genco Company has a 30% contribution margin percentage and fixed costs of $30,000. To earn
a 10% return on sales, Genco must have sales of
a. $150,000.
b. $100,000.
c. $40,000.
d. an amount that cannot be determined without more information.
a 24. If a company is earning a profit, its fixed costs
a. are less than total contribution margin.
b. are equal to total contribution margin.
c. are greater than total variable costs.
d. can be greater than or less than total contribution margin.
a 25. Per-unit variable cost
a. remains constant within the relevant range.
b. increases as volume increases within the relevant range.
c. decreases as volume increases within the relevant range.
d. decreases if volume increases beyond the relevant range.
d 26. An increase in the income tax rate
a. raises the break-even point.
b. lowers the break-even point.
c. decreases sales required to earn a particular after-tax profit.
d. increases sales required to earn a particular after-tax profit.
b 27. Contribution margin is
a. the same as gross margin.
b. revenue minus variable costs.
c. revenue minus variable costs and fixed costs.
d. the ratio of income to sales.
c 28. Classifying a cost as fixed or variable depends on how it behaves
a. per unit, as the volume of activity changes.
b. in total, as the volume of activity changes.
c. both a and b are correct.
d. none of the above.
d 29. Critical to CVP analysis in a multiproduct company is that
a. the products be complementary.
b. the products be sold to the same kinds of customers.
c. all products have about the same contribution margin percentage.
d. the sales mix is relatively constant.
a 30. A fixed cost is the same percentage of sales in three different months. Which of the following is
true?
a. The company had the same sales in each of those months.
b. The cost is both fixed and variable.
c. The company is operating at its break-even point.
d. The company is achieving its target level of profit.
c 31. If a company raises its target dollar profit, its
a. break-even point rises.
b. fixed costs increase.
c. required total contribution margin increases.
8

d. selling price rises.


a 32. If the sales mix shifts toward higher contribution margin products, the break-even point
a. decreases.
b. increases.
c. remains constant.
d. it is impossible to tell without more information.
a 33. In the following graph, revenue is represented by
A
|
*
D
|
*
*
|
*
*
|
*
*
|
* *
|
*
|
* *
|
*
*
| *
*
B|*__________*___________________________________ C
|
*
|
*
|
*
| *
|*______________________________________________
O
E
a. the line OA.
b. the line BD.
c. the vertical distance between the lines OA and BD.
d. the vertical axis.
c 34. In the following graph, the vertical distance between the lines OA and BD represents
|
A
D
|
*
*
|
*
*
|
*
*
|
* *
|
*
|
* *
|
*
*
| *
*
B|*__________* __________________________________ C
|
*
|
*
|
*
| *
|*______________________________________________
O
E
a. revenue.
b. total variable cost.
c. profit or loss.
d. total contribution margin.
d 35. In the following graph, total variable costs are represented by
A
|
*
D
|
*
*
|
*
*
|
*
*
|
* *
|
*
|
* *
|
*
*
| *
*
B|*__________* __________________________________ C
9

|
*
|
*
|
*
| *
|*______________________________________________
O
E
a. the line BD.
b. the line BC.
c. the vertical distance between the lines OA and BD.
d. the vertical distance between the lines BD and BC.
d 36. Target costing is
a. a substitute for CVP analysis.
b. used by companies that cannot classify their costs by behavior.
c. inappropriate if a company has already established a target profit.
d. used in decisions to offer a new product or enter a new market.
c 37. The break-even point in units equals total fixed costs divided by
a. selling price per unit.
b. variable cost per unit.
c. contribution margin per unit.
d. contribution margin percentage.
d 38. The break-even point in dollars equals total fixed costs divided by
a. selling price per unit.
b. variable cost as a percentage of selling price.
c. contribution margin per unit.
d. contribution margin percentage.
c 39. Company A has a lower variable cost per unit and higher total fixed costs than Company B. The
selling prices of their products are the same. Sales fluctuate considerably for both companies.
Therefore,
a. Company A has a lower break-even point than Company B.
b. Company A earns more profit than Company B.
c. Company A is more risky than Company B.
d. Company A has a lower contribution margin percentage than Company B.
b 40. The margin of safety is
a. the profit currently earned in excess of the target profit.
b. the difference between current sales and sales at break-even.
c. the ratio of contribution margin to variable cost.
d. the difference between contribution margin currently earned and contribution margin at break
even.
a 41. The indifference point is the level of volume at which a company
a. earns the same profit under different operating schemes.
b. earns no profit.
c. earns its target profit.
d. any of the above.
d 42. Selling price is $40, unit variable cost is $24, and fixed costs are $400,000. Unit sales required
to break even are
a. 10,000.
b. 12,500.
c. 16,667.
d. 25,000.
d 43. ABC's variable costs are 60% of total revenue. If fixed costs are $300,000, what is the breakeven sales volume?
a. $120,000
b. $180,000
c. $500,00
d. $750,000
b 44. Acme has sales of $200,000, fixed costs of $100,000, and a profit of $20,000. What is Acme's
margin of safety?
a. $ 20,000
b. $ 33,333
c. $100,000
10

d. An amount that cannot be determined without more information.


b 45. Machine A has fixed costs of $450,000 and a variable cost of $20. Machine B has fixed costs of
$600,000 and a variable cost of $14. What is the indifference point, in units?
a. 22,500
b. 25,000
c. 42,858
d. An amount that cannot be determined without more information.
d 46. DJH Company has sales of $360,000, variable costs of $216,000, and fixed costs of $150,000.
To earn a 10% return on sales, DJH must have sales of
a. $375,000.
b. $440,000.
c. $470,000.
d. $500,000.
b 47. DJH Company has sales of $400,000, variable costs of $240,000, and fixed costs of $150,000.
What is the break-even sales volume?
a. $150,000
b. $375,000
c. $390,000
d. $550,000
a 48. Alvarez Inc. sells three products with the following results:
X
Y
Z
------ ----------Sales
$10,000 $20,000 $30,000
Variable costs
4,000 12,000
15,000
What is the weighted average contribution margin percentage?
a. 48.3%
b. 50.0%
c. 51.7%
d. Cannot be determined with the information given.
c 49. Scottso Enterprises has fixed costs of $120,000. At a sales volume of $400,000, return on sales
is 10%; at a $600,000 volume, return on sales is 20%. What is the break-even volume?
a. $160,000
b. $210,000
c. $300,000
d. An amount that cannot be determined without more information.
d 50. Samson Inc. has a contribution margin percentage of 35%. If fixed costs are $630,000, what is
the break-even point?
a. $ 220,500
b. $ 409,500
c. $ 969,231
d. $1,800,000
True-False
F 1. Target costing is a technique for classifying costs according to their behavior.
T 2. "Gross profit" and "contribution margin" refer to different things.
F 3. A company that has no variable costs can never break even.
T 4. A company with no fixed costs has a break-even point of zero.
F 5. If a company's income statement shows a positive contribution margin but a net loss, its fixed
costs are too high.
T 6. As unit sales increase, both average total cost and fixed cost per unit decrease.
T 7. An increase in contribution margin percentage reduces the break-even point.
11

F 8. Return on sales is another name for contribution margin percentage.


F 9. Contribution margin is total variable costs minus fixed costs.
T 10. The weighted-average contribution margin percentage changes with changes in sales mix.
Problems
1. Foris Company's product sells for $16 and has a variable cost per unit of $12. Fixed costs are
$120,000.
a. Compute the break-even point in dollars.
b. Compute the number of units Foris must sell to earn a $30,000 profit.
c. Foris has a target profit of $36,000 and expects to sell 30,000 units. Compute the selling price
Foris must charge to earn the target profit.
d. Foris wants to keep its selling price at $16 per unit and earn a 10% return on sales. Calculate the
number of units Foris must sell to meet the target.
SOLUTION:
a. $480,000, $120,000/25% or $120,000/[($16 - $12)/$16]
b. 37,500, ($120,000 + $30,000)/($16 - $12)
c. $17.20,

$12 + [($120,000 + $36,000)/30,000]

d. 50,000, $120,000/[$16 - $12 - (.10 x $16)]


or [$120,000/(25% - 10%)]/$16
2. Dennis Company sells a product for $20, variable costs are $8 per unit, and fixed costs are
$32,000.
a. What is Dennis' break-even point in units?
b. Find the selling price that Dennis must charge to earn an $8,000 profit selling 1,600 units.
c. Dennis is considering new equipment that would increase fixed costs by $2,000 while reducing
unit variable costs by $1.60 per unit. Find the sales level where Dennis is indifferent between the two
cost structures.
SOLUTION:
a. 2,667

($32,000/$12)

b. $33.00
Profit = Sales - Variable Costs - Fixed Costs
$8,000 = 1,600X - $8 x 1,600 - $32,000
1,600X = $52,800
X = $33.00
c. 1,250 units
Current Costs = Proposed Costs
$32,000 + $8Q = $34,000 + $6.40Q
Q = 1,250
3. Stout Company sells three products. Planned results for next year follow.
Product
A
B
C
---------Selling price
$10
$8
$4
Variable cost
4
6
1
12

------Contribution margin
$6
$2
$3
===
===
===
Sales mix in dollars
25%
25%
50%
Fixed costs are $500,000.
a. Compute the weighted-average contribution margin percentage.
b. Compute the sales (in $) required to earn a $100,000 profit.
c. Suppose now that the sales mix, in UNITS, is 25%, 25%, 50%. Determine the weighted-average
contribution-margin per unit.
d. Determine the total unit sales needed to earn $100,000.
SOLUTION:
a. 58.75%
A
B
C Total
------- ----Contribution margin percentage 60%
25%
75%
Sales mix in dollars
25%
25%
50%
------Weighted-average
15% + 6.25% + 37.5% = 58.75%
b. $1,021,277 [($500,000 + $100,000)/.5875]
c. $3.50
A
B
C
Total
----------Contribution margin per unit $6.00
$2.00 $3.00
Sales mix in units
25%
25%
50%
--------- ----Weighted-average
$1.50 + $0.50 + $1.50 = $3.50
d. 171,429, ($500,000 + $100,000)/$3.50
4. Maple Company has sales of $550,000 and has variable costs of $330,000. Fixed costs are
$180,000.
a. Compute the break-even point.
b. Compute Maple's sales to earn a $50,000 profit.
c. Compute the sales Maple would need to earn a 10% return on sales.
SOLUTION:
a. $450,000 ($180,000/40% = $450,000)
[CM% = ($550,000 - $330,000)/$550,000 = 40%]
b. $575,000 [($180,000 + $50,000)/40% = $575,000]
c. $600,000 [$180,000/(40% - 10%) = $180,000/30% = $600,000]
5. Acme Company's product sells for $80 and has a variable cost per unit of $60. Fixed costs are
$400,000.
a. Compute the break-even point in dollars.
b. Compute the number of units must Acme sell to earn a $100,000 profit.
c. Acme has a target profit of $152,000 and expects to sell 30,000 units. Compute the selling price
13

Acme must charge to earn the target profit.


d. Acme wants to keep its selling price at $40 per unit and earn a 10% return on sales. Calculate
the number of units Acme must sell to meet the target.
SOLUTION:
a. $1,600,000 {$400,000/25% or $400,000/[($80 - $60)/$80]}
b. 25,000

[($400,000 + $100,000)/($80 - $60)]

c. $78.40

{$60 + [($400,000 + $152,000)/30,000]}

d. 33,333 {$400,000/[$80 - $60 - (.10 x $80)]}


or [$400,000/(25% - 10%)]/$80

6. Craik Company sells a product for $25, variable costs are 12 per unit, and fixed costs are $180,000.
a. What is Craik's break-even point?
b. Find the selling price that Craik must charge to earn a $40,000 profit selling 16,000 units.
c. Craik is considering new equipment that would increase fixed costs by $20,000 while reducing
unit variable costs by $2.00 per unit. Find the sales level where Craik is indifferent between the two
cost structures.
SOLUTION:
a. 13,846

($180,000/$13)

b. $25.75
Profit = Sales - Variable Costs - Fixed Costs
$40,000 = 16,000X - $12 x 16,000 - $180,000
16,000X = $412,000
X
= $25.75
c. 10,000 units
Current Costs = Proposed Costs
$180,000 + $12Q = $200,000 + $10Q
Q = 10,000
7. Mound Company has a before-tax return on sales of 9% and a 25% margin of safety. Current sales
are $800,000.
a. Calculate break-even sales.
b. Find Mound's variable cost percentage.
SOLUTION:
a. $600,000 ($800,000 x 75%)
b. 64%
Sales
$800,000
$600,000

Total Cost
$728,000 ($800,000 x 91%)
$600,000 break-even

$728,000 - $600,000
-------------------- = 64%
$800,000 - $600,000
8. Cranmore Company sells three products. Planned results are as follows.
14

Product
Q
R
----Selling price
$20
$8
$6
Variable cost
8
6
3
------Contribution margin
$12
$2
$3
===
===
===
Units sold
10,000 20,000 70,000
P
---

Fixed costs are $200,000.


a. Determine the weighted-average contribution-margin per unit.
b. Determine the break-even point in units sold.
c. Compute the total unit sales required to earn a $75,000 profit.
SOLUTION:
a. $3.70
A
--Units sold
Mix in units

B
C
Total
--------10,000 + 20,000 + 70,000 = 100,000
10%
20%
70%

Contribution margin per unit $12.00


$2.00
$3.00
------ ------ -----Weighted-average
$1.20 + $0.40 + $2.10 = $3.70
b. 54,054

($200,000/$3.70)

c. 74,324

[($200,000 + $75,000)/$3.70]

9. Oak Grove Inc's product sells for $32 and has a variable cost per unit of $20. Fixed costs are
$120,000. The effective tax rate is 40%.
a. Compute the break-even point.
b. Compute the number of units Oak Grove must sell to earn a $30,000 after-tax profit.
c. Oak Grove has an after-tax target profit of $36,000 and expects to sell 20,000 units. Compute
the selling price Oak Grove must charge to earn the target profit.

SOLUTION:
a. 10,000 ($120,000/12)
b. 14,167 [($120,000 + $30,000/60%)/12]
c. $29.00 {$20 + [($120,000 + $36,000/60%)/20,000]}
10. Eleva Company has sales of $350,000, variable costs of $200,000, and fixed costs of $125,000.
Eleva has an effective tax rate of 40%.
a. Compute the break-even point.
b. Compute Eleva's sales needed to earn a $75,000 after-tax profit.
c. Compute the sales Eleva would need to earn a 15% after-tax return on sales.
SOLUTION:
15

a. $291,647 ($125,000/42.86% = $291,647)


CM% = ($350,000 - $200,000)/$350,000 = 42.86%
b. $571,629 [($120,000 + $75,000/60%)/42.86%]
c. $699,888 [$125,000/(42.86% - 25%) = $699,888]
before-tax return on sales = 15%/60% = 25%
CHAPTER 3: COST ANALYSIS
Multiple Choices
b 1. The principal advantage of the scatter-diagram method over the high-low method of cost
estimation is that the scatter-diagram method
a. includes costs outside the relevant range.
b. considers more than two points.
c. can be used with more types of costs than the high-low method.
d. gives a precise mathematical fit of the points to the line.
a 2. The major objective of preparing a scatter-diagram is to
a. derive an equation to predict future costs.
b. perform regression analysis on the results.
c. determine the relevant range.
d. find the high and low points to use for the high-low method of estimating costs.
d 3. The cost estimation method that gives the most mathematically precise cost prediction equation
is
a. the high-low method.
b. the scatter-diagram method.
c. the contribution margin method.
d. regression analysis.
c 4. Which cost is most likely to be mixed for a manufacturer?
a. Raw materials.
b. Direct labor.
c. Manufacturing overhead.
d. Insurance.
b 5. Which combination of object of cost and classification of cost is most reasonable?
Object of Cost
Classification of Cost
-----------------------------------a. Materials used to make products Discretionary fixed cost
b. Advertising cost
Discretionary fixed cost
c. Straight-line depreciation
Variable cost
d. President's salary
Avoidable fixed cost
c 6. A cost is variable if it varies with the
a. number of units manufactured.
b. number of units sold.
c. level of some activity.
d. selling price of the product.
d 7. A non-value-adding cost is
a. usually direct to a product.
b. the same as a discretionary cost.
c. unavoidable.
d. not essential to manufacturing a product.
a 8. Fixed costs that cannot be reduced within a short period of time are
a. committed.
b. variable.
c. avoidable.
d. unnecessary.
b 9. Which cost is most likely to be committed?
a. Repairs and maintenance.
b. Sum-of-the-years'-digits depreciation on the factory building.
16

c. Fee for a consultant on the company's long-range planning.


d. Advertising.
a 10. RST's average cost per unit is the same at all levels of volume. Which of the following is true?
a. RST must have only variable costs.
b. RST must have only fixed costs.
c. RST must have some fixed costs and some variable costs.
d. RST's cost structure cannot be determined from this information.
b 11. A mixed cost
a. increases in steps as volume increases.
b. contains a fixed component and a variable component.
c. varies with more than one measure of volume.
d. cannot be accurately predicted.
b 12. A non-value-adding activity
a. cannot be a cost driver.
b. should be eliminated.
c. usually drives only variable costs.
d. cannot usually be observed by managers.
d 13. A cost-predicting equation determined through regression analysis
a. always gives close predictions.
b. will not work any better than one obtained using the high-low method.
c. can be used only for costs that vary with sales or production.
d. could be severely affected by outliers.
b 14. Which of the following do JIT operations try to eliminate?
a. Discretionary fixed costs.
b. Non-value-adding costs.
c. Avoidable costs.
d. Direct costs.
d 15. ABC Company breaks even at $600,000 sales and earns $60,000 at $700,000 sales. Which of
the following is true?
a. Fixed costs are $40,000.
b. Profit at sales of $800,000 would be $160,000.
c. The selling price per unit is $6.
d. Contribution margin is 60% of sales.
b 16. A seasonal business that sets selling prices at 20% above average cost for the preceding month
will
a. be better off if it closed down during the off-season.
b. charge higher prices in the off-season than in the busy season.
c. always charge higher prices than its competitors.
d. make a consistent return on sales of 20%.
b 17. The components of manufacturing cost are
a. variable costs, fixed costs, and overhead costs.
b. materials, direct labor, and overhead.
c. purchases, wages, and manufacturing overhead.
d. wages and salaries, maintenance and repairs, utilities, and depreciation.
b 18. Which statement is true for a manufacturer?
a. It cannot use the contribution-margin format of the income statement.
b. Many costs vary with production activities, not with sales.
c. The concepts of fixed and variable costs do not apply.
d. Cost-volume-profit analysis is not appropriate.
d 19. Fixed costs that managers can change on short notice are
a. value-adding costs.
b. variable costs.
c. unavoidable costs.
d. discretionary costs.
c 20. A(n) __________ relationship is one that appears to exist even though there is no causal
relationship.
a. Correlation.
b. Outlier.
c. Spurious.
17

d. Value-added.
c 21. Identifying cost drivers
a. is not necessary with regression analysis.
b. is the same as identifying cost pools.
c. is an important part of cost management.
d. is useful only with step-variable costs.
d 22. A cost pool is
a. all of the costs of a particular department.
b. all costs in a group such as variable costs or discretionary fixed costs.
c. all costs related to a product or product line.
d. all costs that have the same driver.
b 23. As volume increases,
a. total fixed costs remain constant and per-unit fixed costs increase.
b. total fixed costs remain constant and per-unit fixed costs decrease.
c. total fixed costs remain constant and per-unit fixed costs remain constant.
d. total fixed costs increase and per-unit fixed costs increase.
d 24. Which cost is NOT subtracted from selling price to
calculate contribution margin per unit?
Variable manufacturing overhead.
b. Variable selling expenses.
c. Direct labor.
d. Fixed manufacturing overhead.
c 25. A committed fixed cost
a. can never be eliminated.
b. can be eliminated in the short-term and in the long-term.
c. can be eliminated in the long-term, but not in the short-term.
d. can be eliminated in the short-term, but not in the long-term.
c 26. Avoidable costs are usually
a. committed.
b. common.
c. direct.
d. fixed.
a 27. Direct costs are
a. associated with a specific activity.
b. always variable.
c. usually committed.
d. usually discretionary.
c 28. Discretionary costs
a. are usually unavoidable.
b. are not necessary for successful operations.
c. can be either direct or indirect.
d. should be the first ones cut in a cost-reduction program.
a 29. Ogden Company had $300,000 overhead cost at 20,000 machine hours, $320,000 overhead
cost at 25,000 hours. Variable overhead cost per machine hour is
a. $ 4.00.
b. $12.80.
c. $15.00.
d. some other number.
b 30. Sacramento Company had $400,000 overhead cost at 50,000 machine hours and $460,000
overhead cost at 60,000 hours. Total fixed overhead is
a. $ 60,000
b. $100,000
c. $120,000.
d. $320,000.
d 31. Which cost is LEAST likely to be discretionary?
a. Salaries of salespeople.
b. Advertising.
c. Building maintenance.
18

d. Insurance.
a 32. Which cost is LEAST likely to be direct to a particular product?
a. Salaries of salespeople who sell all of the company's products.
b. Advertising of the product.
c. License fees paid to the designer of the product.
d. Cost of materials used to make the product.
c 33. Which cost is most likely to be avoidable in deciding whether to shut down one of the four
assembly lines in a factory?
a. Depreciation on the factory building.
b. Salaries of maintenance workers who service all assembly lines.
c. Power used to operate equipment on the assembly line.
d. Heat and light for the building.
c 34. DSP Company earned $100,000 on sales of $1,000,000.
$1,100,000. Variable costs as a percentage of sales are
a. 30%.
b. 40%.
c. 70%.
d. 90%.

It earned 130,000 on sales of

b 35. DSP Company earned $100,000 on sales of $1,000,000.


$1,100,000. Total fixed costs are
a. $0.
b. $200,000.
c. $420,000.
d. $900,000.

It earned $130,000 on sales of

c 36. Predicting costs at activity levels that are outside the relevant range is called
a. association.
b. correlation.
c. extrapolation.
d. none of the above.
a 37. A non-value-adding cost
a. is driven by a non-value-adding activity.
b. is discretionary.
c. is direct to a product.
d. allows the company to charge a higher price for the product.
b 38. Looking at the following scatter diagrams we can conclude that
$
$
|
**
|
**
|
* **
|
** *
|
*** *
| * *
|
* *
| **
|
|
|
|
|__________________
|__________________
activity
activity
Cost A
Cost B
a. cost A will be easier to predict than cost B.
b. cost B will be easier to predict than cost A.
c. cost A is out-of-control.
d. cost B has no fixed component.
b 39. MNO has a break-even point of 200,000 units and earns a $100,000 profit at sales of 250,000
units. Which of the following is true?
a. Fixed costs are $100,000.
b. Total contribution margin at 200,000 units is $400,000.
c. Profit at sales of 300,000 units is $120,000.
d. Selling price per unit is $2.
a 40. The closeness of the relationship between the cost and the activity is called
a. correlation.
b. spurious.
c. regression analysis.
19

d. manufacturing overhead.
d 41. R-squared is a measure of
a. the spurious relationship between cost and activity.
b. the fixed cost component.
c. the variable cost per unit of activity.
d. how well the regression line accounts for the changes in the dependent variable.
c 42. DJH has an average unit cost of $20 at 20,000 units and $13.75 at 40,000 units. What is the
variable cost per unit?
a. $5.00
b. $6.25
c. $7.50
d. An amount that cannot be determined without more information.
b 43. DJH has an average unit cost of $20 at 20,000 units and $13.75 at 40,000 units. What is the
total fixed cost?
a. $125,000
b. $250,000
c. $400,000
d. An amount that cannot be determined without more information.
a 44. GMH Company had $200,000 overhead cost at 25,000 machine hours and $240,000 overhead
cost at 60,000 hours. Variable overhead per machine hour is
a. $4.00.
b. $1.00.
c. $0.83.
d. some other number.
d 45. Elmwood Company had $300,000 overhead cost at 40,000 machine hours, and $360,000
overhead cost at 60,000 hours. Total fixed overhead is
a. $ 36,000
b. $ 40,000
c. $ 60,000.
d. $180,000.
b 46. Crookston Company breaks even at $300,000 sales and earns $40,000 at $400,000 sales.
Which of the following is true?
a. Fixed costs are $120,000.
b. Profit at sales of $500,000 would be $50,000.
c. The selling price per unit is $4.
d. Contribution margin is 10% of sales.
a 47. Glenwood has an average unit cost of $45 at 20,000 units and $25 at 60,000 units. What is the
variable cost per unit?
a. $15
b. $20
c. $35
d. An amount that cannot be determined without more information.
b 48. Glenwood has an average unit cost of $45 at 20,000 units and $25 at 60,000 units. What is the
total fixed cost?
a. $400,000
b. $600,000
c. $900,000
d. An amount that cannot be determined without more information.
b 49. Osceola Company earned $50,000 on sales of $400,000. It earned $70,000 on sales of
$450,000. Contribution margin as a percentage of sales is
a. 30%.
b. 40%.
c. 60%.
d. 70%.
c 50. Osceola Company earned $50,000 on sales of $400,000. It earned $70,000 on sales of
$450,000. Total fixed costs are
a. $
0.
b. $ 50,000.
c. $110,000.
20

d. $180,000.
True-False
F 1. The major variable cost in a manufacturing company is factory overhead.
T 2. In interpreting regression results, the higher the correlation, the better cost predictions are likely
to be.
F 3. Discretionary costs are step-variable.
F 4. Discretionary fixed costs are not necessary to successful operation of the business.
T 5. High-low, scatter diagram, and regression analysis are methods of developing formulas to
predict mixed costs.
F 6. As volume increases, the per-unit amount of a mixed cost increases.
T 7. In developing a cost-prediction equation using regression analysis, you might not select the one
with the highest correlation.
F 8. A company using activity-based costing need not do regression analysis or scatter diagrams.
F 9. An r-squared of .91 with a regression equation means that predictions will be accurate 91% of
the time.
T 10. A multiple regression equation uses more than one driver to predict costs.
Problems
1. Carlson Company incurred $170,000 in overhead costs making 12,000 units in March. It made
15,000 units and incurred $188,000 in overhead costs in April.
a. Compute the variable component of overhead cost.
b. Find the fixed factor of overhead cost.
SOLUTION:
a. $6 [($188,000 - $170,000)/(15,000 - 12,000)]
b. $98,000 [$170,000 - (12,000 x $6), or $188,000 - (15,000 x $6)]
2. The statistician of RST, Inc. has developed the following cost-prediction equation, using
observations from 12,000 to 30,000 machine hours.
Y = $236,837 + $3.7625X, r-squared = .81, standard error = $24,363
Y = total maintenance cost, X = machine hours
a. Find the predicted maintenance cost at 25,000 machine hours.
b. Will maintenance cost at zero machine hours be $236,837? yes no
answer.

Circle the correct

c. About 68% of the time, maintenance cost should be within what amount of the predicted value?
SOLUTION:
a. $330,900 ($236,837 + $3.7625 x 25,000)
b. No, zero is outside the relevant range.
c. $24,363, the standard error
21

3. Genner Company earned $125,000 on sales of $750,000.


$1,000,000.

It earned $225,000 on sales of

a. Find the variable costs as a percentage of sales.


b. Find the total fixed costs.
SOLUTION:
a. 60%
b. $175,000

{1 - [($225,000 - $125,000)/($1,000,000 - $750,000)]}


[$1,000,000 x (1 - 60%) - $225,000]

4. Danner has an average unit cost of $22.50 at a volume of 400,000 units. At 500,000 units the
average unit cost is $20.50.
a. Compute the variable cost per unit.
b. Compute the total fixed cost.
SOLUTION:
a. $12.50 {[($20.50 x 500,000) - ($22.50 x 400,000)]/(500,000 - 400,000)}
b. $4,000,000 ($9,000,000 - $12.50 x 400,000)
5. Tri-County Company incurred $175,000 in overhead costs making 40,000 units in April. It made
24,000 units and incurred $147,000 in overhead costs in May.
a. Compute the variable component of overhead cost.
b. Find the fixed factor of overhead cost.

SOLUTION:
a. $1.75 [($175,000 - $147,000)/(40,000 - 24,000)]
b. $105,000 [$147,000 - (24,000 x $1.75), or $175,000 - (40,000 x $1.75)]
6. Bilbo Company incurred $374,000 in overhead costs making 11,000 units in November. It made
7,500 units and incurred $325,000 in overhead costs in December.
a. Compute the variable component of overhead cost.
b. Find the fixed factor of overhead cost.
SOLUTION:
a. $14 [($374,000 - $325,000)/(11,000 - 7,500)]
b. $220,000 [$325,000 - (7,500 x $14), or $374,000 - (11,000 x $14)]
7. The statistician of Comstock, Inc. has developed the following prediction equation for costs, using
observations from 25,000 to 60,000 machine hours.
Y = $146,374 + $4.892X, r-squared = .86, standard error = $28,638
Y = total repair cost, X = machine hours
a. Find the predicted repair cost at 50,000 machine hours.
22

b. Will repair cost at zero machine hours be $146,374? yes no

Circle the correct answer.

c. About 68% of the time, repair cost should be within what amount of the predicted value?
SOLUTION:
a. $390,974 ($146,374 + $4.892 x 50,000)
b. No, zero is outside the relevant range.
c. $28,638, the standard error
8. Scooter Company earned $150,000 on sales of $1,000,000.
$1,400,000.

It earned $330,000 on sales of

a. Find the contribution margin ratio.


b. Find the total fixed costs.
SOLUTION:
a. 45%

($330,000 - $150,000)/($1,400,000 - $1,000,000)

b. $300,000

[$1,000,000 x 45%) - $150,000]

9. Bennco has an average unit cost of $18.50 at a volume of 100,000 units. At 200,000 units the
average unit cost is $14.25.
a. Compute the variable cost per unit.
b. Compute the total fixed cost.
SOLUTION:
a. $10.00

{[($14.25 x 200,000) - ($18.50 x 100,000)]/(200,000 - 100,000)}

b. $850,000 ($2,850,000 - $10 x 200,000)


10. Parsons Company incurred $475,000 in overhead costs making 40,000 units in August. It made
30,000 units and incurred $447,000 in overhead costs in September.
a. Compute the variable component of overhead cost.
b. Find the fixed factor of overhead cost.
SOLUTION:
a. $2.80 [($475,000 - $447,000)/(40,000 - 30,000)]
b. $363,000 [$447,000 - (30,000 x $2.80), or $475,000 - (40,000 x $2.80)]
CHAPTER 4: ACTIVITY-BASED COSTING AND MANAGEMENT
Multiple Choice
a 1. Activity-based costing
a. requires the identification of cost drivers.
b. is used only in JIT operations.
c. applies only to discretionary fixed costs.
d. does not help to identify activities as value-adding or non-value-adding.
23

a 2. A company using activity-based costing


a. tries to identify cost drivers.
b. allocates all costs to individual products.
c. looks for the activity with which total costs are most closely associated.
d. is probably using the JIT philosophy.
a 3. Machine setups is an example of a(n) ____________ activity.
Batch
Facility-sustaining
Product-sustaining
Unit
d 4. Machine hours is an example of a(n) ____________ activity.
a. Batch
Facility-sustaining
Product-sustaining
Unit
b 5. Landscaping is an example of a(n) _____________ activity.
Batch
Facility-sustaining
Product-sustaining
Unit
d 6. Material cost is an example of a(n) _____________ activity.
Batch
Facility-sustaining
Product-sustaining
Unit
c 7. The activities that drive resource requirements are called the
a. activity drivers
b. cost objects
c. resource drivers
d. sustaining activities
b 8. The resource utilized by a given product divided by the total amount of the resource available is
called the
activity driver
consumption ratio
cost object
sustaining activity
c 9. The segment for which you are estimating the cost is called the
activity driver
consumption ratio
cost object
sustaining activity
a 10. A tool that focuses on manufacturing processes and seeks to reduce or optimize the activities
performed within the process is
process value analysis
re-engineering
caveat analysis
benchmarking
d 11. A tool that compares how tasks are performed internally with the best practices of industry
leaders is
process value analysis
re-engineering
caveat analysis
benchmarking
b 12. An approach to developing new ways to perform existing activities is called
process value analysis
re-engineering
caveat analysis
benchmarking
24

c 13. Which of the following statements is true?


The traditional approach to costing uses many different cost drivers.
Costs that are indirect to products are by definition traceable to directly to products.
Costs that are indirect to products are traceable to some activity.
All of the above statements are true.
a 14. Which of the following is NOT a sign of poor cost data?
Competitors' prices for high-volume products appear much too high.
The company seems to have a highly profitable niche all to itself.
Customers don't balk at price increases for low-volume products.
Competitors' prices for low-volume products appear much too high.
d 15. Which of the following is a sign of poor cost data?
Competitors' prices for high-volume products appear much too high.
The company seems to have a highly profitable niche all to itself.
Customers don't balk at price increases for low-volume products.
All of the statements are true.
a 16. Number of purchase orders is an example of a(n) ____________ activity.
Batch
Facility-sustaining
Product-sustaining
Unit
d 17. Direct labor hours is an example of a(n) ____________ activity.
a. Batch
Facility-sustaining
Product-sustaining
Unit
b 18. Property taxes on the plant is an example of a(n) _____________ activity.
Batch
Facility-sustaining
Product-sustaining
Unit
d 19. Production volume is an example of a(n) _____________ activity.
Batch
Facility-sustaining
Product-sustaining
Unit
d 20. ____________ are those performed each time a unit is produced or sold.
Batch-level activities
Facility-sustaining activities
Sustaining activities
Unit-level activities
a 21. ____________ are those that a company performs when it makes a group of units.
Batch-level activities
Facility-sustaining activities
Sustaining activities
Unit-level activities
b 22. ____________ relate to an entire plant as a whole.
Batch-level activities
Facility-sustaining activities
Sustaining activities
Unit-level activities
c 23. ____________ arise because a company maintains a particular product or service.
Batch-level activities
Facility-sustaining activities
Sustaining activities
Unit-level activities
d 24. Which of the following is not a type of sustaining activity?
Capacity-sustaining
Customer-sustaining
25

Distribution-channel sustaining
Unit-sustaining
a 25. Which of the following is true regarding activity-based management?
ABM is using information about activities to manage portions of the organization other than costs.
ABM is applying ABC to external financial reporting.
ABM requires the use of re-engineering principles.
All of the above are true.
a 26. The quality costs that are incurred to determine whether particular units of product meet
quality standards are
a. appraisal costs.
b. external failure costs.
c. internal failure costs.
d. prevention costs.
c 27.
The cost of downtime on machines while rework is being performed is a(n)
a. appraisal cost.
b. external failure cost.
c. internal failure cost.
d. prevention cost.
b 28. The cost of processing customer complaints is a(n)
a. appraisal cost.
b. external failure cost.
c. internal failure cost.
d. prevention cost.
d 29. Worker training is a(n)
a. appraisal cost.
b. external failure cost.
c. internal failure cost.
d. prevention cost.
b 30. The cost to repair a unit of product that fails after it is sold is a(n)
a. appraisal cost.
b. external failure cost.
c. internal failure cost.
d. prevention cost.
b 31. Genco manufactures two versions of a product. Production and cost information show the
following:
Model A Model B
Units produced
200
400
Material moves (total)
20
80
Direct labor hours per unit 1
2
Material handling costs total $200,000. Under ABC, the material handling costs allocated to each
unit of Model A would be:
a. $10
b. $200
c. $333
d. Some other number
c 32. Genco manufactures two versions of a product. Production and cost information show the
following:
Model A Model B
Units produced
200
400
Material moves (total)
20
80
Direct labor hours per unit 1
2
Material handling costs total $200,000. Under ABC, the material handling costs allocated to each
unit of Model B would be:
a. $200
b. $333
c. $400
d. Some other number
26

a 33. Genco manufactures two versions of a product. Production and cost information show the
following:
Model A Model B
Units produced
200
400
Material moves (total)
20
80
Direct labor hours per unit 1
3
Material handling costs total $200,000. Direct labor hours are used to allocate overhead costs.
The material handling costs allocated to each unit of Model A would be:
a. $143
b. $200
c. $333
d. Some other number
b 34. Cadott Manufacturing produces three products. Production and cost information show the
following:
Model X
Model Y
Model Z
Units produced
1,000
3,000
6,000
Direct labor hours
2,000
1,000
2,000
Number of inspections
20
30
50
Inspection costs totaled $100,000. Using ABC, inspections costs allocated to each unit of Model X
would be
a. $10.00
b. $20.00
c. $40.00
d. Some other number
b 35. Cadott Manufacturing produces three products. Production and cost information show the
following:
Model X
Model Y
Model Z
Units produced
1,000
3,000
6,000
Direct labor hours
2,000
1,000
2,000
Number of inspections
20
30
50
Inspection costs totaled $100,000. Using ABC, inspections costs allocated to each unit of Model Y
would be
a. $ 6.67
b. $10.00
c. $20.00
d. Some other number
b 36. Cadott Manufacturing produces three products. Production and cost information show the
following:
Model X
Model Y
Model Z
Units produced
1,000
3,000
6,000
Direct labor hours
2,000
1,000
2,000
Number of inspections
20
30
50
Inspection costs totaled $100,000. Using ABC, inspections costs allocated to each unit of Model Z
would be
a. $ 6.67
b. $ 8.33
c. $10.00
d. Some other number
c 37. Cadott Manufacturing produces three products. Production and cost information show the
following:
Model X
Model Y
Model Z
Units produced
1,000
3,000
6,000
Direct labor hours
2,000
1,000
2,000
Number of inspections
20
30
50
27

Inspection costs totaled $100,000. Using direct labor hours as the allocation base, inspections
costs allocated to each unit of Model X would be
a. $10.00
b. $20.00
c. $40.00
d. Some other number
a 38. Cadott Manufacturing produces three products. Production and cost information show the
following:
Model X
Model Y
Model Z
Units produced
1,000
3,000
6,000
Direct labor hours
2,000
1,000
2,000
Number of inspections
20
30
50
Inspection costs totaled $100,000. . Using direct labor hours as the allocation base, inspections
costs allocated to each unit of Model Y would be
a. $6.67
b. $10.00
c. $20.00
d. Some other number
a 39. Cadott Manufacturing produces three products. Production and cost information show the
following:
Model X
Model Y
Model Z
Units produced
1,000
3,000
6,000
Direct labor hours
2,000
1,000
2,000
Number of inspections
20
30
50
Inspection costs totaled $100,000. Using direct labor hours as the allocation base, inspections
costs allocated to each unit of Model Z would be
a. $6.67
b. $8.33
c. $10.00
d. Some other number
b 40. Superior Inc. produces three products. Production and cost information is as follows:
Model Q
Model R
Model S
Units produced
2,000
6,000
12,000
Direct labor hours
4,000
2,000
4,000
Number of setups
100
150
250
The consumption ratios for number of setups would be:
Q R S
a. 40%-20%-40%
b. 20%-30%-50%
c. 10%-30%-60%
d. Some other numbers
c 41. Superior Inc. produces three products. Production and cost information is as follows:
Model Q
Model R
Model S
Units produced
2,000
6,000
12,000
Direct labor hours
4,000
2,000
4,000
Number of setups
100
150
250
The consumption ratios based on units produced would be:
Q R S
a. 40%-20%-40%
b. 20%-30%-50%
c. 10%-30%-60%
d. Some other numbers
a 42. Superior Inc. produces three products. Production and cost information is as follows:
Units produced

Model Q
Model R
Model S
2,000
6,000
12,000
28

Direct labor hours


Number of setups

4,000
100

2,000
150

4,000
250

The consumption ratios for direct labor hours would be:


Q R S
a. 40%-20%-40%
b. 20%-30%-50%
c. 10%-30%-60%
d. Some other numbers
c 43. Waupaca Company produces three products with the following production and cost information:
Model A
Model B
Model C
Units produced
2,000
6,000
12,000
Direct labor hours (total) 4,000
2,000
4,000
Number of setups
100
150
250
Number of shipments
200
225
275
Engineering change orders
15
10
5
Overhead costs include setups $90,000; shipping costs $140,000; and engineering costs
$180,000. What would be the per unit overhead cost for Model A if direct labor hours were the
allocation base?
a. $20.50
b. $41.00
c. $82.00
d. Some other number
b 44. Waupaca Company produces three products with the following production and cost information:
Model A
Model B
Model C
Units produced
2,000
6,000
12,000
Direct labor hours (total) 4,000
2,000
4,000
Number of setups
100
150
250
Number of shipments
200
225
275
Engineering change orders
15
10
5
Overhead costs include setups $90,000; shipping costs $140,000; and engineering costs
$180,000. What would be the per unit overhead cost for Model A if activity-based costing were used?
a. $20.50
b. $74.00
c. $82.00
d. Some other number
a 45. Waupaca Company produces three products with the following production and cost information:
Model A
Model B
Model C
Units produced
2,000
6,000
12,000
Direct labor hours (total) 4,000
2,000
4,000
Number of setups
100
150
250
Number of shipments
200
225
275
Engineering change orders
15
10
5
Overhead costs include setups $90,000; shipping costs $140,000; and engineering costs
$180,000. What would be the per unit overhead cost for Model B if activity-based costing were used?
a. $22.00
b. $66.00
c. $123.00
d. Some other number
a 46. Waupaca Company produces three products with the following production and cost information:
Model A
Model B
Model C
Units produced
2,000
6,000
12,000
Direct labor hours (total) 4,000
2,000
4,000
Number of setups
100
150
250
Number of shipments
200
225
275
Engineering change orders
15
10
5
Overhead costs include setups $90,000; shipping costs $140,000; and engineering costs
29

$180,000. What would be the per unit overhead cost for Model C if activity-based costing were used?
a. $10.83
b. $32.50
c. $245.28
d. Some other number
c 47. Kimball Company produces two products in a single factory. The following production and cost
information has been determined:
Model 1 Model 2
Units produced
1,000
200
Material moves (total)
100
40
Testing time (total)
250
125
Direct labor hours per unit 1
5
The controller has determined total overhead to be $480,000. $120,000 relates to material
moves; $150,000 relates to testing; the remainder is related to labor time.
If Kimball uses direct labor hours to allocate overhead to each model, what would overhead per
unit be for Model 1?
a. $10.00
b. $120.00
c. $240.00
d. $400.00
b 48. Kimball Company produces two products in a single factory. The following production and cost
information has been determined:
Model 1 Model 2
Units produced
1,000
200
Material moves (total)
100
40
Testing time (total)
250
125
Direct labor hours per unit 1
5
The controller has determined total overhead to be $480,000. $140,000 relates to material
moves; $150,000 relates to testing; the remainder is related to labor time.
If Kimball uses activity-based costing to allocate overhead to each model, what would overhead
per unit be for Model 1?
a. $400.00
b. $295.00
c. $240.00
d. $120.00
d 49. Kimball Company produces two products in a single factory. The following production and cost
information has been determined:
Model 1 Model 2
Units produced
1,000
200
Material moves (total)
100
40
Testing time (total)
250
125
Direct labor hours per unit 1
5
The controller has determined total overhead to be $480,000. $140,000 relates to material
moves; $150,000 relates to testing; the remainder is related to labor time.
If Kimball uses direct labor hours to allocate overhead to each model, what would overhead per
unit be for Model 2?
a. $158.33
b. $400.00
c. $950.00
d. $1,200.00
c 50. Kimball Company produces two products in a single factory. The following production and cost
information has been determined:
Model 1 Model 2
Units produced
1,000
200
Material moves (total)
100
40
30

Testing time (total)


250
Direct labor hours per unit 1

125
5

The controller has determined total overhead to be $480,000. $140,000 relates to material
moves; $150,000 relates to testing; the remainder is related to labor time.
If Kimball uses activity-based costing to allocate overhead to each model, what would overhead
per unit be for Model 2?
a. $158.33
b. $415.93
c. $925.00
d. Some other number
True-False
F 1. ABC can only be used in a company that produces a single product.
F 2. A company that uses only volume-based measures will overcost its low-volume products.
T 3. ABC will be most useful in estimating fixed costs.
T 4. Two major influences on costs are complexity and diversity.
T 5. Volume-based measures will tend to overcost high volume products.
F 6. Activities that drive resource requirements are known as activity drivers.
F 7. ABC is required for GAAP financial reporting.
F 8. ABC will benefit a JIT operation more than a non-JIT operation.
F 9. External failure costs are made up solely of opportunity costs.
F 10. The acceptable quality level is the point where internal failure costs are minimized.
Problems
1. Scottso Enterprises incurs $300,000 in manufacturing overhead costs each month. The company
has been allocating overhead to individual product lines based on direct labor hours.
Amount
Amount of
In Pool
Activity
---------------$100,000
25,000
150,000
200
50,000
125
-------Total overhead costs
$300,000
========
Two products have the following characteristics:
Cost driver
----------Direct labor hours
Number of batches
Design changes

Direct labor hours


Number of batches
Design changes

Product X
Product Y
----------------1,000
400
10
20
1
15

a. Determine the overhead to be allocated to each product using direct labor hours as the only cost
driver.
b. Determine the overhead to be allocated to each product using the three drivers identified.
SOLUTION:
Product X: $12,000 [1,000 x ($300,000/15,000)]
Product Y: $ 4,800 [400 x ($300,000/15,000)]
31

Product X: $11,900; Product Y: $22,600


Cost driver
----------Direct labor hours
Number of batches
Design changes

Direct labor hours,


Number of batches
Design changes

Amount
Amount of
In Pool
Activity
Rate
----------------------$100,000
25,000
$ 4.00
150,000
200
750.00
50,000
125
400.00
Product X
Product Y
----------------$4,000
$1,600
7,500
15,000
400
6,000
----------$11,900
$22,600

2. Lewis Company has two major segments with the following information:
Upstate
Downstate
Total_
Annual revenue
$200,000
$600,000
$800,000
Annual salesperson salaries
$30,000
$45,000
$75,000
Number of customers
50
75
125
Miles driven
80,000
40,000
The business also has overhead costs as follows:
Cost pool
Travel
Entertainment
Administrative
Total

Cost in pool Cost driver__________


$ 36,000
miles driven
144,000
number of customers
150,000
salaries
-------$330,000

Allocate the overhead costs to


Determine the income of each
Allocate the overhead costs to
Determine the income of each

the segments based on sales revenue.


segment.
the segments using ABC.
segment under ABC.

SOLUTION:
Upstate: $82,500 [$330,000 x ($200,000/$800,000)]
Downstate: $247,500 [$330,000 x ($600,000/$800,000)]
Upstate: $87,500 [$200,000 - $30,000 - $82,500]
Downstate: $307,500 [$600,000 - $45,000 - $247,500]
c. & d.
Upstate
Downstate
Total_
Annual revenue
$200,000
$600,000
$800,000
Annual salesperson salaries
30,000
45,000
75,000
Travel
24,000
12,000
36,000
Entertainment
57,600
86,400
144,000
Administrative
60,000
90,000
150,000
------------------Income
$ 28,400
$366,600
$395,000
3. Johnson & Mathew is an architectural and landscape services firm. The firm operates in three major
segments. The following information has been obtained,
Annual revenues
Number of jobs
Chargeable hours

New Design Remodel Landscape


Total__
$500,000 $1,200,000 $300,000 $2,000,000
50
150
200
400
6,000
10,000
14,000
30,000

Salaries for the year were $800,000; overhead for the year was $1,000,000.
32

Determine the profits for each segment, assuming costs are allocated based on annual revenues.
Determine the profits for each segment, assuming costs are allocated based on the number of jobs.
Determine the profits for each segment, assuming costs are allocated based on chargeable hours.
SOLUTION:
New Design: $50,000 [$500,000 - ($1,800,000 x $500,000/$2,000,000)]
Remodel: $120,000 [$1,200,000 - ($1,800,000 x $1,200,000/$2,000,000)]
Landscape $30,000 [$300,000 - ($1,800,000 x $300,000/$2,000,000)]
New Design: $275,000 [$500,000 - ($1,800,000 x 50/400)]
Remodel: $525,000 [$1,200,000 - ($1,800,000 x 150/400)]
Landscape $(600,000) [$300,000 - ($1,800,000 x 200/400)]
New Design: $140,000 [$500,000 - ($1,800,000 x 6,000/30,000)]
Remodel: $600,000 [$1,200,000 - ($1,800,000 x 10,000/30,000)]
Landscape $(540,000) [$300,000 - ($1,800,000 x 14,000/30,000)]
4. Coleman Company produces three products, X, Y, & Z. The income statement for the firm as a
whole is:
Sales
$1,850,000
Less variable costs
1,310,000
---------Contribution margin
$ 540,000
Less fixed costs
Selling
330,000
Administrative 180,000
------510,000
---------Net income
$ 30,000
The sales, variable costs, and line-sustaining fixed costs for the four products are:
Q___
R___
S_ __
Sales
$250,000 $400,000 $1,200,000
Variable costs
60%
65%
75%
Line-sustaining costs $90,000 $130,000 $ 230,000
Prepare an income statement segmented by product line, including a column for the entire firm. Be
sure to show segment income as well as total enterprise income.
SOLUTION:
Q___
R___
S ___
Total _
$250,000 $400,000 $1,200,000 $1,850,000
150,000 260,000
900,000 1,310,000
------- ------- --------- --------Contribution margin $100,000 $140,000 $ 300,000 $ 540,000
Line-sustaining costs 90,000 130,000
230,000
450,000
------- ------- --------- --------Segment income
$ 10,000 $ 10,000 $ 70,000 $ 90,000
Company-sustaining
60,000
--------Enterprise income
$ 30,000
Sales
Variable costs

5. Ellington & Hodges is a public relations firm. The firm operates in three major segments. The
following information has been obtained,
Political Corporate Celebrity
Total__
Annual revenues
$750,000 $800,000 $1,450,000 $3,000,000
Number of clients
50
150
200
400
Chargeable hours
6,000
10,000
14,000
30,000
Total salaries for the year were $900,000; overhead was $600,000.
Determine the profits for each segment, assuming costs are allocated based on annual revenues.
33

Determine the profits for each segment, assuming overhead costs are allocated based on the number
of clients and salaries are allocated based on chargeable hours.
SOLUTION:
Political: $375,000 [$750,000 - ($1,500,000 x $750,000/$3,000,000)]
Corporate: $400,000 [$800,000 - ($1,500,000 x $800,000/$3,000,000)]
Celebrity: $725,000 [$1,450,000 - ($1,500,000 x $1,450,000/$3,000,000)]
b.

Political Corporate Celebrity


Total__
Annual revenues
$750,000 $800,000 $1,450,000 $3,000,000
Salaries
180,000
300,000
420,000
900,000
Chargeable hours
120,000
200,000
280,000
600,000
-------- -------- --------- --------Profits
$450,000 $300,000 $ 750,000 $1,500,000

6. Danner Company incurs $1,600,000 in manufacturing overhead costs each month. The company
has been allocating overhead to individual product lines based on direct labor hours.
Amount
Amount of
In Pool
Activity
---------------$500,000
40,000
700,000
1,000
400,000
500
---------Total overhead costs
$1,600,000
==========

Cost driver
----------Direct labor hours
Number of setups
Number of tests

Two products have the following characteristics:

Direct labor hours


Number of setups
Number of tests

Product S
Product T
----------------2,000
1,000
20
100
2
150

a. Determine the overhead to be allocated to each product using direct labor hours as the only cost
driver.
b. Determine the overhead to be allocated to each product using the three drivers identified.
SOLUTION:
Product S: $80,000 [2,000 x ($1,600,000/40,000)]
Product T: $40,000 [1,000 x ($1,600,000/40,000)]
Product S: $40,600; Product T: $202,500
Cost driver
----------Direct labor hours
Number of setups
Number of tests

Direct labor hours,


Number of setups
Number of tests

Amount
Amount of
In Pool
Activity
Rate
----------------------$500,000
40,000
$ 12.50
700,000
1,000
700.00
400,000
500
800.00
Product S
Product T
----------------$25,000
$12,500
14,000
70,000
1,600
120,000
-----------$40,600
$202,500

7. Seneca Company has two products with the following information:

Annual revenue
Material costs

Engine
Race
Rebuilds
Cars
Total__
$1,200,000
$1,400,000 $2,600,000
400,000
$600,000 $1,000,000
34

Labor costs
Number of receipts
Number of batches

250,000
8,000
425

150,000
2,000
75

$400,000

The business also has overhead costs as follows:


Cost pool
Receiving
Material moves
Administrative
Total

Cost in pool Cost driver__________


$300,000
number of receipts
275,000
number of batches
225,000
labor cost
-------$800,000

Allocate the overhead costs to


Determine the income of each
Allocate the overhead costs to
Determine the income of each

the segments based on material costs.


segment.
the segments using ABC.
segment under ABC.

SOLUTION:
Engines: $320,000 [$800,000 x ($400,000/$1,000,000)]
Race Cars: $480,000 [$800,000 x ($600,000/$1,000,000)]
Engines: $230,000 [$1,200,000 - $400,000 - $250.000 - $320,000]
Race Cars: $170,000 [$1,400,000 - $600,000 - $150,000 - $480,000]
c. & d.
Annual revenue
Material costs
Labor costs
Receiving
Material moves
Administrative
Income

Engine
Race
Rebuilds
Cars
Total__
$1,200,000
$1,400,000 $2,600,000
400,000
600,000
1,000,000
250,000
150,000
400,000
240,000
60,000
300,000
233,750
41,250
275,000
140,625
84,375
225,000
-------------------$(64,375)
$464,375
$ 400,000

8. Ventura Company produces four products, Q, R, S & T. The income statement for the firm as a
whole is:
Sales
$3,000,000
Less variable costs
1,870,000
--------Contribution margin
$1,130,000
Less fixed costs
Manufacturing $320,000
Selling
290,000
Administrative 168,000
------778,000
--------Net income
$ 352,000
The sales, contribution margin ratios, and line-sustaining fixed costs for the four products are:
Q___
R___
S ___
T_ __
Sales
$200,000 $400,000 $1,000,000 $1,400,000
Contribution margin
35%
40%
20%
50%
Line-sustaining costs
$60,000
$80,000
$168,000
$270,000
Prepare an income statement segmented by product line, including a column for the entire firm. Be
sure to show segment income as well as total enterprise income.
SOLUTION:
Sales

Q___
R___
S_ __
T_ __
Total__
$200,000 $400,000 $1,000,000 $1,400,000 $3,000,000
35

Variable costs

130,000 240,000
800,000
700,000 1,870,000
------- ------- --------- --------- --------Contribution margin
$ 70,000 $160,000 $ 200,000 $ 700,000 $1,130,000
Line-sustaining costs
60,000 80,000
168,000
270,000
578,000
------- ------- --------- --------- --------Segment income
$ 10,000 $ 80,000 $ 32,000 $ 430,000 $ 552,000
Company-sustaining
200,000
--------Enterprise income
$ 352,000
9.

DJH Corp has identified the following costs.


Costs to improve the production process $175,000
Incoming inspection costs
110,000
Design engineer salaries
325,000
Rework of defective units
122,000
Salaries for service repair workers
265,000
Warranty costs
335,000
Finished goods inspection
165,000
Scrap
118,000

Categorize each of the costs into the appropriate quality cost category and prepare a total for
each.
SOLUTION:
Prevention costs:
Costs to improve the production process $175,000
Design engineer salaries
325,000
Total prevention costs
$500,000
Appraisal costs:
Incoming inspection costs
Finished goods inspection
Total appraisal costs

$110,000
165,000
275,000

Internal failure costs:


Rework of defective units
$122,000
Scrap
118,000
Total internal failure costs
240,000
External failure costs:
Salaries for service repair workers
$265,000
Warranty costs
335,000
Total external failure costs
600,000
10. Following is a list of activities that pertains to quality. Classify each as either external failure,
internal failure, appraisal, or prevention.
External Internal
Failure Failure Appraisal Prevention

In-process inspection
Warranty expenses
Worker training
Downtime on machinery due
to rework
Product returns
Product design
Preventive maintenance
Wages for field repair
workers
Quality laboratory
Customer complaints

SOLUTION:
36

External Internal
Failure Failure Appraisal Prevention
In-process inspection
X
Warranty expenses
X
Worker training
X
Downtime on machinery due
to rework
X
Product returns
X
Product design
X
Preventive maintenance
X
Wages for field repair
Workers
X
Quality laboratory
X
Customer complaints
X
CHAPTER 5: Short-Term Decisions and Accounting Information
Multiple Choice
a 1. The salary or wage that you could be earning while you are taking this test is
a. an opportunity cost.
b. a sunk cost.
c. an incremental cost.
d. a joint cost.
d 2. The kind of cost that can be ignored in short-term decision making is
a. a differential cost.
b. an opportunity cost.
c. a relevant cost.
d. a sunk cost.
b 3. The role of sunk costs in decision making can be summed up in which of the following sayings?
a. Nothing ventured, nothing gained.
b. Bygones are bygones.
c. A penny saved is a penny earned.
d. The love of money is the root of all evil.
c 4. Allocated costs are
a. generally separable.
b. generally variable.
c. generally common.
d. especially important in deciding whether to drop a segment.
a 5. The Robinson-Patman Act forbids charging different prices to different customers unless
a. the price differences are justified by differences in distribution costs.
b. prices offered competitors are fully disclosed to all customers.
c. such prices produce profits no greater than normal profits.
d. the customers are located within the seller's state.
c 6. A product should be dropped if
a. it has a negative incremental profit.
b. it has a negative contribution margin.
c. dropping it will increase the total profit of the company.
d. it is not essential to the company's product line.
a 7. From its refining process an oil company obtains three products, one of which can be processed
further into a different product, the other two of which can be sold after further refining. The refining
process is
a. a joint process.
b. a mixed cost process.
c. an unavoidable process.
d. a process whose costs should be allocated to the resulting products.
b 8. The Accessories Department shows sales of $35,000. Variable costs are $30,000 and allocated
unavoidable fixed costs are $9,000, leaving a $4,000 loss. Based on this information and all other
things equal,
a. the department contributes $35,000 to total profits.
37

b. dropping the department will reduce total company profits by $5,000.


c. the department should be closed.
d. the department should be kept only if unit volume can be increased enough to increase sales
by $4,000.
d 9. A major factor in evaluating a special order is
a. the expected contribution margin on the order.
b. the possible effects on sales at regular prices.
c. the availability of capacity to produce the additional units.
d. all of the above.
a 10. Which of the following is true for a make-or-buy decision?
a. The reliability of the outside supplier of the component is important to the decision.
b. Depreciation on equipment used in making the component and having no other use is the
critical factor in the decision.
c. Opportunity costs are irrelevant.
d. The company should make the component if the purchase price is less than the per-unit
variable cost to make the component.
b 11. Which of the following costs is relevant in deciding whether to sell joint products at split-off or
process them further?
a. The unavoidable costs of further processing.
b. The avoidable costs of further processing.
c. The variable cost of operating the joint process.
d. The cost of materials used to make the joint products.
b 12. A manufacturing process that invariably produces two or more products
a. is a complementary process.
b. is a joint process.
c. normally has only fixed costs.
d. usually has primarily variable costs.
d 13. Which of the following is a short-term decision in which opportunity costs are not relevant?
a. Make-or-buy decision.
b. Special-order decision.
c. Drop-a-segment decision.
d. None of the above.
c 14.
Which of the following is a true statement?
Theory of Constraints is useful for identifying physical constraints but cannot incorporate nonphysical
constraints.
Theory of Constraints is useful in analyzing internal constraints but cannot identify external
constraints.
Constraints may be either internal or external.
Internal constraints are physical while external constraints are imaginary.
b 15. A company has space that it uses to make a component. It could rent the space to another
company. The rent is
a. a sunk cost. \
b. an opportunity cost.
c. a joint cost.
d. an avoidable cost.
a 16. Escanaba Company has 200 units of an obsolete component. The variable cost to produce them
was $10 per unit. They could now be sold for $1.75 each and it would cost $7.60 to make them now. If
the units could be used to make a product for a special order, their relevant cost is
a. $ 1.75.
b. $ 7.60.
c. $10.00.
d. some other number.
c 17. In a make-or-buy decision, which of the following is true?
a. Variable costs are the only relevant costs.
b. Allocated fixed costs are relevant.
c. Alternative uses of space and machinery are relevant.
d. Making is the correct decision when there is idle capacity.
a 18. The best characterization of an opportunity cost is that it is
a. relevant to decision making but is not usually reflected in accounting records.
b. not relevant to decision making and is not usually reflected in accounting records.
38

c. relevant to decision making and is usually reflected in accounting records.


d. not relevant to decision making and is usually reflected in accounting records.
a 19. A sunk cost is
a. not avoidable.
b. avoidable under one alternative but not under another.
c. joint or common.
d. direct to a segment.
b 20. Differential costs are costs that are
a. not avoidable.
b. avoidable under one alternative but not under another.
c. joint or common.
d. not direct to a segment.
a 21. A common cost
a. relates to a process that produces more than one product.
b. should be allocated to the segments of a company.
c. can usually be avoided in its entirety by dropping a segment.
d. none of the above.
a 22. An opportunity cost commonly associated with a special order is
a. the contribution margin on lost sales.
b. the variable costs of the order.
c. additional fixed costs related to the increased output.
d. any of the above.
b 23. Which of the following statements pertaining to the Theory of Constraints is true?
Inventory is evil and should never be kept.
Inventory is important to keep immediately before a bottleneck process.
Inventory should be kept before every machining process to prevent any downtime.
None of the above are true.
c 24. Which of the following cost-classification schemes is most relevant to decision making?
a. Fixed--variable.
b. Joint--common
c. Avoidable--unavoidable.
d. Direct--common.
d 25. Which of the following is NOT relevant in deciding whether to process a joint product beyond its
split-off point?
a. The split-off value.
b. The price after additional processing.
c. The cost of further processing.
d. The cost of operating the joint process.
c 26. Benson Company has 200 units of an obsolete part. The variable cost to produce them was $4
per unit. They could now be sold for $3 each and it would cost $6 to make them now. The parts could
be reworked for $8 each and sold for $17. What is the monetary advantage of reworking the parts
over the next-best action?
a. $ 600.
b. $1,000.
c. $1,200.
d. $2,000.
b 27. Pueblo Company sells a product for $60. Variable cost is $32. Pueblo could accept a special
order for 1,000 units at $46. If Pueblo accepted the order, how many units could it lose at the regular
price before the decision became unwise?
a. 1,000.
b. 500.
c. 200.
d. 0.
c 28. Which of the following is NOT a short-term decision?
a. Accept a special order.
b. Make-or-buy a component.
c. Replace a machine.
d. Sell a joint product at split-off or process it further.
39

d 29. The variable cost of a unit of product made yesterday is


a. an incremental cost.
b. an opportunity cost.
c. a differential cost.
d. a sunk cost.
d 30. The most profitable use of a resource that has limited capacity and is needed in the production
of more than one product is a function of which of the following?
a. The number of units of each product the company can sell.
b. The contribution margin of each product.
c. The amount of resource-use required for each unit of each product.
d. All of the above.
c 31. Which of the following is NOT relevant in a make-or-buy decision about a part the entity uses in
some of its products?
a. The reliability of the outside supplier.
b. The alternative uses of owned equipment used to make the part.
c. The outside supplier's per-unit variable cost to make the part.
d. The number of units of the part needed each period.
d 32. Which of the following is NOT relevant to a decision about whether to drop a segment?
a. The contribution margin expected to be produced by the segment.
b. The avoidable fixed costs direct to that segment.
c. The complementary effects of dropping the segment.
d. "None of the above" is the best answer because all of the above are relevant.
a 33. Just-in-time manufacturers are less likely than conventional manufacturing companies to
a. operate a joint process that results in joint products.
b. be able to accommodate special orders.
c. have constraints on their productive capacity.
d. fit any of the above characterizations.
d 34. The total demand for Product Z is relevant to a decision about
a. the best use of a resource that is in limited supply and is used in the production of Product Z
and one other of the company's products.
b. whether to sell Product Z, a joint product, at split-off or process it further into another salable
product.
c. replacing Product Z with another product.
d. all of the above decisions.
c 35. Buchanan Company currently sells 4,000 units of product Q for $1 each. Capacity is 5,000 units.
Variable costs are $0.40 and avoidable fixed costs are $400. A chain store has offered $0.80 per unit
for 400 units of Q. If Buchanan accepts the order, the change in income will be a
a. $60 decrease.
b. $80 decrease.
c. $160 increase.
d. $480 increase.
a 36. Tyler Company currently sells 1,000 units of product M for $1 each. Variable costs are $0.40 and
avoidable fixed costs are $400. A discount store has offered $0.80 per unit for 400 units of product M.
The managers believe that if they accept the special order, they will lose some sales at the regular
price. Determine the number of units they could lose before the order became unprofitable.
a. 267 units
b. 500 units
c. 600 units
d. Some other number.
b 37. Bear Valley produces three products: A, B, and C. One machine is used to produce the products.
The contribution margins, sales demands, and time on the machine (in minutes) are as follows:
time on
Demand
CM
machine
---------------A
100
$25
10
B
80
18
5
C
150
30
10
There are 2400 minutes available on the machine during the week. How many units should be
produced and sold to maximize the weekly contribution?
A B C
40

a. 100
b. 50
c. 90
d. 100

80 150
80 150
0 150
80 100

d 38. Elk Grove produces three products: A, B, and C. A machine is used to produce the products.
The contribution margins, sales demands, and time on the machine (in minutes) are as follows:
time on
Demand
CM
machine
---------------A
120
$20
5
B
80
36
10
C
100
50
15
There are 2400 minutes available on the machine during the week. How many units should be
produced and sold to maximize the weekly contribution?
A B C
a. 120 80 100
b. 20 80 100
c. 120 30 100
d. 120 80 66
a 39. Black Oak Company makes and sells oak boxes for a price of $60 each. Unit costs based on
anticipated monthly sales of 1,000 boxes are as follows:
Direct material cost
$15
Direct labor cost
12
Variable manufacturing overhead
Variable selling overhead
5
Fixed costs
2

A chain store has offered to buy 100 boxes per month at $58 each. To accept this special order,
Black Oak will have to restrict its sales to regular customers to only 900 boxes per monthly because
its production capacity cannot be expanded in the short run. However, no variable selling expenses
will be incurred for this special order. If Black Oak accepts the chain store's offer, its profit will
a. increase by $300.
b. increase by $500.
c. decrease by $200.
d. decrease by $500.
a 40. Medford Corporation operates a plant with a productive capacity to manufacture 20,000 units of
its product a year. The follow information pertains to the production costs at capacity:
Variable costs $160,000
Fixed costs
240,000
-------Total costs
$400,000
=========
A supplier has offered to sell 4,000 units to Medford annually. Assume no change in the fixed
costs. What is the price per unit that makes Medford indifferent between the "make" and "buy"
options?
a. $8
b. $12
c. $20
d. $0
b 41. DJH Company produces 1,000 units of Part X per month. The total manufacturing costs of the
part are as follows:
Direct materials
$10,000
Direct labor
15,000
Variable overhead
5,000
Fixed overhead
30,000
------Total manufacturing cost $60,000
=======
An outside supplier has offered to supply the part at $40 per unit. It is estimated that 20% of the
41

fixed overhead assigned to Part X will no longer be incurred if the company purchases the part from
the outside supplier. If DJH Company purchases 1,000 units of Part X from the outside supplier per
month, then its monthly operating income will
a. decrease by $20,000.
b. decrease by $4,000.
c. not change.
d. increase by $20,000.
c 42. DJH Company produces 1,000 units of Part X per month. The total manufacturing costs of the
part are as follows:
Direct materials
$10,000
Direct labor
15,000
Variable overhead
5,000
Fixed overhead
30,000
------Total manufacturing cost $60,000
=======
An outside supplier has offered to supply the part at $40 per unit. It is estimated that 20% of the
fixed overhead assigned to Part X will no longer be incurred if the company purchases the part from
the outside supplier. What is the maximum price that DJH Company should be willing to pay the
outside supplier?
a. $60
b. $40
c. $36
d. $25
c 43. Scooter Company produces three products from a joint process costing $100,000. The following
information is available:
Costs to Selling Price
Selling Price Process After Further
Units at Split-off Further Processing
----- ------------ ------- ---------A 10,000
$35
$70,000
$40
B 20,000
$40
$30,000
$45
C 30,000
$20
$90,000
$25
Which products should be processed further?
a. A only.
b. A and B.
c. B and C.
d. A, B, and C.
b 44. Genco Company produces three products from a joint process costing $100,000. The following
information is available:
Costs to Selling Price
Selling Price Process After Further
Units at Split-off Further Processing
----- ------------ ------- --------A 2,000
$25
$60,000
$50
B 3,000
$30
$60,000
$45
C 5,000
$40
$80,000
$60
Which products should be sold without further processing?
a. B only.
b. A and B.
c. B and C.
d. A, B, and C.
c 45. Colfax Company expects to incur the following costs at the planned production level of 10,000
units:
Direct materials $100,000
Direct labor
120,000
Variable overhead 60,000
Fixed overhead
30,000
42

The selling price is $50 per unit. The company currently operates at full capacity of 10,000 units.
Capacity can be increased to 13,000 units by operating overtime. Variable costs increase by $14 per
unit for overtime production. Fixed overhead costs remain unchanged when overtime operations
occur. Colfax Company has received a special order from a wholesaler who has offered to buy 1,000
units at $45 each. What is the incremental cost associated with this special order?
a. $14,000
b. $28,000
c. $42,000
d. $45,000
b 46. Colfax Company expects to incur the following costs at the planned production level of 10,000
units:
Direct materials $100,000
Direct labor
120,000
Variable overhead 60,000
Fixed overhead
30,000
The selling price is $50 per unit. The company currently operates at full capacity of 10,000 units.
Capacity can be increased to 13,000 units by operating overtime. Variable costs increase by $14 per
unit for overtime production. Fixed overhead costs remain unchanged when overtime operations
occur. Colfax Company has received a special order from a wholesaler who has offered to buy 1,000
units at $45 each. What is the impact on Colfax's operating income if this special order is accepted?
a. $17,000 increase
b. $3,000 increase
c. no change
d. $5,000 decrease
c 47. GMH Company manufactures 100,000 units of Part X annually for use in one of its main
products. The total manufacturing cost for 100,000 units of Part X is as follows:
Direct materials $120,000
Direct labor
80,000
Variable overhead 40,000
Fixed overhead
160,000
------Total cost
$400,000
========
Selin Company has offered to sell GMH 100,000 units of Part X per year. If GMH accepts this offer, the
facilities used to produce Part X can be used in the production of other components. This change
would save GMH $10,000 in rent for the leased production facility used at present to support the
production of other components. What is the amount of relevant costs for this make-or-buy decision?
a. $200,000
b. $240,000
c. $250,000
d. $400,000
d 48. GMH Company manufactures 100,000 units of Part X annually for use in one of its main
products. The total manufacturing cost for 100,000 units of Part X is as follows:
Direct materials $120,000
Direct labor
80,000
Variable overhead 40,000
Fixed overhead
160,000
-------Total cost
$400,000
========
Sutton Company has offered to sell GMH 100,000 units of Part X per year. If GMH accepts this
offer, the facilities used to produce Part X can be used in the production of other components. This
change would save GMH $10,000 in rent for the leased production facility used at present to support
the production of other components. What is the maximum price that GMH should be willing to pay
Sutton for part X?
a. $1.20
b. $2.00
c. $2.40
d. $2.50
43

d 49. Barrie, Inc., produces three products: A, B, and C. Two machines are used to produce the
products. The contribution margins, sales demands, and time on each machine (in minutes) is as
follows:
time
time
Demand
CM
on M1
on M2
A
100
$12
5
10
B
80
18
10
5
C
100
25
15
5
There are 2,400 minutes available on each machine during the week. How many units should be
produced and sold to maximize the weekly contribution?
A B C
a. 100 80 100
b. 20 80 100
c. 100 40 100
d. 100 80 73
b 50. Barrie, Inc., produces three products: A, B, and C. Two machines are used to produce the
products. The contribution margins, sales demands, and time on each machine (in minutes) is as
follows:
time
time
Demand
CM
on M1
on M2
A
100
$12
5
10
B
80
18
10
5
C
150
25
5
10
There are 2,400 minutes available on each machine during the week. How many units should be
produced and sold to maximize the weekly contribution?
A B C
a. 100 80 150
b. 50 80 150
c. 90 0 150
100 80 100
True-False
F 1. If the results of a decision are not as good as expected, there has been an error in the decisionmaking process.
T 2. The costs of operating a joint process can be fixed or variable.
T 3. Incremental costs can be either fixed or variable.
T 4. In general, the smaller the segment being considered in a decision, the fewer the avoidable
costs.
T 5. A given fixed cost might be separable and relevant for the purpose of one decision and common
and irrelevant for the purpose of another decision in the same company.
F 6. Opportunity cost is usually the amount paid for a resource.
T 7. Fixed costs that are allocated to several segments are normally irrelevant to decisions for one of
those segments.
T 8. The only revenues or costs that are relevant in decision making are the differential revenues or
costs.
T 9. Constraints may be internal to the firm or external to the firm.
T 10. Management's objective should be to exploit a constraint rather than to eliminate it.
Problems
1. Wilson Company expects the following results, without considering any of the changes described
below.
Product A Product B Total
--------- --------- ----44

Sales
Variable costs

$100
$300
$400
40
100
140
---------Contribution margin
$ 60
$200
$260
Fixed costs - avoidable
(20)
(30)
(50)
- unavoidable (50)
(100)
(150)
---------Profit (loss)
$(10)
$ 70
$ 60
=====
====
====
The unavoidable costs are allocated based on unit sales of 1,000 A and 2,000 B. CONSIDER EACH
QUESTION INDEPENDENTLY UNLESS TOLD OTHERWISE.
a. Compute Wilson's income if product A is dropped.
b. If product A were dropped and the unit sales of product B increased by 30%, what would the
company's income be?
c. Product A can be dropped and replaced with a new product, C, which would have avoidable fixed
costs of $50. Product C would sell for $0.60, have variable costs of $0.20, and expected volume of
400 units. Compute Wilson's income if A were replaced by C.
d. Suppose now that products A and B are joint products that are being sold at split-off. All of the
costs shown on the income statement are the materials, labor, and overhead of the joint process.
Find income if product B were processed further at additional costs of $90 and sold for $350.
SOLUTION:
a. Wilson's income: $20 ($200 CM from B - $30 - $150)
b. Wilson's income: $80 ($260 CM from B - $30 - $150)
c. Wilson's income: $130 [$20 + 400($0.60 - $0.20) - $50]
d. Wilson's income: $20 ($60 CM from A + $50 incremental revenue
$90 incremental cost)
2. Arapahoe Corp. can make three products from a joint process.
process is $10,000. Following are data about the three products.

Product
------A
B
C

The monthly cost of the joint

Sales Value
Sales Value
Costs of
at
if Further
Additional
Split-off Point Processed
Processing
--------------- ----------- ---------$ 8,000
$12,000
$2,500
$ 9,000
$10,000
$2,000
$
0
$ 2,500
$1,000

a. Which product(s) should be sold at the split-off point?


b. Arapahoe is currently processing all three products rather than selling any of them at the split-off
point. Find its current income.
SOLUTION:
a. Product B should be sold at split-off.
b. Income: $9,000 ($12,000 + $10,000 + $2,500 - $2,500 - $2,000 - $1,000 - $10,000)
3. Madison Co. operates a joint process. Three products, B, C, and D emerge from that process, each
of which can be sold immediately or processed further. Monthly output is 50,000 gallons; 50% is B,
30% is C, and 20% is D. You have the following information.
B
C
D
------- ------- ------Per-gallon split-off price
$8
$9
$6
45

Per-gallon price after further


processing
$13
$15
$12
Per-gallon variable cost of
further processing
$4
$2
$4
Avoidable direct fixed costs of
further processing, per month
$35,000 $45,000 $18,000
Unavoidable direct fixed costs
of further processing, per month $18,000 $40,000 $ 7,000
Which product(s), if any, should be sold at split-off?

SOLUTION:
B should be sold at split-off.
B: [50,000 x 50%] x [$13 - $8 - $4] - $35,000 = -$10,000
C: [50,000 x 30%] x [$15 - $9 - $2] - $45,000 = $15,000
D: [50,000 x 20%] x [$12 - $6 - $4] - $18,000 = $2,000
4. Milton Company has three products: A, B, and C. Three machines are used to produce the products.
The contribution margins, sales demands, and time on each machine (in minutes) is as follows:

A
B
C

time
time
time
Demand
CM
on M1
on M2
on M3
100
$45
10
15
12
80
$30
10
5
8
60
$40
5
10
5

There are 2,400 minutes available on each machine during the week. All materials needed are
readily available on a just-in-time basis.
What are the load factors for each of the three machines?
Which machine is the bottleneck?
How many units of A, B, and C should be produced during the week?
SOLUTION:
M1: 87.5% [(10 x 100) + (10 x 80) + (5 x 60)]/2,400
M2: 104.2% [(15 x 100) + (5 x 80) + (10 x 60)]/2,400
M3: 89.2% [(12 x 100) + (8 x 80) + (5 x 60)]/2,400
The bottleneck is M2 as identified by the load factor exceeding 100%
c.

A: 93, B: 80, C: 60
A: $45/15 = $3
B: $30/5 = $6
C: $40/10 = $4
Time available
2,400
Produce B first 80 x 5 = 400
Time remaining
2,000
Produce C: 60 x 10 =
600
Time remaining
1,400
Produce A 93 x 15
1,395
Time remaining
5

5. LaCrosse Company expects the following results, without considering any of the changes described
below.
Product A Product B Total
--------- --------- -----Sales
$1,000
$3,000
$4,000
Variable costs
400
1,000
1,400
------------Contribution margin
$ 600
$2,000
$2,600
Fixed costs - avoidable
(200)
(300)
(500)
46

- unavoidable (500)
(1,000) (1,500)
-------------Profit (loss)
$ (100)
$ 700
$ 600
=====
======
======
The unavoidable costs are allocated based on unit sales of 1,000 A and 2,000 B. An exporter has
offered $0.80 per unit for 200 units of A.
a. Find the change in income if LaCrosse accepts the o rder, assuming no loss of regular sales.
b. The managers believe that if they accept the special order, they will lose some sales at the
regular price. Determine the number of units they could lose before the order became unprofitable.
c. The managers believe that they will lose 80 units at the regular price if they accept the order.
Calculate the price they must charge for the special order to increase income by $50.
SOLUTION:
a. Change in income: $80 increase [200 x ($0.80 - $0.40 variable cost per unit)]
b. Sales to lose: 133 units [$80/($1.00 - $0.40)] = 133
c. Price: $0.89
Lost contribution margin (80 x $0.60)
$48.0
Desired profit
50.0
----Contribution margin required from special order $98.0
Divided by 20. units
200
----Equals contribution margin per unit
$0.49
Plus variable cost
0.40
----Equals required price
$0.89
=====
6. Mays Company manufactures 200,000 units of part XYZ annually. The following information has
been collected:
Materials
$200,000
Direct labor
110,000
Variable overhead
50,000
Fixed overhead
100,000
-------Total costs
$460,000
========
Clemens Company has offered to provide part XYZ for $2 per unit. Assume no other productive use
of the space exists.
a. What would be the dollar impact if Mays accepted the offer?
b. What is the maximum price Mays is willing to pay for the part?
SOLUTION:
a. $40,000 less profits ($360,000 make - $400,000 buy)
Cost to buy the part: 200,000 x $2 = $240,000
Cost to make the part: $200,000 + 110,000 + 50,000 = $360,000
b. $1.80 ($360,000/200,000)
7. Gonzalez can produce any of three products with its current production line. The heat treating
equipment has 400 hours available during any given month. Per unit production, sales, and cost
statistics are as follows:
47

A
---

B
C
----Selling price
$15
$20
$10
Variable cost
$9
$12
$7
Required time in heat treat 1.5 hrs 2.5 hrs. 1.0 hrs
Maximum demand per month
100
100
100
a. How many of each product should Gonzalez produce and sell?
b. Suppose the selling price of C increases to $12. How many of each product should Gonzalez
produce and sell?
SOLUTION:
a. 100 A, 100 B, 0 C
A: ($15 - 9)/1.5 = $4.00/hr 100 x 1.5 hrs = 150.0 hrs
B: ($20 - 12)/2.5 = $3.20/hr 100 x 2.5 hrs = 250.0
C: ($10 - 7)/1.0 = $3.00/hr
0 (no hours remaining)
b. 100 A, 60 B, 100 C
C: ($12 - 7)/1.0 = $5.00/hr
100 x 1.0 hrs = 100.0 hrs
A: ($15 - 9)/1.5 = $4.00/hr
100 x 1.5 hrs = 150.0
B: ($20 - 12)/2.5 = $3.20/hr
(400 - 100 - 150)/2.5 hrs = 60 units
8. Scottso Enterprises has the following products and costs:
A
Unit demand per month

B
2,000

3,000

4,000

Selling price
$500
$600
$800
Materials
150
300
350
Labor and overhead
180
210
300
Sales commission
50
75
100
Labor and overhead are applied to each product at a rate of $30 per machine hour. Management
considers both labor and overhead to be fixed costs.
Scottso currently has 60,000 hours available for production each month. How many units should be
produced and sold for each product?
An exporter has approached Scottso with an offer to purchase 500 units of product C for a discounted
price. This is a one-time order and will not affect normal sales. No commission will be paid. What is
the minimum price Scottso should accept?
SOLUTION:
a. A: 2,000 B: 1,142 C: 4,000
Throughput/hour for A: $50 [(500 - 150 - 50)/(180/30)]
Throughput/hour for B: $32.14 [(600 - 300 - 75)/(210/30)]
Throughput/hour for C: $35 [(800 - 350 - 100)/(300/30)]
Hours available
60,000
Produce A: 2,000 x 6
12,000
Time remaining
48,000
Produce C: 4,000 x 10
40,000
Time remaining
8,000
Produce B: 1,142 x 7
7,994
Time remaining
6
b. $575
Lost throughput of B: $32.14 x 7 hours = $225
Materials
350
Minimum price
$575
9. Miami Company currently sells 3,000 units of product A for $1.25 each. Variable costs are $0.60,
avoidable fixed costs are $750, and unavoidable allocated fixed costs are $1,500. An exporter has
offered $0.90 per unit for 800 units of product A.
48

a. Find the change in income if Miami can accept the order without affecting current sales.
b. The managers believe that if they accept the special order, they will lose some sales at the
regular price. Determine the number of units they could lose before the order became unprofitable.
c. The managers believe that they will lose 270 units at the regular price if they accept the order.
Calculate the price they must charge for the special order to increase income by $200.
SOLUTION:
a. Change in income: $240 increase [800 x ($0.90 - $0.60)]
b. Sales to lose: 369 units [$240/($1.25 - $0.60)] = 369
c. Price: $1.07
Lost contribution margin (270 x $0.65)
$175.5
Desired profit
200.0
----Contribution margin required from special order $375.5
Divided by 800 units
800
----Equals contribution margin per unit
$0.47 rounded
Plus variable cost
0.60
----Equals required price
$1.07
=====
10. Arpeggio Company manufactures 1,000 units of part XYZ annually. The following information has
been collected:
Materials
$200,000
Direct labor
110,000
Variable overhead
50,000
Fixed overhead
100,000
-------Total costs
$460,000
========
Mobile Company has offered to provide part XYZ for $400 per unit. If Arpeggio accepts the offer
another product will be moved into the space vacated, saving $60,000 a year in rent.
a. What would be the dollar impact if Arpeggio accepted the offer?
b. What is the maximum price Arpeggio is willing to pay for the part?
SOLUTION:
a. $20,000 more profits ($420,000 make - $400,000 buy)
Cost to buy the part: 1,000 x $400 = $400,000
Cost to make the part: $200,000 + 110,000 + 50,000 = $360,000 + 60,000 rent = $420,000
b. $420 ($420,000/1,000)
CHAPTER 6: OPERATIONAL AND FINANCIAL BUDGETING
Multiple Choice
a 1. The starting point in preparing a comprehensive budget is
a. the sales forecast.
b. the cash budget.
c. the budgeted income statement.
d. the flexible expense budget.
d 2. Budgets are related to which of the following management functions?
a. Planning.
b. Control.
49

c. Performance evaluation.
d. All of the above.
d 3. Which of the following should be used to forecast sales?
a. Regression analysis.
b. The scatter diagram.
c. The judgment of the most experienced managers.
d. Whatever method produces the most accurate forecast.
a 4. A critical factor for using indicator methods to forecast sales is
a. the availability of a forecasted value for the indicator.
b. an upward trend in the value of the indicator.
c. governmental collection of data for computing and reporting the value of the indicator.
d. the availability of an indicator that covers the entire country.
d 5. Which of the following equations can be used to budget purchases? (BI = beginning inventory,
EI = ending inventory desired, CGS = budgeted cost of goods sold)
a. Budgeted purchases = CGS + BI - EI
b. Budgeted purchases = CGS + BI
c. Budgeted purchases = CGS + EI + BI
d. Budgeted purchases = CGS + EI - BI
b 6. A flexible budget is
a. one that can be changed whenever a manager so desires.
b. adjusted to reflect expected costs at the actual level of activity.
c. one that uses the formula total cost = cost per unit x units produced.
d. the same as a continuous budget.
b 7. The use of flexible (as opposed to static) budget allowances is LEAST important for which of the
following?
a. Costs of the production department.
b. Costs of the general accounting department.
c. Costs of the product shipping department.
d. Costs of the material receiving department.
d 8. Budgets set at very high levels of performance (i.e., very low costs)
a. assist in planning the operations of the company.
b. stimulate people to perform better than they ordinarily would.
c. are helpful in evaluating the performance of managers.
d. can lead to low levels of performance.
c 9. Inventory policy is most critical in the budgeting of
a. sales.
b. cost of goods sold.
c. purchases.
d. expenses.
a 10. Budgeting expenditures by purpose is called
a. program budgeting.
b. zero-based budgeting.
c. line budgeting.
d. flexible budgeting.
c 11. Which of the following is a difference between a static budget and a flexible budget?
a. A flexible budget includes only variable costs, a static budget includes only fixed costs.
b. A flexible budget includes all costs, a static budget includes only fixed costs.
c. A flexible budget gives different allowances for different levels of activity; a static budget does
not.
d. None of the above.
a 12. A static budget is most appropriate for a department
a. with only fixed costs.
b. with only variable costs.
c. with mostly mixed costs.
d. with any of the above characteristics.
d 13. Which of the following is NOT an advantage of budgeting?
a. It requires managers to state their objectives.
b. It facilitates control by permitting comparisons of budgeted and actual results.
50

c. It facilitates performance evaluation by permitting comparisons of budgeted and actual results.


d. It provides a check-up device that allows managers to keep close tabs on their subordinates.
b 14. An imposed budget
a. is the same as a static budget.
b. can lead to poor performance.
c. is best for planning purposes.
d. eliminates the need for a sales forecast.
b 15. Prohibiting managers from overspending budget allowances
a. improves company performance.
b. can harm company performance.
c. eliminates the need for comparisons of budgeted and actual amounts.
d. usually reduces the need to prepare a cash budget.
b 16. Which of the following will occur if X Co.'s actual sales in May are lower than its budgeted sales
for that month?
a. X won't have enough cash to cover bills requiring payment in May.
b. X's actual inventory at the end of May will be higher than budgeted.
c. X's actual purchases in June will be higher than budgeted.
d. All of the above.
c 17. JIT manufacturers are more likely than conventional manufacturers to
a. use static budget allowances for manufacturing costs.
b. prepare production budgets without a sales forecast.
c. budget unit production equal to budgeted unit sales.
d. experience budget variances.
a 18. If cash receipts from customers are greater than sales, which of the following is most likely to be
true?
a. The balance of accounts receivable will decrease.
b. The company's outstanding debt will decrease.
c. The company's cash balance will increase.
d. The company will show a profit.
c 19. A cash budget is NOT prepared until a company has
a. obtained a commitment from its bank that cash will be available as needed.
b. prepared the pro forma balance sheet.
c. prepared its purchases budget.
d. determined that enough cash is available to meet dividend payments.
a 20. Which of the following is LEAST likely to be affected if unit sales for this month are lower than
budgeted?
a. Production for this month.
b. Production for next month.
c. Cash receipts for next month.
d. Inventory at the end of this month.
b 21. "Incremental budgeting" refers to
a. line-by-line approval of expenditures.
b. setting budget allowances based on prior year expenditures.
c. requiring top management approval of increases in budgets.
d. using incremental revenues and costs in budgeting.
b 22. The principal DISADVANTAGE of line budgeting is
a. it can only be used by not-for-profit entities.
b. it limits the flexibility of managers to accomplish the entity's objectives.
c. it works only in conjunction with zero-based budgeting.
d. none of the above.
a 23. The cash receipts budget
a. requires a sales forecast.
b. requires a purchases or production budget.
c. is prepared after the cash disbursements budget.
d. has none of the above characteristics.
c 24. The type of company most likely to run short of cash during the year is one with
a. little seasonality.
b. high contribution margin percentage.
51

c. high seasonality and rapid sales growth.


d. relatively low fixed costs.
d 25. If a company is earning a profit,
a. its cash balance is increasing.
b. its monthly cash disbursements will be stable.
c. its inventory is increasing.
d. it might have to borrow money.
a 26. One difference between budgeting in for-profit and not-for-profit entities is that not-for-profit
entities usually
a. budget expenses before revenues.
b. don't need a cash budget.
c. are less likely to use incremental budgeting.
d. use computer software-packages to facilitate the budgeting process.
d 27. To prepare its cash disbursements budget, a company uses information from
a. its balance sheet at the end of the prior period.
b. its purchases budget.
c. its capital budget.
d. all of the above sources.
b 28. Just-in-time manufacturers are more likely than conventional manufacturers to
a. prepare production budgets without a sales forecast.
b. budget materials purchases equal to the current month's needs for production.
c. budget unit production for the month at greater than budgeted unit sales for the month.
d. experience cash shortages.
c 29. Quorum Company desires an ending inventory of $120,000. It expects sales of $240,000 and
has a beginning inventory of $80,000. Cost of sales is 60% of sales. Budgeted purchases are
a. $120,000.
b. $144,000.
c. $184,000.
d. $264,000.
d 30. Garamond Company budgeted purchases of $200,000. Cost of sales was $240,000 and the
desired ending inventory was $84,000. The beginning inventory was
a. $40,000.
b. $64,000.
c. $84,000.
d. $124,000.
a 31. Wildwood Company budgeted purchases of 20,000 units. The budgeted beginning inventory was
4,800 units and the budgeted ending inventory was 6,000 units. Budgeted sales were
a. 18,800 units.
b. 21,200 units.
c. 24,800 units.
d. 26,000 units.
c 32. Menomonie Company budgeted sales of 18,000 units. The budgeted beginning inventory was
3,000 units and the budgeted ending inventory was 5,000 units. Budgeted production is
a. 23,000 units.
b. 21,000 units.
c. 20,000 units.
d. 16,000 units.
d 33. Baker Company budgets supplies as $20,000 + ($1.20 x direct labor hours). Baker has
budgeted 18,000 direct labor hours, $130,000 direct labor cost. The flexible budget allowance for
supplies is
a. $18,000.
b. $20,000.
c. $150,000.
d. some other number.
b 34. Equinox Company budgeted sales of 44,000 units for January, 60,000 for February. The
budgeted beginning inventory for January 1 was 14,000 units. Equinox desires an ending inventory
equal to one-half of the following month's sales needs. Budgeted production for January is
a. 74,000 units.
b. 60,000 units.
52

c. 52,000 units.
d. 28,000 units.
c 35. Sams Company manufactures a single product. It keeps its inventory of finished goods at 75%
the coming month's budgeted sales, inventory of raw materials at 50% of the coming month's
budgeted production needs. Each unit of product requires two pounds of materials. The production
budget is, in units: May, 1,000; June, 1,200; July, 1,300; August, 1,600. Raw material purchases in
June would be
a. 1,525 pounds.
b. 2,550 pounds.
c. 2,800 pounds.
d. 3,050 pounds.
a 36. Hayward Company desires an ending inventory of $70,000. It expects sales of $400,000 and has
a beginning inventory of $65,000. Cost of sales is 65% of sales. Budgeted purchases are
a. $265,000.
b. $395,000.
c. $405,000.
d. $535,000.
c 37. Bryce Company budgeted sales of 50,000 units for January, 60,000 for February. Bryce Company
desires an ending inventory equal to one-half of the following month's sales needs. Inventory on
January 1 was as desired. Budgeted production for January is
a. 22,000 units.
b. 52,000 units.
c. 55,000 units.
d. 74,000 units.
c 38. Chetek Company budgeted purchases of 19,000 units. The budgeted beginning inventory was
12,400 units and the budgeted ending inventory was 13,000 units. Budgeted sales were
a. 32,000 units.
b. 31,400 units.
c. 18,400 units.
d. 19,600 units.
d 39. Barron Company manufactures a single product. Barron keeps inventory of raw materials at
50% of the coming month's budgeted production needs. Each unit of product requires three pounds
of materials. The production budget is, in units: May, 1,000; June, 1,200; July, 1,300; August, 1,600.
Raw material purchases in July would be
a. 1,450 pounds.
b. 2,400 pounds.
c. 3,900 pounds.
d. some other number.
c 40. Acker Company has prepared the following flexible budget for production costs: total production
costs = $260,000 + $5X, where X is the number of machine hours. Acker produced 20,000 units,
using 34,000 machine hours at a total cost of $425,000. The flexible budget allowance for production
costs is
a. $260,000.
b. $425,000.
c. $430,000.
d. $525,000.
c 41. Scooter Inc. has projected sales to be $130,000 in June, $135,000 in July and $150,000 in
August. Scooter collects 30% of a month's sales in the month of sale, 50% in the month following the
sale, and 16% in the second month following the sale. Cash collections in August would be
a. $ 45,000.
b. $127,300.
c. $133,300.
d. $138,500.
d 42. Rundall Co. makes payments for purchases 30% during the month of purchase and the
remainder the following month. April purchases are projected to be $160,000; May purchases will be
$240,000. Cash payments in May will be
a. $ 72,000.
b. $108,000.
c. $168,000.
d. $184,000.
c 43. Randall Co. makes payments for purchases 30% during the month of purchase and the
53

remainder the following month. April purchases are projected to be $80,000; May purchases will be
$120,000. The accounts payable balance on May 31 will be
a. $36,000.
b. $54,000.
c. $84,000.
d. $92,000.
c 44. Alfuth Co. makes payments for purchases 10% during the month of purchase, 60% in the
following month, and the remainder in the second month following the purchase. Purchases are
projected to be $260,000 in January, $280,000 in February, and $320,000 in March. March payments
will be
a. $ 32,000.
b. $168,000.
c. $278,000.
d. some other number.
d 45. Reid Co. makes payments for purchases 10% during the month of purchase, 60% in the
following month, and the remainder in the second month following the purchase. Purchases are
projected to be $130,000 in January, $140,000 in February, and $160,000 in March. The March 31
accounts payable balance will be
$48,000.
b. $96,000.
c. $144,000.
d. $186,000.
c 46. Andover Inc. has projected sales to be: February, $10,000; March, $9,000; April, $8,000; May,
$10,000; and June, $11,000. Andover has 30% cash sales and 70% sales on account. Accounts are
collected 40% in the month following the sale and 55% collected the second month. Total cash
receipts in May would be
a. $3,000.
b. $8,150.
c. $8,705.
d. some other number.
d 47. Conde Inc. has projected sales to be: February, $20,000; March, $18,000; April, $16,000; May,
$20,000; and June, $22,000. Conde has 30% cash sales and 70% sales on account. Accounts are
collected 40% in the month following the sale and 60% collected the second month. Accounts
receivable for May 31 would be
a. $ 6,160.
b. $13,300.
c. $14,000.
d. $20,720.
d 48. Holmgren estimates its supplies purchases to be $21,000 in August and $28,000 in September.
Holmgren pays 70% of its accounts in the month of purchase with the remainder paid the following
month. September payments would be
a. $14,700.
b. $19,600.
c. $23,100.
d. $55,900.
c 49. Danner Inc. has projected sales to be $100,000 in June, $90,000 in July, and $70,000 in August.
Danner collects 50% of a month's sales in the month of sale, 30% in the month following the sale, and
16% in the second month following the sale. Cash collections in August would be
a. $35,000.
b. $62,000.
c. $78,000.
d. $86,000.
a 50. Clearwater Inc. has projected sales to be $160,000 in April, $200,000 in May, and $240,000 in
June. Clearwater collects 40% of a month's sales in the month of sale, 40% in the month following the
sale, and 20% in the second month following the sale. The accounts receivable balance on June 30
would be
a. $184,000.
b. $144,000.
c. $ 40,000.
d. some other number.
True-False
54

F 1. A just-in-time manufacturer does NOT need a sales budget.


T 2. A flexible budget allowance is NOT especially useful for budgeting discretionary costs.
F 3. The purchases budget is prepared before the sales budget because the company cannot
estimate what it will sell until it has some idea of what will be on hand.
F 4. The longer the time period covered by a budget, the more useful the budget will be for
controlling operations.
F 5. A purchases budget is normally prepared after the company has forecast how much cash it will
have available to pay for purchases.
F 6. Imposed budgets are exceptionally ambitious goals not likely to be achieved without making
fundamental changes in the way a job is done.
F 7. A JIT manufacturer that maintains no inventory doesn't need a cash disbursements budget.
F 8. The budget for a retailer is likely to be more complex than that for a manufacturer because a
retailer has a wider variety of customers.
F 9. The increasing public demand for accountability from governmental and other not-for-profit
organizations has resulted in an increased use of incremental budgeting.
T 10. Line-by-line budget authorization is common in governmental units.

Problems
1. Ballan Inc. estimates its units sales for the coming months to be as follows:
March
280,000
April
260,000
May
250,000
June
230,000
July
240,000
August
225,000
Ballan maintains inventory at budgeted sales needs for the next month. March 1 inventory will be
248,000 units.
a. Prepare a monthly purchasing schedule for March through July.
SOLUTION:
a. March purchases:

292,000 units [280,000 + 260,000 248,000]

April purchases: 250,000 units [260,000 + 250,000 260,000]


May purchases:

230,000 units [250,000 + 230,000 250,000]

June purchases:240,000 units [230,000 + 240,000 230,000]


July purchases: 225,000 units [240,000 + 225,000 240,000]
2. Superior Company manufactures a single product. It keeps its inventory of finished goods at twice
the coming month's budgeted sales and inventory of raw materials at 150% of the coming month's
budgeted production. Each unit of product requires five pounds of materials, which cost $3 per
pound. The sales budget is, in units: May, 10,000; June, 12,400; July, 12,600; August, 13,200.
a. Compute budgeted production for June.
b. Compute budgeted production for July.
c. Compute budgeted material purchases for June in pounds and dollars.
55

SOLUTION:
a. June production: 12,800 units [12,400 + (2 x 12,600) - (2 x 12,400)]
b. July production: 13,800 units [12,600 + (2 x 13,200) - (2 x 12,600)]
c. June materials purchases: 71,500 pounds; $214,500
Used in production (5 lbs. x 12,800)
64,000 lbs.
Ending inventory (5 lbs. x 13,800 x 150%)
103,500
------Total
167,500
Less beginning inventory (5 lbs. x 12,800 x 150%) 96,000
------Purchases
71,500
Times cost per pound
$3
------Equals dollar purchases
$214,500
========
3. Ironwood sells a single product for $10. The purchase cost is $4 per unit and Ironwood pays a 20%
sales commission. Fixed costs are $45,000 per month including $12,000 depreciation, and the
company maintains inventory equal to budgeted sales needs for the following month. The following
budgeted data are available.
Inventory on hand, February 1
28,000 units
Budgeted sales - February
24,000 units
- March
26,000 units
- April
25,000 units
a. Compute total budgeted income for February and March.
b. Find budgeted inventory at March 31 in units and dollars.
c. Find budgeted purchases for March in units and dollars
SOLUTION:
a. Budgeted income: $110,000
Sales [(24,000 + 26,000) x $10]
$500,000
Cost of sales (50,000 x $4)
200,000
------Gross profit
$300,000
Commissions at 20%
100,000
------Contribution margin
$200,000
Fixed costs (2 x 45,000)
90,000
------Income
$110,000
========
b. Budgeted inventory: 25,000 units; $100,000 ($4 x 25,000)
c. Budgeted purchases: 25,000 units; $100,000
Cost of sales
Ending inventory

26,000 units
$104,000
25,000
100,000
------------Total required
51,000
$204,000
Less beginning inventory
26,000
104,000
------------Purchases
25,000 units x $4 $100,000
======
========
4. Westrum estimates production overhead costs equal to $300,000 + $2X, where X is the number of
machine hours used. Westrum budgeted 40,000 machine hours for 20X4. Westrum produced 23,000
56

units in 20X4, each requiring 3 machine hours. Actual production costs were $420,000.
a. Calculate the flexible budget allowance for production overhead costs for 20X4.
b. Find the amount and direction of the budget variance for 20X4 for production overhead.
(favorable unfavorable) Circle one answer.
SOLUTION:
a. Flexible budget allowance, $438,000 [$300,000 + (23,000 x 3 x $2)]
b. Budget variance: $18,000 favorable ($438,000 - $420,000)
5. Acme Inc. estimates its dollar sales for the coming months to be as follows.
June
$340,000
July
360,000
August
300,000
September 260,000
October 240,000
November 200,000
Acme has an average gross margin of 40% of sales and maintains inventory at 75% of budgeted
sales needs for the next month. Acme began June with $150,000 in inventory.
a. Prepare a monthly purchasing schedule (in $) for as many months as is possible.
SOLUTION:
a.

June
July
August September October
$340,000 $360,000 $300,000 $260,000 $240,000
x .40
x .40
x .40
x .40
x .40
-------- -------- -------- -------- -------Cost of Sales
$136,000 $144,000 $120,000 $104,000 $ 96,000
+ Ending Inv
108,000
90,000
78,000
72,000
60,000
- Beg Inv
(150,000) (108,000) (90,000)
(78,000)
(72,000)
-------- -------- -------- -------- -------Purchases
$ 94,000
$126,000 $108,000 $ 98,000
$ 84,000
======== ======== ======== ========
Sales
x 40%

========

6. Bay City estimates production overhead costs equal to $200,000 + $4X + $7Y, where X is the
number of direct labor hours used and Y is the number of machine hours used. Bay City budgeted
20,000 direct labor hours and 50,000 machine hours for 20X9. Bay City produced 30,000 units in
20X9, each requiring 1 direct labor hour and 2.5 machine hours. Actual production costs were
$890,000.
a. Calculate the flexible budget allowance for production overhead costs for 20X9.
b. Find the amount and direction of the budget variance for 20X2 for production overhead.
(favorable unfavorable) Circle one answer.
SOLUTION:
a. Flexible budget allowance, $845,000 [$200,000 + (30,000 x 1 x $4) + (30,000 x 2.5 x 7)]
b. Budget variance: $45,000 unfavorable ($845,000 - $890,000)
7.

Webster Company has the following sales budget.


January
February
March
April

$200,000
$240,000
$300,000
$360,000

Cost of sales is 70% of sales. Sales are collected 40% in the month of sale and 60% in the
following month. Webster keeps inventory equal to double the coming month's budgeted sales
57

requirements. It pays for purchases 80% in the month of purchase and 20% in the month after
purchase. Inventory at the beginning of January is $190,000. Webster has monthly fixed costs of
$30,000 including $6,000 depreciation. Fixed costs requiring cash are paid as incurred.
a. Compute budgeted cash receipts in March.
b. Compute budgeted accounts receivable at the end of March.
c. Compute budgeted inventory at the end of February.
d. Compute budgeted purchases in February.
e. March purchases are $290,000. Compute budgeted cash payments in March to suppliers of
goods.
f. Compute budgeted accounts payable for goods at the end of February.
g. Cash at the end of February is $45,000. Cash disbursements are not required for anything other
than payments to suppliers and fixed costs. Compute the budgeted cash balance at the end of March.
SOLUTION:
a. March receipts: $264,000 [($240,000 x 60%) + ($300,000 x 40%)]
b. Receivables at end of March: $180,000 [$300,000 x (100% - 40%)]
c. Inventory at end of February: $420,000 ($300,000 x 70% x 2)
d. February purchases: $252,000
($240,000 x 2 x 70%)]

[($240,000 x 70%) + ($300,000 x 2 x 70%)

e. March payments: $282,400 [(252,000 x 20%) + ($290,000 x 80%)]


f. AP at end of February: $50,400 ($252,000 x 20%)
g. Cash at end of March: $2,600 ($25,000 + $264,000 - $282,400 - $24,000)
8.

Weasel Company has the following sales projections for 20X9:


January
February
March
April
May
June

$200,000
210,000
225,000
230,000
245,000
240,000

Weasel collects 40% of its sales in the month of sale, 45% in the month following the sale and
13% in the second month following the sale. Records show that sales were $225,000 in November
and $208,000 in December 20X2.
a. Prepare a schedule of cash receipts for the first three months of 20X3.
b.What would be the accounts receivable (net of bad debts) balance on March 31, 20X3?
SOLUTION:
a.

January collections: (13% x 225,000) = $29,250


(45% x 208,000) = 93,600
(40% x 200,000) = 80,000
------$202,850
========
February collections: (13% x 208,000) = $27,040
(45% x 200,000) = 90,000
(40% x 210,000) = 84,000
------58

$201,040
========
March collections: (13% x 200,000) = $26,000
(45% x 210,000) = 94,500
(40% x 225,000) = 90,000
------$210,500
========
b.

$157,800

$ 27,300 = February sales 210,000 x 13%


$130,500 = March sales 225,000 x (45% + 13%)
-------$157,800
========

9. Bismarck has the following sales budget:


March
April
May
June

$300,000
$312,000
$320,000
$348,000

Cost of sales is 55% of sales. Bismarck keeps an inventory equal to one-fourth the coming
month's budgeted sales requirements. It pays for purchases 40% in the month of purchase and 60%
in the month after purchase. Accounts Payable is $94,800 on March 1.
a. Prepare a monthly purchasing schedule for March through May.
b. Prepare a monthly cash payment schedule for March through May.
c. Compute the accounts payable balance as of May 31.
SOLUTION:
a.

March April
May
Sales
$300,000 $312,000 $320,000
x 55%
x .55
x .55
x .55
-------- -------- -------Cost of Sales
$165,000 $171,600 $176,000
+ Ending Inv
42,900
44,000
51,975
- Beg Inv
(41,250)
(42,900)
(44,000)
-------- -------- -------Purchases $166,650 $172,700 $183,975
======== ======== ========

b.

March payments: (40% x 166,650) = $ 66,660


Mar 1 Acct Pay = 94,800
------$161,460
========
April payments:(40% x 172,700) = $ 69,080
(60% x 166,650) = 99,990
------$169,070
========
May payments:

c.

(40% x 183,975) = $ 73,590


(60% x 172,700) = 103,620
------$177,210
========

Accounts Payable, May 31: $110,385

[60% x $183,975]

59

10. Hicks Company has the following sales projections for 20X9:
January $160,000 March 175,000
May
195,000
February
168,000
April
180,000
June
190,000
Hicks collects 30% of its sales in the month of sale, 45% in the month following the sale, and 24%
in the second month following the sale. Records show that sales were $160,000 in November and
$168,000 in December 20X8.
a. Prepare a schedule of cash receipts for the first three months of 20X9.
b.What would be the accounts receivable balance (net of bad debts) on March 31, 20X9?
SOLUTION:
a.

January collections: (24% x 160,000) = $38,400


(45% x 168,000) = 75,600
(30% x 160,000) = 48,000
------$162,000
========
February collections: (24% x 168,000) = $40,320
(45% x 160,000) = 72,000
(30% x 168,000) = 50,400
------$162,720
========
March collections: (24% x 160,000) = $38,400
(45% x 168,000) = 75,600
(30% x 175,000) = 52,500
------$166,500
========

b.

$161,070

$40,320 = February sales (168,000 x 24%)


120,750 = March sales [175,000 x (45% + 24%)]
------$161,070
========

CHAPTER 7: CAPITAL BUDGETING DECISIONSPART I


Multiple Choice
d 1. Calculating the payback period for a capital project requires knowing which of the following?
a. Useful life of the project.
b. The company's minimum required rate of return.
c. The project's NPV.
d. The project's annual cash flow.
c 2. The payback criterion for capital investment decisions
a. is conceptually superior to the IRR criterion.
b. takes into consideration the time value of money.
c. gives priority to rapid recovery of cash.
d. emphasizes the most profitable projects.
a 3. Which of the following is NOT relevant in calculating annual net cash flows for an investment?
a. Interest payments on funds borrowed to finance the project.
b. Depreciation on fixed assets purchased for the project.
c. The income tax rate.
d. Lost contribution margin if sales of the product invested in will reduce sales of other products.
a 4. If the present value of the future cash flows for an investment equals the required investment,
the IRR is
a. equal to the cutoff rate.
60

b. equal to the cost of borrowed capital.


c. equal to zero.
d. lower than the company's cutoff rate of return.
b 5. The relationship between payback period and IRR is that
a. a payback period of less than one-half the life of a project will yield an IRR lower than the target
rate.
b. the payback period is the present value factor for the IRR.
c. a project whose payback period does not meet the company's cutoff rate for payback will not
meet the company's criterion for IRR.
d. none of the above.
c 6. Which of the following events is most likely to reduce the expected NPV of an investment?
a. The major competitor for the product to be manufactured with the machinery being considered
for purchase has been rated "unsatisfactory" by a consumer group.
b. The interest rate on long-term debt declines.
c. The income tax rate is raised by the Congress.
d. Congress approves the use of faster depreciation than was previously available.
a 7. If an investment has a positive NPV,
a. its IRR is greater than the company's cost of capital.
b. cost of capital exceeds the cutoff rate of return.
c. its IRR is less than the company's cutoff rate of return.
d. the cutoff rate of return exceeds cost of capital.
c 8. Which of the following describes the annual returns that are discounted in determining the NPV
of an investment?
a. Net incomes expected to be earned by the project.
b. Pre-tax cash flows expected from the project.
c. After-tax cash flows expected from the project.
d. After-tax cash flows adjusted for the time value of money.
b 9. Which of the following capital budgeting methods does NOT consider the time value of money?
a. IRR.
b. Book rate of return.
c. Time-adjusted rate of return.
d. NPV.
b 10. All other things being equal, as cost of capital increases
a. more capital projects will probably be acceptable.
b. fewer capital projects will probably be acceptable.
c. the number of capital projects that are acceptable will change, but the direction of the change
is not determinable just by knowing the direction of the change in cost of capital.
d. the company will probably want to borrow money rather than issue stock.
d 11. Which of the following is a basic difference between the IRR and the book rate of return (BRR)
criteria for evaluating investments?
a. IRR emphasizes expenses and BRR emphasizes expenditures.
b. IRR emphasizes revenues and BRR emphasizes receipts.
c. IRR is used for internal investments and BRR is used for external investments.
d. IRR concentrates on receipts and expenditures and BRR concentrates on revenues and
expenses.
a 12. If a project has a payback period shorter than its life,
a. its NPV may be negative.
b. its IRR is greater than cost of capital.
c. it will have a positive NPV.
d. its incremental cash flows may not cover its cost.
c 13. Cost of capital is
a. the amount the company must pay for its plant assets.
b. the dividends a company must pay on its equity securities.
c. the cost the company must incur to obtain its capital resources.
d. the cost the company is charged by investment bankers who handle the issuance of equity or
long-term debt securities.
d 14. The normal methods of analyzing investments
a. cannot be used by not-for-profit entities.
b. do not apply if the project will not produce revenues.
61

c. cannot be used if the company plans to finance the project with funds already available
internally.
d. require forecasts of cash flows expected from the project.
a 15. Which of the following is NOT a defect of the payback method?
a. It ignores cash flows because it uses net income.
b. It ignores profitability.
c. It ignores the present values of cash flows.
d. It ignores the pattern of cash flows beyond the payback period.
b 16. A company with cost of capital of 15% plans to finance an investment with debt that bears 10%
interest. The rate it should use to discount the cash flows is
a. 10%.
b. 15%.
c. 25%.
d. some other rate.
c 17. Which of the following events will increase the NPV of an investment involving a new product?
a. An increase in the income tax rate.
b. An increase in the expected per-unit variable cost of the product.
c. An increase in the expected annual unit volume of the product.
d. A decrease in the expected salvage value of equipment.
b 18. An investment has a positive NPV discounting the cash flows at a 14% cost of capital. Which
statement is true?
a. The IRR is lower than 14%.
b. The IRR is higher than 14%.
c. The payback period is less than 14 years.
d. The book rate of return is 14%.
a 19. The technique most concerned with liquidity is
a. payback.
b. NPV.
c. IRR.
d. book rate of return.
d 20. The technique that does NOT use cash flows is
a. payback.
b. NPV.
c. IRR.
d. book rate of return.
a 21. If there were no income taxes,
a. depreciation would be ignored in capital budgeting.
b. the NPV method would not work.
c. income would be discounted instead of cash flow.
d. all potential investments would be desirable.
a 22. Two new products, X and Y, are alike in every way except that the sales of X will start low and
rise throughout its life, while those of Y will be the same each year. Total volumes over their five-year
lives will be the same, as will selling prices, unit variable costs, cash fixed costs, and investment. The
NPV of product X
a. will be less than that of product Y.
b. will be the same as that of product Y.
c. will be greater than that of product Y.
d. none of the above.
d 23. Which of the following events is most likely to increase the number of investments that meet a
company's acceptance criteria?
a. Top management raises the target rate of return.
b. The interest rate on long-term debt rises.
c. The income tax rate rises.
d. The IRS allows companies to expense purchases of fixed assets, instead of depreciating them
over their lives.
d 24. Investment A has a payback period of 5.4 years, investment B one of 6.7 years. From this
information we can conclude
a. that investment A has a higher NPV than B.
b. that investment A has a higher IRR than B.
62

c. that investment A's book rate of return is higher than B's.


d. none of the above.
d 25. Investment A has a book rate of return of 26%, investment B one of 18%. From this information
we can conclude
a. that investment A has a higher NPV than B.
b. that investment A has a higher IRR than B.
c. that investment A has a shorter payback period than B.
d. none of the above.
c 26. A dollar now is worth more than a dollar to be received in the future because of
a. inflation.
b. uncertainty.
c. the opportunity cost of waiting.
d. none of the above.
a 27. In contrast to the payback and book rate of return methods, the NPV and IRR methods
a. consider the time value of money.
b. ignore depreciation.
c. use after-tax cash flows.
d. all of the above.
a 28. Which of the following is a discounted cash flow method?
a. NPV.
b. Payback.
c. Book rate of return.
d. All of the above.
a 29. Which statement describes the relevance of depreciation in calculating cash flows?
a. Depreciation is relevant only when income taxes exist.
b. Depreciation is always relevant.
c. Depreciation is never relevant.
d. Depreciation is relevant only with discounted cash flow methods.
b 30. As the discount rate increases
a. present value factors increase.
b. present value factors decrease.
c. present value factors remain constant.
d. it is impossible to tell what happens to the factors.
a 31. As the length of an annuity increases
a. present value factors increase.
b. present value factors decrease.
c. present value factors remain constant.
d. it is impossible to tell what happens to present value factors.
a 32. The only future costs that are relevant to deciding whether to accept an investment are those
that will
a. be different if the project is accepted rather than rejected.
b. be saved if the project is accepted rather than rejected.
c. be deductible for tax purposes.
d. affect net income in the period that they are incurred.
a 33. Which of the following is true of an investment?
a. The lower the cost of capital, the higher the NPV.
b. The lower the cost of capital, the higher the IRR.
c. The longer the project's life, the shorter its payback period.
d. The higher the project's NPV, the shorter its life.
c 34. Which of the following methods FAILS to distinguish between return of investment and return on
investment?
a. NPV.
b. IRR.
c. Payback.
d. Book rate of return.
c 35. If a company is NOT subject to income tax, which of the following is true of a proposed
investment?
a. The project's IRR equals the entity's cost of capital.
63

b. The project's NPV is zero.


c. Depreciation on assets required for the project is irrelevant to the evaluation.
d. The expected annual increase in future cash flows equals the investment required to undertake
the project.
d 36. Which of the following increases NPV and IRR?
a. An upward revision in expected annual net cash flows.
b. An upward revision of expected life.
c. An upward revision of the residual value of the long-lived assets being acquired for the project.
d. All of the above.
d 37. Qualitative issues could increase the acceptability of a project under which of the following
conditions?
a. The IRR is less than the company's cutoff rate.
b. The project has a negative NPV.
c. The payback period is longer than the company's cutoff period.
d. All of the above.
a 38. If Co. X wants to use IRR to evaluate long-term decisions and to establish a cutoff rate of return,
X must be sure the cutoff rate is
a. at least equal to its cost of capital.
b. at least equal to the rate used by similar companies.
c. greater than the IRR on projects accepted in the past.
d. greater than the current book rate of return.
a 39. Which of the following is NOT relevant in calculating net cash flows for Project N?
a. Interest payments on funds that would be borrowed to finance Project N.
b. Depreciation on assets purchased for Project N.
c. The contribution margin the company would lose if sales of the product introduced by Project N
will reduce sales of other products.
d. The income tax rate applicable to the entity.
b 40. If the IRR on an investment is zero,
a. its NPV is positive.
b. its annual cash flows equal its required investment.
c. it is generally a wise investment.
d. its cash flows decrease over its life.
d 41. If depreciation on a new asset exceeds its savings in cash operating costs, which of the
following is true?
a. The project is usually unacceptable.
b. The annual after-tax cash flow on the new asset will be greater than the savings in cash
operating costs.
c. The project has a negative NPV.
d. All of the above.
d 42. Cost of capital is
a. the interest rate an entity must pay to borrow money.
b. the return an entity's stockholders expect on their investment.
c. the rate of return the entity can earn from investing available cash.
d. a concept of managerial finance incorporating all of the above ideas.
b 43. An investment opportunity costing $75,000 is expected to yield net cash flows of $23,000
annually for five years. The NPV of the investment at a cutoff rate of 14% would be
a. $(3,959).
b. $3,959.
c. $75,000.
d. $78,959.
b 44. An investment opportunity costing $55,000 is expected to yield net cash flows of $22,000
annually for five years. The payback period of the investment is
a. 0.4 years.
b. 2.5 years.
c. $33,000.
d. some other number.
c 45. An investment opportunity costing $180,000 is expected to yield net cash flows of $53,000
annually for five years. The IRR of the investment is between
a. 10 and 12%.
64

b. 12 and 14%.
c. 14 and 16%.
d. 16 and 18%.
b 46. An investment opportunity costing $150,000 is expected to yield net cash flows of $45,000
annually for five years. The cost of capital is 10%. The book rate of return would be
a. 10%.
b. 20%.
c. 30%.
d. 33.3%.
a 47. An investment opportunity costing $150,000 is expected to yield net cash flows of $36,000
annually for six years. The NPV of the investment at a cutoff rate of 12% would be
a. $(2,004).
b. $2,004.
c. $150,000.
d. $147,996.
c 48. An investment opportunity costing $100,000 is expected to yield net cash flows of $22,000
annually for seven years. The payback period of the investment is
a. 0.22 years.
b. 3.08 years.
c. 4.55 years.
d. some other number.
a 49. An investment opportunity costing $200,000 is expected to yield net cash flows of $39,000
annually for eight years. The IRR of the investment is between
a. 10 and 12%.
b. 12 and 14%.
c. 14 and 16%.
d. 16 and 18%.
b 50. An investment opportunity costing $80,000 is expected to yield net cash flows of $25,000
annually for four years. The cost of capital is 10%. The book rate of return would be
a. 10.0%.
b. 12.5%.
c. 21.3%.
d. 32.0%.
True-False
T 1. Payback period is the length of time it will take a company to recoup its outlay for an
investment.
T 2. Discounted cash flow techniques apply to investments that involve either costs only, or both
costs and revenues.
F 3. Cost of capital is the interest rate that a company expects to pay to finance a particular capital
investment project.
F 4. The higher the cost of capital, the higher the present value of future cash inflows.
F 5. If the IRR on a capital project is positive, its NPV will be positive.
T 6. Salvage value is usually ignored in computing the tax depreciation on an investment in
depreciable assets.
F 7. IRR can be computed for even cash flows, but not for uneven cash flows.
T 8. If IRR is less than the cost of capital, the NPV will be negative.
F 9. IF NPV is negative, IRR is equal to the cost of capital.
T 10. Payback emphasizes the return of the investment and ignores the return on the investment.
Problems
65

1. An investment opportunity costing $180,000 is expected to yield net cash flows of $60,000
annually for five years.
a. Find the NPV of the investment at a cutoff rate of 12%.
b. Find the payback period of the investment.
c. Find the IRR on the investment.
SOLUTION:
a. NPV: $36,300 [(3.605 x $60,000) - $180,000]
b. Payback period: 3 years ($180,000/$60,000)
c. IRR: between 18 % and 20% (3.0 is between 3.127 and 2.991)
2. Tofte is considering the purchase of a machine. Data are as follows:
Cost
$100,000
Useful life
10 years
Annual straight-line depreciation
$ 10,000
Expected annual savings in cash
operation costs
$ 18,000
Tofte's cutoff rate is 12% and its tax rate is 40%.
a. Compute the annual net cash flows for the investment.
b. Compute the NPV of the project.
SOLUTION:
a. Annual net cash flows: $14,800 [$18,000 pretax - 40% x ($18,000 - $10,000 depreciation)]
b. NPV: Negative $16,380 [($14,800 x 5.650) - $100,000]
3. Willow Company is considering the purchase of a machine with the following characteristics.
Cost
$150,000
Estimated useful life
10 years
Expected annual cash cost savings $35,000
Marquette's tax rate is 40%, its cost of capital is 12%, and it will use straight-line depreciation for
the new machine.
a. Compute the annual after-tax cash flows for this project.
b. Find the payback period for this project.
SOLUTION:
a. Annual cash flows: $27,000 [$35,000 - 40% x ($35,000 - $15,000)]
b. Payback period: 5.56 years ($150,000/$27,000)
4. Bilt-Rite Co. has the opportunity to introduce a new product. Bilt-Rite expects the product to sell
for $60 and to have per-unit variable costs of $40 and annual cash fixed costs of $3,000,000.
Expected annual sales volume is 250,000 units. The equipment needed to bring out the new product
costs $5,000,000, has a four-year life and no salvage value, and would be depreciated on a straightline basis. Bilt-Rite's cost of capital is 10% and its income tax rate is 40%.
66

a. Find the increase in annual after-tax cash flows for this opportunity.
b. Find the payback period on this project.
c. Find the NPV for this project.
SOLUTION:
a. Increase in annual cash flows: $1,700,000
Income before taxes, 250,000 x ($60 - $40)
- $3,000,000 - $5,000,000/4
$ 750,000
Income tax
(300,000)
---------Net income
$ 450,000
Plus depreciation
1,250,000
---------Net cash flow
$1,700,000
==========
b. Payback period: 2.94 years ($5,000,000/$1,700,000)
c. NPV: $389,000 [($1,700,000 x 3.170) - $5,000,000]
5. An investment opportunity costing $600,000 is expected to yield net cash flows of $120,000
annually for ten years.
a. Find the NPV of the investment at a cutoff rate of 12%.
b. Find the payback period of the investment.
c. Find the IRR on the investment.
SOLUTION:
a. NPV: $78,000 [(5.650 x $120,000) - $600,000]
b. Payback period: 5 years ($600,000/$120,000)
c. IRR: 15% (5.0 is about halfway between 5.216 and 4.833)
6. Scottso has an investment opportunity costing $300,000 that is expected to yield the following
cash flows over the next six years:
Year
Year
Year
Year
Year
Year

One
$75,000
Two
$90,000
Three $115,000
Four $130,000
Five
$100,000
Six$90,000

a. Find the payback period of the investment.


b.Find the book rate of return of the investment.
c. Find the NPV of the investment at a cutoff rate of 10%.
SOLUTION:
a.

Payback period: 3.15 years (75,000 + 90,000 + 115,000 + .15 x 130,000)

b.

Book rate of return: 33.3%


Average return: $100,000 ($600,000 total / 6 years)
Depreciation:
50,000 ($30,000 / 6 years)
------Average income $50,000
67

Average investment: $300,000 / 2 = $150,000


Book rate of return = $50,000 / 150,000 = 33.3%
c.

NPV: $130,530
Cash
Factor PV
---------------1 75,000 .909
68,175
2 90,000 .826
74,340
3 115,000
.751
86,365
4 130,000
.683
88,790
5 100,000
.621
62,100
6 90,000 .564
50,760
------430,530
Investment
300,000
------NPV
130,530
======

7. Acme is considering the purchase of a machine. Data are as follows:


Cost
$160,000
Useful life
10 years
Annual straight-line depreciation
$ ???
Expected annual savings in cash
operation costs
$ 33,000
Acme's cutoff rate is 12% and its tax rate is 40%.
a. Compute the annual net cash flows for the investment.
b. Compute the NPV of the project.
c. Compute the IRR of the project.
SOLUTION:
a. Annual net cash flows: $26,200 [$33,000 pretax - 40% x ($33,000 - $16,000 depreciation)]
b. NPV: Negative $11,970 [($26,200 x 5.650) - $160,000]
c.

IRR: between 10% and 12% [factor of 6.107 (160,000/26,200) is between 6.145 and 5.650]

8. Scottso has an investment opportunity costing $180,000 that is expected to yield the following
cash flows over the next five years:
Year
Year
Year
Year
Year

One
Two
Three
Four
Five

$
$
$
$
$

30,000
60,000
90,000
60,000
30,000

a. Find the payback period of the investment.


b.Find the book rate of return of the investment.
c. Find the NPV of the investment at a cutoff rate of 12%.
SOLUTION:
a.

Payback period: 3.0 years (30,000 + 60,000 + 90,000)

b.

Book rate of return: 20%


68

Average return: $54,000 ($270,000 total / 5 years)


Depreciation:
36,000 ($180,000 / 5 years)
-----Average income $18,000
Average investment: $180,000 / 2 = $90,000
Book rate of return = $18,000 / $90,000 = 20%
c.

NPV: $6,930
Cash
-----30,000
60,000
90,000
60,000
30,000

Factor PV
----------1
.893
26,790
2
.797
47,820
3
.712
64,080
4
.636
38,160
5
.567
17,010
------193,860
Investment
180,000
------NPV
13,860
======
9. Reno Company is considering the purchase of a machine with the following characteristics.
Cost
$160,000
Estimated useful life
5 years
Expected annual cash cost savings $56,000
Expected salvage value
none
Reno's tax rate is 40%, its cost of capital is 12%, and it will use straight-line depreciation for the
new machine.
a. Compute the annual after-tax cash flows for this project.
b. Find the payback period for this project.
c. Compute the NPV for this project.
SOLUTION:
a. Annual cash flows: $46,400 [$56,000 - 40% x ($56,000 - 32,000)]
b. Payback period: 3.45 years ($160,000/$46,400)
c. NPV: $7,272 [($46,400 x 3.605) - $160,000]
10. Whitehall Co. has the opportunity to introduce a new product. Whitehall expects the project to
sell for $40 and to have per-unit variable costs of $27 and annual cash fixed costs of $1,500,000.
Expected annual sales volume is 200,000 units. The equipment needed to bring out the new product
costs $3,500,000, has a four-year life and no salvage value, and would be depreciated on a straightline basis. Whitehall's cutoff rate is 10% and its income tax rate is 40%.
a. Find the increase in annual after-tax cash flows for this opportunity.
b. Find the payback period on this project.
c. Find the NPV for this project.
SOLUTION:
a. Increase in annual cash flows: $1,100,000
Income before taxes, [200,000 x ($40 - $27)
69

- $1,500,000 - $3,500,000/4]
$ 225,000
Income tax
( 90,000)
---------Net income
$ 135,000
Plus depreciation
875,000
---------Net cash flow
$1,010,000
==========
b. Payback period: 3.47 years ($3,500,000/$1,010,000)
c. NPV: negative $298,300 [($1,010,000 x 3.170) - $3,500,000]
CHAPTER 8: CAPITAL BUDGETING DECISIONSPART II
Multiple Choice
c 1. Which of the following groups of capital budgeting techniques uses the time value of money?
a. Book rate of return, payback, and profitability index.
b. IRR, payback, and NPV.
c. IRR, NPV, and profitability index.
d. IRR, book rate of return, and profitability index.
b 2. Discounted cash flow techniques for analyzing capital budgeting decisions are NOT normally
applied to projects
a. requiring no investment after the first year of life.
b. having useful lives shorter than one year.
c. that are essential to the business.
d. involving replacement of existing assets.
d 3. The profitability index
a. does not use present values of cash flows.
b. is generally preferable to any other approach for evaluating mutually exclusive investment
alternatives.
c. produces the same ranking of investment alternatives as does the IRR criterion.
d. is a discounted cash flow method.
a 4. Companies using MACRS for tax purposes and straight-line depreciation for financial reporting
purposes usually find that the relationship between the tax basis and book value of their assets is
a. the tax basis is lower than book value.
b. the tax basis is higher than book value.
c. the tax basis is the same as book value.
d. none of the above.
c 5. A company that wants to use MACRS for tax purposes must
a. request permission from the IRS.
b. acquire new assets at or near the middle of the year.
c. ignore salvage value in calculating depreciation.
d. do none of the above.
c 6. The government could encourage increases in investment by
a. increasing tax rates.
b. lengthening the MACRS periods.
c. letting a company expense fixed assets in the year acquired instead of through annual
depreciation charges.
d. taking actions that would increase interest rates.
a 7. In choosing from among mutually exclusive investments the manager should normally select the
one with the highest
a. NPV.
b. IRR.
c. profitability index.
d. book rate of return.
a 8. In deciding whether to replace a machine, which of the following is NOT a sunk cost?
a. The expected resale price of the existing machine.
b. The book value of the existing machine.
c. The original cost of the existing machine.
70

d. The depreciated cost of the existing machine.


a 9. A company is considering replacing a machine with one that will save $50,000 per year in cash
operating costs and have $20,000 more depreciation expense per year than the existing machine.
The tax rate is 40%. Buying the new machine will increase annual net cash flows of the company by
a. $38,000.
b. $30,000.
c. $20,000.
d. $12,000.
c 10. Not-for-profit entities
a. cannot use capital budgeting techniques because profitability is irrelevant to them.
b. cannot use discounted cash flow techniques because the time value of money is irrelevant to
them.
c. might have serious problems in quantifying the benefits expected from an investment.
d. should use the IRR method to make investment decisions.
c 11. A major difference between an investment in working capital and one in depreciable assets is
that
an investment in working capital is never returned, while most depreciable assets have some residual
value.
b. an investment in working capital is returned in full at the end of a project's life, while an investment
in depreciable assets has no residual value.
c. an investment in working capital is not tax-deductible when made, nor taxable when returned,
while an investment in depreciable assets does allow tax deductions.
d. because an investment in working capital is usually returned in full at the end of the project's
life, it is ignored in computing the amount of the investment required for the project.
d 12. The proper treatment of an investment in receivables and inventory is to
a. ignore it.
b. add it to the required investment in fixed assets.
c. add it to the required investment in fixed assets and subtract it from the annual cash flows.
d. add it to the investment in fixed assets and add the present value of the recovery to the
present value of the annual cash flows.
a 13. If a company uses a five-year MACRS period to depreciate assets instead of a 10-year life with
straight-line depreciation,
a. the NPV of the investment is higher.
b. the IRR of the investment is lower.
c. there is no difference in either NPV or IRR.
d. total cash flows over the useful life would be lower.
a 14. The NPV and IRR methods give
a. the same decision (accept or reject) for any single investment.
b. the same choice from among mutually exclusive investments.
c. different rankings of projects with unequal lives.
d. the same rankings of projects with different required investments.
d 15. An investment with a positive NPV also has
a. a positive profitability index.
b. a profitability index of one.
c. a profitability index less than one.
d. a profitability index greater than one.
b 16. Classifying an asset in a MACRS life category is based on
a. useful life estimated by the company.
b. asset depreciation range (ADR) guidelines.
c. the cost of the asset.
d. any of the above factors.
d 17. Which of the following makes investments more desirable than they had been?
a. An increase in the income tax rate.
b. An increase in interest rates.
c. An increase in the number of years over which assets must be depreciated.
d. None of the above.
c 18. Which of the following statements is true?
a. All revenue is taxed.
b. All expenses are tax-deductible.
71

c. Some revenues and expenses have no tax effects.


d. Income taxes are based solely on revenues and expenses.
b 19. The profitability index is the ratio of
a. total cash inflows to the cost of the investment.
b. the present value of cash inflows to the cost of the investment.
c. the NPV of the investment to the cost of the investment.
d. the IRR to the company's cost of capital.
c 20. With respect to income taxes, the principal advantage of MACRS over straight-line depreciation
is that
a. total taxes will be lower under MACRS.
b. taxes will be constant from year to year under MACRS.
c. taxes will be lower in the earlier years under MACRS.
d. taxes will decline in future years under MACRS.
a 21. If the profitability index is less than one,
a. the IRR is less than cost of capital.
b. the IRR is the same as cost of capital.
c. the IRR is greater than cost of capital.
d. none of the above is true.
c 22. Which of the following combinations is possible?
Profitability Index NPV
IRR
------------------- -------- ------------------------a. greater than 1
positive equals cost of capital
b. greater than 1
negative less than cost of capital
c. less than 1
negative less than cost of capital
d. less than 1
positive less than cost of capital
d 23. Which of the following combinations is NOT possible?
Profitability Index NPV
IRR
------------------- -------- -------------------------a. greater than 1
positive more than cost of capital
b. equals 1
zero
equals cost of capital
c. less than 1
negative less than cost of capital
d. less than 1
positive less than cost of capital
b 24. In capital budgeting, sensitivity analysis is used
a. to determine whether an investment is profitable.
b. to see how a decision would be affected by changes in variables.
c. to test the relationship of the IRR and NPV.
d. to evaluate mutually exclusive investments.
b 25. A unique feature of the analysis of a replacement decision is that
a. the analysis considers total rather than differential costs.
b. the amount used as the cost of the investment is not likely to equal the price to be paid for the
new asset.
c. the time value of money is ignored.
d. such decisions seldom involve cash flows.
a 26. Because of idle capacity, a company is considering two assets for sale. They are identical in all
respects except that asset A has a higher tax basis than asset B. Only one need be sold now and the
market price is the same for both assets. Which of the following is true?
a. The cash flow is greater from selling asset A.
b. The cash flow is greater from selling asset B.
c. The cash flow is the same no matter which one is sold.
d. It is not possible to determine how the cash flows from sales of the assets will differ.
a 27. If the tax law were changed so that owners of apartment buildings had to depreciate them over
50 years instead of the current 31.5 years,
a. rents would rise.
b. rents would fall because annual depreciation charges would fall.
c. rents would stay about the same.
d. more people would invest in apartment buildings.
b 28. Which statement could express the results of a sensitivity analysis of an investment decision?
a. The NPV of the project is $50,000.
b. A 5% decline in volume will make the project unprofitable.
c. This project ranks third out of the five available.
72

d. This project does not meet the cutoff rate of return.


c 29. XYZ Co. is adopting just-in-time principles. When evaluating an investment project that would
reduce inventory, how should XYZ treat the reduction?
a. Ignore it.
b. Decrease the cost of the investment and decrease cash flows at the end of the project's life.
c. Decrease the cost of the investment.
d. Decrease the cost of the investment and increase the cash flow at the end of the project's life.
b 30. Which of the following combinations of capital budgeting techniques includes only discounted
cash flow techniques?
a. Book rate of return, payback, and profitability index.
b. NPV, IRR, and profitability index.
c. IRR, payback, and NPV.
d. Profitability index, NPV, and payback.
d 31. An investment whose profitability index is 1.00
a. has an IRR equal to the prevailing interest rate.
b. returns to the company only the cash outlay for the investment.
c. has a payback period equal to its useful life.
d. has an NPV of zero.
a 32. In connection with a capital budgeting project, an investment in working capital is normally
recovered
a. at the end of the project's life.
b. in the first year of the project's life.
c. evenly through the project's life.
d. when the company goes out of business.
b 33. For investments that have only costs (no revenues or cost savings), an appropriate decision rule
is to accept the project that has the
a. longest payback period.
b. lowest present value of cash outflows.
c. higher present value of future cash outflows.
d. lowest internal rate of return.
b 34. The cash inflow from the return of an investment in working capital is
a. adjusted for taxes due.
b. discounted to present value.
c. ignored if any depreciable assets also involved in the project have no expected residual value.
d. not real.
d 35. NPV is appropriate to use to analyze which decision relating to a joint-products company?
a. Whether or not to sell facilities now used for additional processing of one of the joint products.
b. Whether or not to acquire facilities needed for additional processing of one of the joint
products.
c. Whether or not to sell facilities now used to operate the joint process.
d. All of the above.
d 36. If X Co. expects to get a one-year bank loan to help cover the initial financing of capital project
Q, the analysis of Q should
a. offset the loan against any investment in inventory or receivables required by the project.
b. show the loan as an increase in the investment.
c. show the loan as a cash outflow in the second year of the project's life.
d. ignore the loan.
d 37. A project that has a negative NPV
a. has a payback period longer than its life.
b. has a negative profitability index.
c. must be rejected.
d. doesn't necessarily fit any of the above descriptions.
c 38. A company evaluates a project using straight-line depreciation over its 10-year estimated useful
life and then reevaluates it using a 7-year MACRS class life. The second analysis will show
a. a lower IRR for the project.
b. the same NPV and IRR for the project.
c. a higher NPV for the project.
d. lower total cash flows over the 10 years.
73

a 39. Assuming that a project has already been evaluated using the following techniques, the
evaluation under which technique is least likely to be affected by an increase in the estimated
residual value of the project?
a. Payback period.
b. IRR.
c. NPV.
d. PI.
d 40. Qualitative factors can influence managers to
a. accept an investment project having negative NPV.
b. reject an investment project having an IRR greater than the company's cutoff rate.
c. raise the "ranking" of an investment project.
d. take any of the above courses of action.
a 41. The replacement decision is
a. an example of a decision among mutually exclusive alternatives.
b. best arrived at by using the total-project rather than the differential approach.
c. devoid of qualitative issues.
d. none of the above.
c 42. Acme is considering the sale of a machine with a book value of $160,000 and 3 years remaining
in its useful life. Straight-line depreciation of $50,000 annually is available. The machine has a current
market value of $200,000. What is the cash flow from selling the machine if the tax rate is 40%?
a. $50,000
b. $160,000
c. $184,000
d. $200,000
c 43. Hoff is considering the sale of a machine with a book value of $160,000 and 3 years remaining
in its useful life. Straight-line depreciation of $50,000 annually is available. The machine has a current
market value of $100,000. What is the cash flow from selling the machine if the tax rate is 40%?
a. $50,000
b. $100,000
c. $124,000
d. $160,000
a 44. Altoona Company is considering replacing a machine with a book value of $200,000, a
remaining useful life of 4 years, and annual straight-line depreciation of $50,000. The existing
machine has a current market value of $175,000. The replacement machine would cost $320,000,
have a 4 year life, and save $100,000 per year in cash operating costs. If the replacement machine
would be depreciated using the straight-line method and the tax rate is 40%, what would be the
increase in annual income taxes if the company replaces the machine?
a. $28,000
b. $40,000
c. $42,000
d. $64,000
b 45. An investment opportunity costing $300,000 is expected to yield net cash flows of $100,000
annually for five years. The profitability index of the investment at a cutoff rate of 14% would be
a. 3.0.
b. 1.14.
c. 0.33.
d. 14%.
d 46. A project has a NPV of $30,000 when the cutoff rate is 10%. The annual cash flows are $41,010
on an investment of $100,000. The profitability index for this project is
a. 1.367.
b. 3.333.
c. 2.438.
d. 1.300.
c 47. A project has an IRR in excess of the cost of capital. The profitability index for this project would
be
a. less than zero.
b. between zero and one.
c. greater than one.
d. cannot be determined without more information.
74

b 48. A project has an IRR less than the cost of capital. The profitability index for this project would be
a. less than zero.
b. between zero and one.
c. greater than one.
d. cannot be determined without more information.
b 49. Portage Press Company is considering replacing a machine with a book value of $200,000, a
remaining useful life of 5 years, and annual straight-line depreciation of $40,000. The existing
machine has a current market value of $200,000. The replacement machine would cost $300,000,
have a 5-year life, and save $100,000 per year in cash operating costs. If the replacement machine
would be depreciated using the straight-line method and the tax rate is 40%, what would be the
increase in annual net cash flow if the company replaces the machine?
a. $60,000
b. $68,000
c. $76,000
d. $84,000
b 50. Winneconne Company is considering replacing a machine with a book value of $400,000, a
remaining useful life of 5 years, and annual straight-line depreciation of $80,000. The existing
machine has a current market value of $400,000. The replacement machine would cost $550,000,
have a 5-year life, and save $75,000 per year in cash operating costs. If the replacement machine
would be depreciated using the straight-line method and the tax rate is 40%, what would be the net
investment required to replace the existing machine?
a. $90,000
b. $150,000
c. $330,000
d. $550,000
True-False
T 1. The higher the IRR on an investment project, the higher its profitability index.
F 2. If the payback period of an investment project is shorter than its life, the project's profitability
index is greater than 1.
F 3. If a company has decided that a certain task must be performed and three machines accomplish
that task, the machine with the lowest initial cash outlay should be selected.
T 4. An investment with an IRR greater than cost of capital has a profitability index greater than 1.
T 5. The only costs and revenues relevant to a replacement decision are those that will change if a
replacement is made.
T 6. Both the incremental and the total-project approaches to analyzing a replacement decision
should yield the same decision.
F 7. Both the IRR and the book rate of return methods of analyzing investments should yield the
same decision.
F 8. If the payback period of an investment is shorter than its life, its profitability index is greater
than l.
T 9. When compared with straight-line depreciation, using MACRS will result in a larger NPV.
F 10. IRR and book rate of return will usually yield the same value for an investment.
Problems
1. Stockholm Company is considering the sale of a machine with the following characteristics.
Book value
$120,000
Remaining useful life
5 years
Annual straight-line depreciation $ 24,000
Current market value
$ 70,000
If the company sells the machine its cash operating expenses will increase by $30,000 per year due
75

to an operating lease. The tax rate is 40%.


a. Find the cash flow from selling the machine.
b. Calculate the increase in annual net cash outflows as a result of selling the machine.
SOLUTION:
a. Cash flow from sale: $90,000 ($70,000 + 40% tax savings on the $50,000 tax loss)
b. Increase in annual cash outflows: $27,600 ($30,000 pretax cost increase - $2,400 decrease in
income taxes; the $30,000 increase in cash costs is partially offset by losing a $24,000 depreciation
deduction)
2. Pepin Company is considering replacing a machine that has the following characteristics.
Book value
$100,000
Remaining useful life
5 years
Annual straight-line depreciation $ ???
Current market value
$ 60,000
The replacement machine would cost $150,000, have a five-year life, and save $50,000 per year
in cash operating costs. It would be depreciated using the straight-line method. The tax rate is 40%.
a. Find the net investment required to replace the existing machine.
b. Compute the increase in annual income taxes if the company replaces the machine.
c. Compute the increase in annual net cash flows if the company replaces the machine.
SOLUTION:
a. Net investment: $74,000 [$150,000 - $60,000 - 40%($100,000 - 60,000)]
b. Increase in income taxes: $16,000 [40% x ($50,000 pretax flow - $30,000 depreciation + $20,000
lost depreciation)]
c. Increase in cash flows: $34,000 ($50,000 - $16,000 increase in income taxes)
3. Cable Company is considering the purchase of a machine with the following characteristics.
Cost
$100,000
Useful life
10 years
Expected annual cash cost savings $30,000
Cable's income tax rate is 40% and its cost of capital is 12%. Cable expects to use straight-line
depreciation for tax purposes.
a. Compute the expected increase in annual net cash flow for this project.
b. Compute the profitability index for the project.
c. How would the profitability index for this project be affected if Cable were to use MACRS
depreciation for tax purposes and the machine fell into the 7-year MACRS class? (increase decrease
not affected) Circle the appropriate answer.
SOLUTION:
a. Increase in annual net cash flow: $22,000 [$30,000 - (40% x ($30,000 - $10,000)]
b. Profitability index: 1.24 [($22,000 x 5.65)/$100,000]
c. Effect on profitability index: Increase (PI would increase because the tax shield of depreciation
would occur earlier and so be more valuable when considering the time value of money.)
76

4. Frank Co. has the opportunity to introduce a new product. Frank expects the product to sell for $60
and to have per-unit variable costs of $35 and annual cash fixed costs of $4,000,000. Expected
annual sales volume is 275,000 units. The equipment needed to bring out the new product costs
$6,000,000, has a four-year life and no salvage value, and would be depreciated on a straight-line
basis. Frank's cost of capital is 14% and its income tax rate is 40%.
a. Compute the annual net cash flows for the investment.
b. Compute the NPV of the project.
c. Suppose that some of the 275,000 units expected to be sold would be to customers who currently
buy another of Frank's products, the X-10, which has a $12 per-unit contribution margin. Find the
sales of X-10 that can Frank lose per year and still have the investment in the new product return at
least the 14% cost of capital.
d. Suppose that selling the new product has no complementary effects but that Frank's production
engineers anticipate some production problems in making the new product and are not confident of
the $35 estimate of per-unit variable costs for the new product. Find the amount by which Frank's
estimate of per-unit variable cost could be in error and the investment still have a return at least
equal to the 14% cost of capital.
SOLUTION:
a. Annual net cash flows:
depreciation)]

$2,325,000

[$2,875,000 pretax - 40% x ($2,875,000 - $1,500,000

pretax income = 275,000 x ($60 - $35) - $4,000,000 = $2,875,000


b. NPV: $775,050 [($2,325,000 x 2.914) - $6,000,000]
c. Allowable loss of X-10 sales, approximately 36,941 units [($775,050/2.914)/60%]/12
d. Allowable error in per-unit VC, $1.61
{[($775,050/2.914)/60%]/275,000 units}
5. Zenex is considering the purchase of a machine. Data are as follows:
Cost
$240,000
Useful life
10 years
Annual straight-line depreciation
$ ???
Expected annual savings in cash
operation costs
$ 80,000
Additional working capital needed
$100,000
Zenex's cutoff rate is 12% and its tax rate is 40%.
a. Compute the annual net cash flows for the investment.
b. Compute the NPV of the project.
c. Compute the profitability index of the project.
SOLUTION:
a. Annual net cash flows: $57,600 [$80,000 pretax - 40% x ($80,000 - $24,000 depreciation)]
b. NPV: $17,640 [($57,600 x 5.650) - $240,000 - $100,000 + ($100,000 x .322)]
c. PI: 1.052 {[($57,600 x 5.650) + ($100,000 x .322)]/($240,000 + $100,000)}
6. Darwin Company is considering the sale of a machine with the following characteristics.
77

Book value
$110,000
Remaining useful life
5 years
Annual straight-line depreciation $ ???
Current market value
$120,000
If the company sells the machine its cash operating expenses will increase by $20,000 per year.
The tax rate is 40%.
a. Find the cash flow from selling the machine.
b. Calculate the increase in annual net cash outflows as a result of selling the machine.
SOLUTION:
a. Cash flow from sale: $116,000 ($120,000 - 40% tax on the $10,000 tax gain)
b. Increase in annual cash outflows: $20,800 ($20,000 pretax cost increase + $800 increase in
income taxes; the $20,000 increase in cash costs is more than offset by losing a $22,000 depreciation
deduction)
7. Rusk Company is considering replacing a machine that has the following characteristics.
Book value
$200,000
Remaining useful life
4 years
Annual straight-line depreciation $ ???
Current market value
$160,000
The replacement machine would cost $300,000, have a four-year life, and save $37,500 per year
in cash operating costs. It would be depreciated using the straight-line method. The tax rate is 40%.
a. Find the net investment required to replace the existing machine.
b. Compute the increase in annual income taxes if the company replaces the machine.
c. Compute the increase in annual net cash flows if the company replaces the machine.
SOLUTION:
a. Net investment: $124,000 [$300,000 - $160,000 - 40% x ($200,000 - 160,000)]
b. Increase in income taxes: $5,000
lost depreciation)]

[40% x ($37,500 pretax flow - $75,000 depreciation + $50,000

c. Increase in cash flows: $32,500 ($37,500 - $5,000 increase in income taxes)


8. Zmolek Company is considering the purchase of a machine costing $700,000 with a useful life of
10 years. Annual cash cost savings are expected to be $200,000. Zmolek's income tax rate is 40%
and its cost of capital is 12%. Zmolek expects to use straight-line depreciation for tax purposes.
a. Compute the expected increase in annual net cash flow for this project.
b. Compute the profitability index for the project.
SOLUTION:
a. Increase in annual net cash flow: $148,000 [$200,000 - 40% x ($200,000 - $70,000)]
b. Profitability index: 1.19 [($148,000 x 5.65)/$700,000]
9. Racine Co. has the opportunity to introduce a new product. Racine expects the project to sell for
$200 and to have per-unit variable costs of $130 and annual cash fixed costs of $6,000,000.
Expected annual sales volume is 125,000 units. The equipment needed to bring out the new product
costs $7,200,000, has a four-year life and no salvage value, and would be depreciated on a straight78

line basis. Working capital of $500,000 would be necessary to support the increased sales. Racine's
cost of capital is 12% and its income tax rate is 40%.
a. Compute the NPV of this opportunity.
b. Compute the profitability index of this opportunity.
SOLUTION:
a. NPV: negative $184,310
Annual cash flow: $2,370,000 = 60% x [125,000 x ($200 - $130)]
- 60% x $6,000,000 + 40% x $7,200,000/4
NPV: [($2,370,000 x 3.037) - $7,200,000 - 500,000 + ($500,000 x .636)]
b. PI: 0.976 [($2,370,000 x 3.037 + 500,000 x .636)/($7,200,000 + 500,000)]
10. Seiler is considering the purchase of a machine. Data are as follows:
Cost
$2,000,000
Useful life
8 years
Annual straight-line depreciation
$ ???
Expected annual savings in cash
operation costs
$ 750,000
Additional working capital needed
$ 500,000
Seiler's cutoff rate is 12% and its tax rate is 40%.
a. Compute the annual net cash flows for the investment.
b. Compute the NPV of the project.
c. Compute the profitability index of the project.
SOLUTION:
a. Annual net cash flows: $550,000 [$750,000 - 40% x ($750,000 - $250,000 depreciation)]
b. NPV: $434,400 [($550,000 x 4.968) - $2,000,000 - $500,000 + ($500,000 x .404)]
c. PI: 1.17 {[($550,000 x 4.968) + ($500,000 x .404)]/($2,000,000 + $500,000)}

CHAPTER 9: RESPONSIBILITY ACCOUNTING


Multiple Choice
c 1. Goal congruence exists when
a. the goals of the company harmonize with each other.
b. the company's managers are pursuing their own goals effectively.
c. the company's managers are pursuing the goals of the company.
d. all of the above are true.
c 2. Goal congruence is most likely to result when
a. reports to managers include all costs.
b. managers' behavior is affected by the criteria used to judge their performances.
c. performance evaluation criteria encourage behavior in the company's best interests as well as
in the manager's best interests.
d. a manager knows the criteria used to judge his or her performance.
d 3. In responsibility accounting the most relevant classification of costs is
a. fixed and variable.
b. incremental and nonincremental.
c. discretionary and committed.
d. controllable and noncontrollable.
79

c 4. Which of the following is critically important for a responsibility accounting system to be


effective?
a. Each employee should receive a separate performance report.
b. Service department costs should be allocated to the operating departments that use the
service.
c. Each manager should know the criteria used for evaluating his or her performance.
d. The details on the performance reports for individual managers should add up to the totals on
the report to their supervisor.
c 5. Which of the following items is LEAST likely to appear on the performance report of the manager
of a product line?
a. Variable manufacturing costs for products in the line.
b. Selling expenses for the line.
c. A share of company-wide advertising.
d. Revenues from the line.
b 6. The sequence that reflects increasing breadth of responsibility is
a. cost center, investment center, profit center.
b. cost center, profit center, investment center.
c. profit center, cost center, investment center.
d. investment center, cost center, profit center.
a 7. The criteria used for evaluating performance
a. should be designed to help achieve goal congruence.
b. can be used only with profit centers and investment centers.
c. should be used to compare past performance with current performance.
d. motivate people to work in the company's best interests.
b 8. A balanced scorecard approach to performance measurement
a. can only be used in profit or investment centers.
b. balances financial measures with nonfinancial measures.
c. uses only qualitative data to evaluate performance.
d. uses budgeted data rather than historical data.
b 9. If a company has a favorable sales volume variance, its
a. sales price variance is also favorable.
b. total contribution margin might be less than planned.
c. total contribution margin will be more than planned.
d. income will be positive.
c 10. Transfer prices
a. reduce employee turnover.
b. are necessary for investment centers.
c. should encourage the kinds of behavior that upper-level management wants.
d. are not used for departments with high amounts of fixed costs.
b 11. A transfer price is
a. an accounting device to turn profit centers into investment centers.
b. the price charged by one segment of the company for goods or services provided to another
segment.
c. only useful in a segment that deals with outsiders as well as with other segments of the same
company.
d. the amount charged by a cost center for a service performed for a profit center.
c 12. The cost allocation policy most likely to encourage use of a service is based on
a. budgeted total costs of the service department.
b. actual total costs of the service department.
c. budgeted variable costs for the service department.
d. actual variable costs for the service department.
c 13. Which of the following statements is true?
a. A company changes its total income when it changes the bases used to allocate indirect costs.
b. A company should select an allocation basis so as to raise or lower reported income on given
products.
c. A company's total income will remain unchanged no matter how indirect costs are allocated.
d. Costs should be allocated on an "ability-to-bear" basis.
a 14. If a company allocates costs of a service department to other departments, it should
a. consider the likely effects of the allocations on the use of the services.
80

b. use the method that best reflects the relative sizes of the departments.
c. turn the service department into an investment center.
d. allocate only the fixed costs of the service department.
a 15. If a computer department does work for other departments, charging a flat price per hour, the
computer department is
a. an artificial profit center.
b. a cost center.
c. an investment center.
d. none of the above.
a 16. The WORST method of allocating service department costs is
a. to allocate total actual costs based on actual use of the service.
b. to allocate total budgeted costs based on long-term expected use of the service.
c. to allocate total budgeted costs based on actual use of the service.
d. none of the above, because all the above are equally undesirable.
b 17. As a general rule, the best transfer price to use to transfer the costs of a service center to an
operating department is
a. the price charged by an outside company for the same service.
b. the price that encourages goal congruence.
c. one that is based on budgeted variable cost.
d. one that is based on budgeted total cost.
b 18. Which of the following costs is LEAST likely to appear on the performance report for the foreman
of a production department?
a. Wages of direct laborers.
b. Rent on machinery used in department.
c. Repairs to machinery used in department.
d. Cost of materials used.
d 19. ABC Company operates a factory that makes components for other ABC factories to assemble.
The factory could be treated as
a. a cost center.
b. an artificial profit center.
c. an investment center.
d. any of the above.
d 20. For reports to follow the principles of responsibility accounting, which of the following must be
true?
a. Each segment of the entity is an artificial profit center.
b. The company is decentralized.
c. The company uses transfer prices.
d. The reports show controllable costs separately from noncontrollable costs.
c 21. The effective use of responsibility accounting requires that performance reports for cost centers
a. show only variable costs.
b. show a fair share of allocated costs.
c. distinguish between controllable and noncontrollable costs.
d. show a fair share of revenues attributable to the center.
b 22. Criteria for evaluating performance should be carefully selected because
a. they must be approved by the IRS.
b. a manager's behavior can be affected by the criteria used to judge his or her performance.
c. managers may find out what they are.
d. stockholders inquire about them at annual meetings.
d 23. Which of the following is NOT a good reason for allocating indirect costs to operating
departments?
a. To remind managers of the need to cover indirect costs.
b. So that operating managers will encourage service department managers to keep costs down.
c. To encourage managers to use services wisely.
d. To determine the true costs of operating departments.
b 24. An artificial profit center
a. has no investment.
b. does not provide its goods or services outside the entity.
c. cannot control its costs.
d. could not be operated as a cost center.
81

c 25. A responsibility center is


a. any department.
b. any manager.
c. any area of activity for which a manager is responsible.
d. only large departments.
a 26. ABC's actual selling price was less than planned and actual unit volume more than planned.
Therefore,
a. ABC had a favorable sales volume variance.
b. ABC's total contribution margin was more than planned.
c. ABC had a favorable sales price variance.
d. ABC's actual total sales equaled planned total sales.
b 27. The term "dual rates" refers to
a. allocating costs to several operating departments.
b. allocating fixed costs based on capacity requirements and variable costs based on use.
c. allocating both actual costs and budgeted costs.
d. using the budgeted rate to allocate some costs, the actual rate to allocate others.
a 28. Which of the following methods of allocating the costs of service departments provides the
broadest recognition of departments served?
a. Reciprocal allocation.
b. Step-down allocation.
c. Direct allocation.
d. Arbitrary allocation.
d 29. Which of the following is a good reason for allocating indirect costs to operating departments?
a. The company could lose money if the operating departments do not pay for the services they
use.
b. To remind managers of the need to cover indirect costs.
c. To encourage managers to use more services.
d. To determine the true costs of operating departments.
b 30. When a manager takes an action that benefits his or her responsibility center, but not the
company as a whole,
a. it is a non-controllable action.
b. there is a lack of goal congruence.
c. the center must be an artificial profit center.
d. the manager should be fired.
d 31. Which of the following is a good reason for NOT allocating indirect costs to operating
departments?
a. The company saves money if the operating departments do not pay for the services they use.
b. To remind managers of the need to cover indirect costs.
c. To encourage managers to use more services.
d. The costs are not controllable by the operating departments.
d 32. Which of the following is a good reason for NOT allocating indirect costs to operating
departments?
a. To remind managers that revenues must cover indirect costs.
b. To recognize that operating departments benefit from the services.
c. To encourage managers to use services wisely.
d. Because allocating them might prompt operating managers to use nonincremental costs in
making decisions.
b 33. A profit center is a responsibility center
a. that sells its output outside the company.
b. whose manager is responsible for both revenues and costs.
c. that provides a service to other responsibility centers.
d. within an investment center.
d 34. An investment center is
a. larger than a cost center.
b. larger than a profit center.
c. seldom the responsibility of a single manager.
d. not truthfully characterized in any of the above statements.
a 35. The managerial level at which a particular cost is controllable
a. varies from company to company.
82

b. depends on whether the cost is fixed or variable.


c. depends on whether the cost is direct or indirect.
d. is irrelevant to the preparation of performance reports.
d 36. If at all possible, a manager's performance report should
a. consider the results that the manager can control.
b. consider only the results that the manager can control.
c. not be influenced by the results of decisions made by other managers.
d. reflect all of the above characteristics.
d 37. Comparing budgeted and actual amounts is important in evaluating the performance of
a. the manager of a cost center.
b. the manager of a profit center.
c. the manager of an investment center.
d. any manager.
c 38. Direct, step-down, and reciprocal are names for
a. the allocation methods most likely to produce goal congruence.
b. transfer-pricing methods.
c. methods for allocating costs of service departments to operating departments.
d. alternative organizational structures.
b 39. Cascade Company had the following results in June.
Planned
Actual
------------Sales
$80,000
$78,900
Variable costs
50,000
48,500
------------Contribution margin $30,000
$30,400
=======
=======
Planned sales were 10,000 units; actual sales were 9,700 units. The sales price variance is
a. $1,100 U.
b. $1,000 F.
c. $900 U.
d. $400 F.
c 40. Cascade Company had the following results in June.
Planned
Actual
------------Sales
$80,000
$78,900
Variable costs
50,000
48,500
------------Contribution margin $30,000
$30,400
=======
=======
Planned sales were 10,000 units, actual sales were 9,700 units. The sales volume variance is
a. $1,100 U.
b. $1,000 F.
c. $900 U.
d. $400 F.
b 41. Certainty Stores has three stores and one service center. The percentage of services used in
the current year are Store X, 35%; Store Y, 40%; and Store Z, 25%. The service center costs were
budgeted at $160,000 fixed and $240,000 variable. Actual fixed costs were $140,000 and actual
variable costs were $270,000. Actual service center costs are allocated to the stores based on actual
usage of the service center. Service center costs allocated to Store Y are
a. $64,000.
b. $164,000.
c. $410,000.
d. some other number.
c 42.
Certainty Stores has three stores and one service center. The percentage of services used in
the current year are Store X, 35%; Store Y, 40%; and Store Z, 25%. The service center costs were
budgeted at $350,000 fixed and $250,000 variable. Actual fixed costs were $370,000 and actual
variable costs were $280,000. Budgeted service center costs are allocated to the stores based on
actual usage of the service center. Service center costs allocated to Store Y are
83

a. $140,000.
b. $148,000.
c. $240,000.
d. $260,000.
c 43.
Wabasha Co. has two service departments (A and B) and two producing departments (X and
Y). Data provided are as follows:
Service Depts. Operating Depts.
-------------- --------------A
B
X
Y
------- ------ ------ -----Direct costs
$240 $400
Services performed by Dept. A
40% 40%
Services performed by Dept. B. 20%
70%

20%
10%

Wabasha uses the direct method to allocate service department costs. The service department
cost allocated to Department Y is
a. $88.
b. $96.
c. $130.
d. $240.
c 44.
Wabasha Co. has two service departments (A and B) and two producing departments (X and
Y). Data provided are as follows:
Service Depts. Operating Depts.
-------------- --------------A
B
X
Y
------- ------ ------ -----Direct costs
$250 $400
Services performed by Dept. A
40% 40%
Services performed by Dept. B. 20%
70%

20%
10%

Wabasha uses the step-down method to allocate service department costs. Department A costs
are allocated first. The service department cost allocated to Department Y is
a. $90.
b. $97.50.
c. $112.50.
d. $130.
c 45.
Wabasha Co. has two service departments (A and B) and two producing departments (X and
Y). Data provided are as follows:
Service Depts. Operating Depts.
-------------- --------------A
B
X
Y
------- ------ ------ -----Direct costs
$150 $300
Services performed by Dept. A
40% 40%
20%
Services performed by Dept. B. 20%
70%
10%
Wabasha uses the reciprocal method to allocate service department costs. The service
department cost allocated to Department Y is
a. $60.
b. $75.
c. $85.
d. $135.
d 46. Olson Stores has three stores and one service center. The percentage of services used in the
current year are Store A, 40%; Store B, 25%; and Store C, 45%. The expected long-term budgeted
usages are Store A, 30%; Store B, 30%; and Store C, 40%. The service center costs were budgeted at
$450,000 fixed and $550,000 variable. Actual fixed costs were $430,000 and actual variable costs
were $570,000. Olson allocates the budgeted variable costs of the central purchasing unit based on
actual use of the unit's services, and allocates budgeted fixed costs based on expected long-term use
of the unit's services. Service center costs allocated to Store A are
a. $135,000.
b. $220,000.
c. $300,000.
d. $355,000.
84

b 47. Olson Stores has three stores and one service center. The percentage of services used in the
current year are Store A, 45%; Store B, 35%; and Store C, 20%. The expected long-term budgeted
usages are Store A, 30%; Store B, 40%; and Store C, 30%. The service center costs were budgeted at
$450,000 fixed and $550,000 variable. Actual fixed costs were $430,000 and actual variable costs
were $570,000. Olson allocates the budgeted variable costs of the central purchasing unit based on
actual use of the unit's services, and allocates budgeted fixed costs based on expected long-term use
of the unit's services. Service center costs allocated to Store B are
a. $350,000.
b. $372,500.
c. $400,000.
d. $550,000.
d 48. Basin Co. has two service departments (A and B) and two producing departments (X and Y).
Data provided are as follows:
Service Depts. Operating Depts.
A
B
X
Y
------- ------ ------ -----Direct costs
$200 $400
Services performed by Dept. A
20% 40%
Services performed by Dept. B. 30%
60%

40%
10%

Basin uses the direct method to allocate service department costs. The service department cost
allocated to Department X is
a. $280.
b. $300.
c. $320.
d. $443.
a 49. Basin Co. has two service departments (A and B) and two producing departments (X and Y).
Data provided are as follows:
Service Depts. Operating Depts.
A
B
X
Y
------- ------ ------ -----Direct costs
$200 $400
Services performed by Dept. A
20% 40%
Services performed by Dept. B. 30%
60%

40%
10%

Basin uses the step-down method to allocate service department costs. Department A costs are
allocated first. The service department cost allocated to Department X is
a. $457.
b. $443.
c. $320.
d. $300.
c 50.
Basin Co. has two service departments (A and B) and two producing departments (X and Y).
Data provided are as follows:
Service Depts. Operating Depts.
A
B
X
Y
------- ------ ------ -----Direct costs
$200 $400
Services performed by Dept. A
20% 40%
Services performed by Dept. B. 30%
60%

40%
10%

Basin uses the reciprocal method to allocate service department costs. The service department
cost allocated to Department X is
a. $300.
b. $340.
c. $417.
d. $468.
True-False
F 1. All responsibility centers are either natural or artificial.
F 2. The sales volume variance is the difference between actual and planned unit sales multiplied by
85

the actual contribution margin per unit.


F 3. The principle of controllability is less important to the internal reporting for a centralized
company than for a decentralized one.
F 4. Allocated costs are less important to the internal reporting for a centralized company than for a
decentralized company.
F 5. Achieving goal congruence is less important in a centralized organization than in a decentralized
one.
T 6. It is not always possible to separate the variable and fixed components of actual costs.
F 7. A profit center will always have sales to outside customers.
T 8. The sales price variance is the difference between the actual selling price and the planned
selling price multiplied by actual units sold.
T 9. The direct method of allocating service department costs ignores all of the interactions between
service departments.
F 10. The reciprocal method of allocating service department costs considers only the usage by the
producing departments in determining the allocations.
Problems
1.

The following data are for Billings Stores, which has two stores and one service center.
Helena Butte
------- ----Percentage of services used in current year
20%
80%
Expected long-term use of services
30%
70%

Budgeted central purchasing costs were $225,000 fixed and $125,000 variable. Actual fixed costs
were $240,000 and actual variable costs were $115,000. The managers wish to allocate the actual
central purchasing costs to the stores based on actual use of the central purchasing service.
a. Compute the allocation to the Helena store.
b.Compute the allocation to the Butte store.
SOLUTION:
a.

To Helena: $71,000 [20% x ($240,000 + $115,000)]

b.

To Butte: $284,000 [80% x ($240,000 + $115,000)]

2.

The following data are for Billings Stores, which has two stores and one service center.
Helena Butte
------- ----Percentage of services used in current year
20%
80%
Expected long-term use of services
30%
70%

Budgeted central purchasing costs were $225,000 fixed and $125,000 variable. Actual fixed costs
were $240,000 and actual variable costs were $115,000. The company wishes to allocate the
budgeted variable costs of the central purchasing unit based on actual use of the unit's services and
to allocate budgeted fixed costs based on expected long-term use of the unit's services.
a. Compute the total cost allocated to the Helena store for the services of the central purchasing
unit.
b.Compute the total cost allocated to the Butte store for the services of the central purchasing unit.
SOLUTION:
a.

To Helena: $92,500 ($125,000 x 20% + $225,000 x 30%)

b.

To Butte: $257,500 ($125,000 x 80% + $225,000 x 70%)

3.

Following are data about Alphabet Co.'s two service departments and two operating departments.
86

Service Depts. Operating Depts.


-------------- --------------A
B
X
Y
------- ------ ------ -----Direct costs
$200 $500 $1,500 $2,000
Services performed by Dept. A
20%
40%
40%
Services performed by Dept. B. 10%
90%
a. Alphabet allocates costs of its service departments using the direct method of allocation. Find the
total cost that will be allocated to Dept. X.
b.Alphabet allocates the costs of its service departments using the step-down method, beginning
with Dept. A. Find the total amount of cost that will be allocated to Dept. X.
SOLUTION:
a.

Allocated to X: $600 [($200 x 40/(40 + 40)] + [$500 x (90/90)]

b.

Allocated to X: $620
A
B
X
Y
---- ---- ------A's direct cost
$200
A's cost allocated
(200) $ 40
$80
B's direct cost
500
----Total for allocating
$540
B's costs allocated
(540) 540
-----Allocated to X
$620
Allocated to Y
$80

4.

$80

Following are data about Alphabet Co.'s two service departments and two operating departments.
Service Depts. Operating Depts.
-------------- --------------A
B
X
Y
------- ------ ------ -----Direct costs
$400 $1,000 $3,000 $4,000
Services performed by Dept. A
20%
40%
40%
Services performed by Dept. B. 10%
90%
-

Alphabet allocates costs of its service departments using the reciprocal method of allocation. Find
the total cost that will be allocated to Dept. X.
SOLUTION:
Allocated to X: $1,195.92
A = $400 + .1B A = 510.20
B = $1,000 + .2A B = 1,102.04
A
B
X
Y
------- ------- ------------Direct costs
$400.00 $1,000.00
A's cost allocated
(510.20) 102.04 $204.08
B's costs allocated
110.20 (1,102.04) 991.84
------------Allocated to X
$1,195.92
Allocated to Y
$204.08
5.

$204.08
0

The following data are for Lexington Stores, which has two stores and one service center.
Concord Graham
------- -----Percentage of services used in current year
40%
60%
Expected long-term use of services
30%
70%

Budgeted central purchasing costs were $100,000 fixed and $75,000 variable. Actual fixed costs
were $140,000 and actual variable costs were $105,000. The managers wish to allocate the actual
central purchasing costs to the stores based on actual use of the central purchasing service.
a. Compute the allocation to the Concord store.
87

b.Compute the allocation to the Graham store.


SOLUTION:
a.

To Concord: $98,000 [40% x ($140,000 + $105,000)]

b.

To Graham: $147,000 [60% x ($140,000 + $105,000)]

6.

The following data are for Lexington Stores, which has two stores and one service center.
Concord Graham
------- -----Percentage of services used in current year
40%
60%
Expected long-term use of services
30%
70%

Budgeted central purchasing costs were $100,000 fixed and $75,000 variable. Actual fixed costs
were $140,000 and actual variable costs were $105,000. The company wishes to allocate the
budgeted variable costs of the central purchasing unit based on actual use of the unit's services and
to allocate budgeted fixed costs based on expected long-term use of the unit's services.
a. Compute the total cost allocated to the Concord store for the services of the central purchasing
unit.
b.Compute the total cost allocated to the Graham store for the services of the central purchasing
unit.
SOLUTION:
a.

To Concord: $60,000 ($75,000 x 40% + $100,000 x 30%)

b.

To Graham: $115,000 ($75,000 x 60% + $100,000 x 70%)

7.

Following are data about Hamilton Co.'s two service departments and two operating departments.
Service Depts. Operating Depts.
-------------- --------------A
B
X
Y
------- ------ ------ -----Direct costs
$400 $600 $2,000 $3,000
Services performed by Dept. A
30%
30%
40%
Services performed by Dept. B. 20%
70%
10%

a. Hamilton allocates costs of its service departments using the direct method of allocation. Find the
total cost that will be allocated to each of the operating departments.
b.Hamilton allocates the costs of its service departments using the step-down method, beginning
with Dept. A. Find the total amount of cost that will be allocated to each of the operating
departments.
c. Hamilton allocates costs of its service departments using the reciprocal method of allocation. Find
the total cost that will be allocated to each of the operating departments.
SOLUTION:
a.

Allocated to X: $696.43 {$400 x [30/(30 + 40)] + $600 x [70/(70 + 10)]}


Allocated to Y: $303.57 {$400 x [40/(30 + 40)] + $600 x [10/(70 + 10)]}

b.

Allocated to X: $750.00,
Allocated to Y: $250.00
A
B
X
Y
---- ---- ------- ------A's direct cost
$400
A's cost allocated
(400) $120 $120.00 $160.00
B's direct cost
600
---Total for allocating
$720
B's costs allocated
(720) 630.00 90.00
------- -----Allocated to X
$750.00
Allocated to Y
$250.00
88

c.

Allocated to X: $702.13,
Allocated to Y: $297.87
A = $400 + .2B A = 553.19
B = $600 + .3A B = 765.96
A
B
X
Y
------- ------- ------------Direct costs
$400.00 $600.00
A's cost allocated
(553.19) 165.96 $165.96
B's costs allocated
153.19 (765.96) 536.17
------------Allocated to X
$702.13
Allocated to Y
$297.87

$221.27
76.60

8. Following are data about Hawley Co.'s two service departments and three operating departments.
Service Depts.
Operating Depts.
-------------- ---------------------A
B
X
Y
Z
------- ------ ------ ------ -----Direct costs
$400 $600
Services performed by Dept. A
30% 40% 20%
10%
Services performed by Dept. B. 40%
20% 20%
20%
Hawley allocates costs of its service departments using the reciprocal method of allocation. Find
the total costs that will be allocated to each of the operating departments.
SOLUTION:
Allocated to x: $454.55, allocated to Y: $309.09, Allocated to Z: $236.36
A = $400 + .4B A = 727.27
B = $600 + .3A B = 818.18
A
B
X
Y
Z
------- ------- ------- ------- ------Direct costs
$400.00 $600.00
A's cost allocated
(727.27) 218.18 $290.91 $145.45 $ 72.72
B's costs allocated
327.27 (818.18) 163.64 163.64 163.64
------- ------- ------Allocated to X
$454.55
Allocated to Y
$309.09
Allocated to Z
$236.36
9. Following are data about Augusta Co.'s three service departments and two operating
departments.
Service Depts.
Operating Depts.
--------------------- ---------------A
B
C
X
Y
------- ------ ------ ------ -----Direct costs
$150 $300 $350
Services performed by Dept. A
20% 30% 40%
10%
Services performed by Dept. B. 10%
20% 50%
20%
Services performed by Dept. C 30%
40%
15%
15%
a. Augusta allocates costs of its service departments using the direct method of allocation. Find the
total cost that will be allocated to Dept. X.
b.Augusta allocates the costs of its service departments using the step-down method, beginning
with Dept. A followed by Dept. B. Find the total amount of cost that will be allocated to Dept. X.
SOLUTION:
a. Allocated to X: $509.29
15)]}
b.

{$150 x [40/(40 + 10)] + $300 x [50/(50 + 20)] + $350 x [15/(15 +

Allocated to X: $477.50
A
B
C
X
Y
---- ---- ------- ------- ------A's direct cost
$150
A's cost allocated
(150) $ 30 $ 45.00 $ 60.00 $ 15.00
B's direct cost
300
89

---Total for allocating


B's costs allocated
C's direct cost
Total for allocating
C's costs allocated
Allocated to X
Allocated to Y

$330
(330) 73.33 183.34 73.33
350.00
------$468.33
(468.33) 234.16 234.16
------ -----$477.50
$322.50

10. Osseo Company had the following results in June.


Planned
Actual
--------------Sales
$160,000
$162,500
Variable costs at $5 per unit
100,000
102,500
--------------Contribution margin
$ 60,000
$ 60,000
========
========
Planned sales were 20,000 units, actual sales were 20,500 units.
a. Find the sales price variance. Indicate F or U
b. Find the sales volume variance. Indicate F or U
SOLUTION:
a. $1,500 U {20,500 x [($162,500/20,500) - $8]}
b. $1,500 F [$3 x (20,500 - 20,000)]
CHAPTER 10: Divisional Performance MEASUREMENT
Multiple Choice
c 1. Both ROI and RI can be used for performance evaluation of
a. cost centers.
b. profit centers.
c. investment centers.
d. all of the above.
b 2. The best transfer price is usually
a. actual cost plus a percentage markup.
b. a reliable market price.
c. budgeted full cost plus a percentage markup.
d. budgeted variable cost plus a percentage markup.
d 3. This year Division A made sales to Division B at a higher transfer price than was used last year.
All other things equal, which of the following is true?
a. A's profit this year should be about the same as last year.
b. B's profit this year should be about the same as last year.
c. The company's total profit should be higher this year than last year.
d. The company's total profit should be about the same this year as last year.
b 4. Goal congruence is especially relevant to all of the following EXCEPT
a. setting transfer prices for an artificial profit center.
b. quoting prices for outside customers of an investment center.
c. selecting costs to be included in performance reports.
d. setting transfer prices for an investment center.
d 5. For a division, ROI
a. is usually less than ROI for the company as a whole.
b. eliminates the distortion that cost allocation can produce in other measures of performance.
c. usually cannot be computed if divisional assets are valued at their replacement costs.
d. is a performance measure inferior, for some purposes, to residual income.
a 6. Divisional ROI is usually
90

a. higher than that for the company as a whole.


b. lower than that for an outside company operating in the same industry.
c. lower than return on sales for the division.
d. lower than that for the enterprise as a whole.
c 7. Divisional profits should
a. exclude revenues and expenses related to dealings with other divisions within the same
enterprise.
b. be computed so that the total profits of all the divisions equals the total profit for the company.
c. be based on the principle of controllability.
d. be based on cash flows rather than accrual basis accounting.
c 8. Divisional profit
a. is computed in essentially the same way as is income for the company as a whole.
b. should include a deduction for an appropriate share of the company's common costs.
c. normally includes the results of intracompany sales.
d. is not affected by depreciation methods.
d 9. Using replacement costs for assets in computing ROI and RI
a. is prohibited because it violates generally accepted accounting principles.
b. will increase both ROI and RI for a division.
c. is unfair to divisional managers.
d. is less popular than the use of book values in those computations.
c 10. Using residual income for evaluating performance
a. penalizes managers whose segments have low ROIs.
b. penalizes managers of relatively large segments.
c. encourages managers to maximize dollars of profit after a required ROI has been achieved.
d. encourages managers to maximize ROI for the company.
c 11. Which item is usually NOT relevant to a decision by a divisional manager to reduce a transfer
price to meet a price offered to another division by an outside supplier?
a. Opportunity cost.
b. Variable manufacturing costs.
c. Fixed divisional overhead.
d. The price offered by the outside supplier.
c 12. Division A earns $6,000 on an investment of $36,000. On an investment of $84,000, Division B
earns $12,000. Which of the following is true?
a. Division A's profits are too low.
b. If there are further costs that are common to both divisions, the total company's ROI is
probably greater than 15%.
c. If the minimum desired ROI is 10%, Division A's residual income is lower than that of Division B.
d. ROI for Division B is greater than ROI for Division A.
d 13. Which equation describes ROI? (I = investment, S = sales, and
N = income)
a. S/I
b. S/I x N
c. S/I x S/N
d. N/S x S/I
a 14. Which equation describes residual income? (I = investment, N = income, and K = minimum
required ROI)
a. N - (K x I)
b. (K x I) - N
c. N/I - K
d. (K x I) - (N/I)
c 15. If Division C has a 10% return on sales, income of $10,000, and an investment turnover of 4
times, its sales are
a. $10,000.
b. $40,000.
c. $100,000.
d. $400,000.
b 16. If Division C has a 10% return on sales, income of $10,000, and an investment turnover of 4
times, divisional investment is
a. $10,000.
91

b. $25,000.
c. $40,000.
d. $100,000.
c 17. If Division C has a 10% return on sales, income of $10,000, and an investment turnover of 4
times, its ROI is
a. 5000%.
b. 100%.
c. 40%.
d. 10%.
b 18. If a division's ROI and the minimum required ROI are the same, the division's residual income is
a. positive.
b. zero.
c. negative.
d. none of the above.
a 19. If residual income for Division Q of Company Z is negative, which of the following is true?
a. Q's ROI is less than Z's minimum required ROI.
b. Q's ROI equals Z's minimum required ROI.
c. Q's ROI is higher than Z's minimum required ROI.
d. None of the above.
b 20. Market-based transfer prices are best for
a. the company when the selling division is operating below capacity.
b. the company when the selling division is operating at capacity.
c. the buying division if it is operating at capacity.
d. the buying division.
d 21. The worst transfer-pricing method is to base the prices on
a. market prices.
b. budgeted variable costs.
c. budgeted total costs.
d. actual total costs.
d 22. All other things remaining constant, if a division doubles its investment turnover, its ROI will
a. decrease.
b. remain constant.
c. increase.
d. double.
c 23. Residual income
a. is always the best measure of divisional performance.
b. is not as good a measure of performance as ROI.
c. overcomes some of the problems associated with ROI.
d. cannot be used by divisions that deal with others in the same company.
b 24. If two divisions earn the same ROI and RI, which of the following is true?
a. Their managers must be about equally skillful.
b. Their incomes and investments must be the same.
c. Both divisions are doing as well as they should be.
d. All of the above.
c 25. Which of the following is most likely to be included in calculating divisional profit?
a. Interest on corporate debt.
b. Income taxes.
c. Sales to other divisions within the company.
d. A share of corporate administration expenses.
b 26. If sales increase, while income and investment remain constant, which of the following is true?
a. Investment turnover decreases.
b. ROS decreases.
c. ROI increases.
d. ROI could increase or decrease.
c 27. Compared to a jewelry store, a supermarket has
a. higher margin and higher turnover.
b. higher margin and lower turnover.
c. lower margin and higher turnover.
d. lower margin and lower turnover.
92

c 28. If income increases while sales and investment remain constant, which of the following is true?
a. Investment turnover increases.
b. ROS decreases.
c. ROI increases.
d. ROI could increase or decrease.
a 29. Which transfer price is ideal for the company when the selling division is at capacity?
a. Market price.
b. Incremental cost.
c. Budgeted full cost.
d. Actual variable cost plus a percentage profit.
c 30. From the standpoint of the company, the important question in transfer pricing is
a. what is fair to the divisions.
b. how to determine the profit of the divisions.
c. whether or not the transfer should take place.
d. when the transfer should be made.
d 31. The ROI of Division A relative to that of Division B can be influenced by
a. the industry in which each division operates.
b. the transfer price used for sales to Division B.
c. the tax structures of the countries in which the divisions operate.
d. all of the above.
c 32. Considering liabilities in computing divisional investment
a. encourages managers of divisions to pay their bills faster.
b. discourages managers of divisions from acquiring long-term financing.
c. raises divisional ROI above what it would otherwise be.
d. is a bad managerial practice.
d 33. Interdivisional sales
a. lower the company's public image.
b. minimize income taxes.
c. are ignored when computing divisional ROI.
d. do none of the above.
a 34. Which of the following is true about transfer prices for sales between divisions located in
different countries?
a. They should consider the tax structures in the two countries.
b. They are usually set by the governments of the two countries.
c. They cannot affect the total income of the company.
d. All of the above.
d 35. Multinational companies face special problems in which of the following areas of managerial
practice?
a. Performance evaluation.
b. Transfer prices.
c. Allocating common costs.
d. All of the above.
b 36. Which of the following describes the computation of ROI?
a. Return on Sales x Investment
b. Investment Turnover x Return on Sales
c. Income - (Investment x Minimum RI)
d. Sales x Investment Turnover
b 37. If the investment turnover increased by 20% and ROS decreased by 30%, the ROI would
a. increase by 20%.
b. decrease by 16%.
c. increase by 4%.
d. none of the above.
b 38. Scottso Division has the following results for the year:
Revenues
$1,080,000
Variable expenses
440,000
Fixed expenses
400,000
93

Total divisional assets are $1,600,000.


percent. Residual income for Scottso is
a. $(64,000).
b. $16,000.
c. $151,200.
d. $224,000.

The company's minimum required rate of return is 14

c 39. Scottso Division has the following results for the year:
Revenues
$1,080,000
Variable expenses
440,000
Fixed expenses
400,000
Total divisional assets are $1,600,000. The company's minimum required rate of return is 14
percent. Return on investment for Scottso is
a. 54%.
b. 18%.
c. 15%.
d. 10%.
b 40. Monrovia Division has net income of $240,000 on sales of $3,200,000. If the investment is
$1,600,000 what is ROS?
a. 15.0%
b. 7.5%
c. 10.0
d. 2.0
c 41. Scottso Division has the following results for the year:
Revenues
$1,080,000
Variable expenses
440,000
Fixed expenses
400,000
Total divisional assets are $1,600,000.
percent. Return on sales for Scottso is
a. 1.5%.
b. 15.0%.
c. 22.2%.
d. 67.5%.

The company's minimum required rate of return is 14

d 42. Monrovia Division has net income of $240,000 on sales of $3,200,000. If the investment is
$1,600,000 what is asset turnover?
a. 15.0%
b. 7.5%
c. 10.0
d. 2.0
a 43. Monrovia Division has net income of $240,000 on sales of $3,200,000. If the investment is
$1,600,000 what is ROI?
a. 15.0%
b. 7.5%
c. 10.0
d. 2.0
b 44. Alcatraz Division of XYZ Corp. sells 80,000 units of part X to the outside market. Part X sells for
$40, has a variable cost of $22, and a fixed cost per unit of $10. Alcatraz has a capacity to produce
100,000 units per period. Capone Division currently purchases 10,000 units of part X from Alcatraz for
$40. Capone has been approached by an outside supplier willing to supply the parts for $36. What is
the effect on XYZ's overall profit if Alcatraz REFUSES the outside price and Capone decides to buy
outside?
a. no change
b. $140,000 decrease in XYZ profits
c. $80,000 decrease in XYZ profits
d. $40,000 increase in XYZ profits
a 45. Alcatraz Division of XYZ Corp. sells 80,000 units of part X to the outside market. Part X sells for
$40, has a variable cost of $22, and a fixed cost per unit of $10. Alcatraz has a capacity to produce
100,000 units per period. Capone Division currently purchases 10,000 units of part X from Alcatraz for
$40. Capone has been approached by an outside supplier willing to supply the parts for $36. What is
94

the effect on XYZ's overall profit if Alcatraz ACCEPTS the outside price and Capone continues to buy
inside?
a. no change
b. $140,000 decrease in XYZ profits
c. $80,000 decrease in XYZ profits
d. $40,000 increase in XYZ profits
c 46. If the investment turnover decreased by 20% and ROS decreased by 30%, the ROI would
a. increase by 30%.
b. decrease by 20%.
c. decrease by 44%.
d. none of the above.
c 47. If the investment turnover increased by 10% and ROS increased by 20%, the ROI would
a. increase by 10%.
b. increase by 20%.
c. increase by 30%.
d. increase by 32%.
b 48. Durand Division has the following results for the year:
Revenues
Net income

$470,000
130,000

Total divisional assets are $625,000. The company's minimum required rate of return is 12
percent. Residual income for Durand is
a. $3,760.
b. $55,000.
c. $73,600.
d. cannot be determined without further information.
c 49. Durand Division has the following results for the year:
Revenues
Net income

$470,000
130,000

Total divisional assets are $625,000. The company's minimum required rate of return is 12
percent. Return on investment for Durand is
a. 9.0%.
b. 18.3%.
c. 20.8%.
d. 27.7%.
d 50. Durand Division has the following results for the year:
Revenues
Net income

$470,000
130,000

Total divisional assets are $625,000.


percent. Return on sales for Durand is
a. 9.0%.
b. 18.3%.
c. 20.8%.
d. 27.7%.

The company's minimum required rate of return is 12

True-False
F 1. Multinational companies cannot use transfer prices.
T 2. Long-term debt is seldom considered in determining divisional ROI.
F 3. The measure most commonly used for evaluating divisional performance is investment turnover.
T 4. Allocating all common assets, liabilities, and costs to divisions does not affect the ROI of the
company as a whole.
T 5. Using residual income as a criterion for evaluating divisional performance requires that the
company establish a minimum desired rate of return on investment.
95

F 6. Return on investment is the product of return on sales and inventory turnover.


F 7. Return on investment for a multidivision company will be lower than the ROI for the division with
the lowest ROI.
T 8. Transfer prices equal to market prices are least appropriate when the selling division has excess
productive capacity.
F 9. Multinational companies must use transfer prices based on actual costs.
F 10. Return on investment is defined as net income divided by stockholders' equity.
Problems
1. The following information is available about the status and operations of A-Klop Company, which
has a minimum required ROI of 15%. ANSWER EACH ITEM INDEPENDENTLY OF THE OTHERS.
Division Division
A
B
---------- ---------Divisional investment $ 500,000 $1,500,000
Divisional profit
$ 150,000 $ 540,000
Divisional sales
$1,000,000 $3,600,000
a. Compute ROI for Division A.
b. Compute residual income for Division B.
c. Division A could increase its profit by $40,000 by increasing its investment by $150,000.
Compute its total residual income.
d. Division A could increase its return on sales by one percentage point, while keeping the same
total sales and investment. Compute its ROI.
e. Division B could reduce its investment so that its asset turnover increased by one time, while
holding total sales constant. Compute its ROI.
SOLUTION:
a. ROI for A: 30% ($150,000/$500,000)
b. RI for B: $315,000 [$540,000 - ($1,500,000 x 15%)]
c. RI for A: $92,500 [$150,000 + $40,000 - 15% x ($500,000 + $150,000)]
d. ROI for A:
32%
[$150,000/$1,000,000 = 15% ROS + 1% = 16%, turnover = 2
($1,000,000/$500,000), so 16% x 2 = 32%]
e. ROI for B:
51%
[$3,600,000/1,500,000 = 2.4 times + 1 = 3.4 times x ROS of 15%
($540,000/$3,600,000) = 51%]
2. Division A of Getz Company expects the following results.
INDEPENDENTLY.
To Division B To Outsiders
------------- -----------Sales (40,000 x $10)
$400,000
(40,000 x $12)
$480,000
Variable costs at $6
240,000
240,000
--------------Contribution margin
$160,000
$240,000
Fixed costs, all common, allocated
on the basis of relative units
120,000
120,000
--------------Profit
$ 40,000
$120,000
========
=======

ANSWER EACH QUESTION

Division B has the opportunity to buy its needs for 40,000 units from an outside supplier at $8
each.
a. Division A refuses to meet the $8 price, sales to outsiders cannot be increased, and Division B
buys from the outside supplier. Compute the effect on the income of Getz.
96

b. Division A cannot increase its sales to outsiders, does meet the $8 price, and Division B continues
to buy from A. Compute the effect on the income of Getz.
c. Suppose that Division A could sell the 40,000 units now taken by Division B to outsiders at $9
each without disturbing sales at the regular $12 price. Division B buys outside at $8 and Division A
increases its outside sales. Find the effect on the income of Getz.
SOLUTION:
a. Getz's income: Decreases $80,000 [40,000 units x ($8 outside price - $6 variable cost)]
b. Getz's income: No change
c. Getz's income: $40,000 increase
outsider by B)

($360,000 added revenue from outsiders - $320,000 paid to the

3. The following information relates to Zimmer Division of Purdy Inc. Purdy's desired ROI for its
segments is 20%.
Sales
$2,000,000 Variable costs
Direct fixed costs 300,000 Investment

1,500,000
500,000

a. Find Zimmer's ROI.


b. Find Zimmer's residual income.
SOLUTION:
a. ROI: 40% [($2,000,000 - $1,500,000 - $300,000)/$500,000]
b. RI: $100,000 [$200,000 - ($500,000 x 20%)]
4. Bayfield Division of Ashland Inc. has a capacity of 200,000 units and expects the following results.
Sales (160,000 units at $4) $640,000
Variable costs, at $2
(320,000)
Fixed costs
(260,000)
-------Income
$ 60,000
========
Washburn Division of Ashland Inc. currently purchases 50,000 units of a part for one of its
products from an outside supplier for $4 per unit. Washburn's manager believes he could use a minor
variation of Bayfield's product instead, and offers to buy the units from Bayfield at $3.50. Making the
variation desired by Washburn would cost Bayfield an additional $0.50 per unit and would increase
Bayfield's annual cash fixed costs by $20,000. BAYFIELD'S MANAGER AGREES TO THE DEAL OFFERED
BY WASHBURN'S MANAGER.
a. Find the effect of the deal on Washburn's income and circle the correct direction. (increase
decrease none)
b. Find the effect of the deal on Bayfield's income and circle the correct direction.
decrease none)

(increase

c. Find the effect of the deal on the income of Ashland Inc. and circle the correct direction.
(increase decrease none)
SOLUTION:
a. Washburn's income, + $25,000 [50,000 x ($4 - $3.50)]
b. Bayfield's income, + $10,000 {50,000 x ($3.50 - $2 - $0.50) - [lost contribution margin of 10,000
x ($4 - $2)] - $20,000 new fixed costs}
c. Ashland's income, + $35,000 ($25,000 + $10,000)
5. Crosby Division has the following information for the most recent period:
Divisional income

$ 1,500,000
97

Divisional investment $ 6,500,000


Divisional sales
$12,000,000
Crosby has a minimum required return of 18%.
a. Compute Crosby's return on investment.
b. Compute Crosby's investment turnover.
c. Compute Crosby's residual income.
d. Compute Crosby's return on sales.
SOLUTION:
a. ROI: 23.1% ($1,500,000/$6,500,000)
b. IT: 1.85 ($12,000,000/$6,500,000)
c. RI: $330,000 [$1,500,000 - (18% x $6,500,000)]
d. ROS: 12.5% ($1,500,000/$12,000,000)
6. The following information is available about the status and operations of Stills Company, which
has a minimum required ROI of 20%. ANSWER EACH ITEM INDEPENDENTLY OF THE OTHERS.
Division Division
A
B
-------- ---------Divisional investment
$400,000 $1,250,000
Divisional profit
$120,000 $ 580,000
Divisional sales
$800,000 $2,600,000
a. Compute ROI for Division B.
b. Compute residual income for Division A.
c. Division B could increase its profit by $80,000 by increasing its investment by $300,000.
Compute its total residual income.
d. Division A could increase its return on sales by one percentage point, while keeping the same
total sales. Compute its ROI.
e. Division A could increase its sales so that its asset turnover increased by one time, while holding
total assets constant. Compute its ROI.
SOLUTION:
a. ROI for B: 46.4% ($580,000/$1,250,000)
b. RI for A: $20,000 [$120,000 - ($400,000 x 20%)]
c. RI for B: $350,000 [$580,000 + $80,000 - 20% x ($1,250,000 + $300,000)]
d. ROI for A: 32% [$120,000/$800,000 = 15% ROS + 1% = 16%, turnover = 2 ($800,000/$400,000),
so 16% x 2 = 32%]
e. ROI for A: 45% [$800,000/400,000 = 2 times + 1 = 3 times x ROS of 15% ($120,000/$800,000) =
45%]
7. Division A of Nash Company expects the following results.
INDEPENDENTLY.
To Division B To Outsiders
------------- -----------Sales (5,000 x $60)
$300,000
(25,000 x $72)
$1,800,000
Variable costs at $36
180,000
900,000
---------------Contribution margin
$120,000
$ 900,000
98

ANSWER EACH QUESTION

Fixed costs, all common, allocated


on the basis of relative units
60,000
300,000
---------------Profit
$ 60,000
$ 600,000
========
==========
Division B has the opportunity to buy its needs for 5,000 units from an outside supplier at $45
each.
a. Division A refuses to meet the $45 price, sales to outsiders cannot be increased, and Division B
buys from the outside supplier. Compute the effect on the income of Nash.
b. Division A cannot increase its sales to outsiders, does meet the $45 price, and Division B
continues to buy from A. Compute the effect on the income of Nash.
c. Suppose that Division A could sell the 5,000 units now taken by Division B to outsiders at $57
each without disturbing sales at the regular $72 price. Division B buys outside at $45 and Division A
increases its outside sales. Find the effect on the income of Nash.

SOLUTION:
a. Nash's income: Decreases $45,000 [5,000 units x ($45 outside price - $36 variable cost)]
b. Nash's income: No change
c. Nash's income: $60,000 increase ($285,000 added revenue from outsiders - $225,000 paid to the
outsider by B)
8. The following information relates to Bradley Division of Allen Company. Allen's minimum cost of
capital for its segments is 15%.
Sales
$4,000,000 Variable costs
Direct fixed costs 1,800,000 Investment

1,400,000
4,800,000

a. Find Bradley's ROI.


b. Find Bradley's economic value added.
SOLUTION:
a. ROI: 16.7% [($4,000,000 - $1,400,000 - $1,800,000)/$4,800,000]
b. EVA: $80,000 [$800,000 - ($4,800,000 x 15%)]
9. Rosalie Division of Lachene Inc. has a capacity of 100,000 units and expects the following results
for.
Sales (90,000 units at $30)
$2,700,000
Variable costs, at $20
(1,800,000)
Fixed costs
(700,000)
--------Income
$ 200,000
==========
Katarina Division of Lachene Inc. currently purchases 20,000 units of a part for one of its products
from an outside supplier at $32 per unit. Katarina's manager believes he could use a minor variation
of Rosalie's product instead, and offers to buy the units from Rosalie at $26. Making the variation
desired by Katarina would cost Rosalie an additional $5 per unit and would increase Rosalie's annual
cash fixed costs by $80,000. ROSALIE'S MANAGER AGREES TO THE DEAL OFFERED BY KATARINA'S
MANAGER.
a. Find the effect of the deal on Katarina's income and circle the correct direction. (increase
decrease none)
b. Find the effect of the deal on Rosalie's income and circle the correct direction.
decrease none)
99

(increase

c. Find the effect of the deal on the income of Lachene Inc. and circle the correct direction.
(increase decrease none)
SOLUTION:
a. Katarina's income, + $120,000 [20,000 x ($32 - $26)]
b. Rosalie's income, - $160,000 {20,000 x ($26 - $20 - $5) - [lost contribution margin of 10,000 x
($30 - $20)] - $80,000 new fixed costs)}
c. Lachene's income, - $40,000 ($120,000 - $160,000)
10. Young Division has the following information for the most recent period:
Divisional income
$ 11,000,000
Divisional investment
$ 85,000,000
Divisional sales
$100,000,000
Young has a minimum required return of 15%
a. Compute Young's return on investment.
b. Compute Young's investment turnover.
c. Compute Young's residual income.
d. Compute Young's return on sales.
SOLUTION:
a. ROI: 12.9% ($11,000,000/$85,000,000)
b. IT: 1.18 ($100,000,000/$85,000,000)
c. RI: ($1,750,000) [$11,000,000 - (15% x $85,000,000)]
d. ROS: 11.0% ($11,000,000/$100,000,000)
CHAPTER 11: CONTROL AND EVALUATION OF COST CENTERS
Multiple Choice
c 1. The two general types of variable cost variances are the
a. rate variance and spending variance.
b. price variance and budget variance.
c. price variance and quantity variance.
d. quantity variance and efficiency variance.
d 2. A major advantage of using standard costs is that
a. they are easier to compute than actual costs.
b. they are lower than actual costs.
c. products with standard costs can be sold at lower prices.
d. they provide information for control purposes.
a 3. Setting standards
a. has important behavioral implications.
b. is largely a matter of calculating rates and quantities.
c. should be done to make them as tight as possible.
d. is done only for manufacturing activities.
d 4. Which of the following is NOT a quantity variance?
a. Material use variance.
b. Labor efficiency variance.
c. Variable overhead efficiency variance.
d. Fixed overhead budget variance.
a 5. A major drawback to setting standards based on historical results is that such standards
a. can perpetuate inefficiencies.
b. are harder to compute than are engineered standards.
c. are usually too hard to meet because of inflation.
d. are usually not well received by workers.
100

b 6. Cascade Company, which has a $3 standard cost per unit and budgeted production at 1,000
units, actually produced 1,200 units. Total standard cost for the period is
a. $3,000.
b. $3,600.
c. an amount that cannot be determined without knowing the variances for the period.
d. none of the above.
c 7. Which variance is LEAST likely to be affected by hiring workers with less skill than those already
working?
a. Material use variance.
b. Labor rate variance.
c. Material price variance.
d. Variable overhead efficiency variance.
c 8. Which variance is MOST likely to be affected by buying a more expensive material that produces
less waste and is easier to handle?
a. Labor rate variance.
b. Variable overhead spending variance.
c. Direct labor efficiency variance.
d. Fixed overhead budget variance.
b 9. Filter Company's budget for overhead costs is:
total overhead cost = $50,000 + ($4 x direct labor hours)
Standard direct labor time is 1.5 hours per unit of product. The standard wage rate is $6 per
hour. Standard variable overhead cost for a unit of product is
a. $4.00.
b. $6.00.
c. $9.00.
d. $10.00.
b 10. The major variance used in controlling fixed costs is the
a. efficiency variance.
b. budget variance.
c. use variance.
d. none of the above.
d 11. If the variable overhead standard is based on direct labor hours and actual hours worked exceed
standard hours allowed, the result is
a. a favorable labor efficiency variance.
b. an unfavorable variable overhead spending variance.
c. a favorable variable overhead spending variance.
d. an unfavorable variable overhead efficiency variance.
a 12. Control charts are used
a. to decide whether to investigate variances.
b. to develop standard costs.
c. to calculate variances.
d. for all of the above purposes.
c 13. An unfavorable labor efficiency variance
a. means that workers were inefficient and their supervisor did a poor job.
b. causes a favorable variable overhead efficiency variance.
c. can result from an action taken by a manager other than the supervisor of the workers.
d. should always be investigated and corrected.

d 14. The sum of the material price variance and material use variance always equals the difference
between
a. actual and standard material purchases.
b. actual material purchases and standard material use.
c. standard material purchases and standard material use.
d. none of the above pairs of amounts.
c 15. Which set of terms describes the same type of variance?
a. Price variance, rate variance, use variance.
b. Price variance, rate variance, efficiency variance.
101

c. Use variance, efficiency variance, quantity variance.


d. Use variance, efficiency variance, spending variance.
d 16. A product requires 0.60 standard labor hours, the standard labor rate is $10 per hour, and
production was 300 units. Actual labor cost was $1,862 at $9.80 per hour. Which of the following is
true?
a. The labor rate variance was $98 favorable.
b. The labor rate variance was $62 unfavorable.
c. The labor efficiency variance was $62 unfavorable.
d. The labor efficiency variance was $100 unfavorable.
d 17. Cascade Company bought 10,000 pounds of material and used 9,500. The material price
variance was $300 unfavorable and the standard price per pound is $3. The cost of materials
purchased was
a. $28,200
b. $28,800
c. $29,700
d. $30,300
c 18. The standard price of a material is $2 per pound. The company bought 2,000 pounds at $1.90
per pound and used 1,700 pounds. Standard use was 1,800 pounds. The material price variance was
a. $170 favorable.
b. $180 favorable.
c. $200 favorable.
d. $400 favorable.
c 19. A company made 1,200 units with a $550 favorable labor use variance. There was no labor rate
variance and actual labor cost was $19,250. The actual wage rate was $11. Standard labor time per
unit is
a. 0.5 hours
b. 1.0 hour
c. 1.5 hours
d. 2.0 hours
a 20. Which formula calculates a price or rate variance? (AQ = actual quantity of the factor, AP =
actual price of the factor, SQ = standard price of the factor, SQ = standard quantity of the factor)
a. (AQ x AP) - (AQ x SP).
b. (AQ x AP) - (SQ x AP).
c. (AQ x SP) - (SQ x SP)
d. (AQ x SP) - (SQ x AP).
b 21. Which formula calculates a use or efficiency variance? (AQ = actual quantity of the factor, AP =
actual price of the factor, SQ = standard price of the factor, SQ = standard quantity of the factor)
a. (AQ x AP) - (SQ x SP).
b. (AQ x SP) - (SQ x SP).
c. (AQ x AP) - (AQ x SP)
d. (SQ x SP) - (SQ x AP).
b 22. Using variances to evaluate performance
a. is especially useful to JIT companies.
b. can be misleading because of interdependence among variances.
c. cannot be used with activity-based overhead standards.
d. all of the above.
b 23. Using activity-based costing in setting standards
a. is most valuable for direct labor standards.
b. should provide better variable overhead standards.
c. is unnecessary.
d. gives the same standards that traditional methods do.
c 24. A purchasing manager bought cheaper-than-normal materials that are difficult to handle. Which
combination of variances is LEAST likely to be affected by this decision?
a. Material use and direct labor use.
b. Material use, direct labor use, and variable overhead efficiency.
c. Direct labor rate and variable overhead budget.
d. Direct labor use and variable overhead efficiency.
d 25. Standard costs are useful for
a. planning.
102

b. control.
c. performance evaluation.
d. all of the above.
b 26. Which kinds of variances should be investigated?
a. Those that are large and unfavorable.
b. Those that are large and either favorable or unfavorable.
c. All variances, despite their size.
d. Only use variances.
b 27. The material price variance is calculated
a. the same as the labor rate variance.
b. on the quantity of materials bought, not the quantity used.
c. on the quantity of materials used, not the quantity bought.
d. by multiplying the difference between the actual and standard price of materials times the
quantity of materials used.
b 28. Probably the best level at which to set standards is
a. historical performance.
b. currently attainable performance.
c. ideal performance.
d. any of the above.
d 29. The use of ideal standards
a. motivates workers to perform well.
b. results in mostly favorable variances.
c. is preferred by most managers.
d. can cause performance to suffer.
d 30. An 80% learning curve means that
a. the incremental time for each unit is 80% of the time of the unit before it.
b. the cumulative average time is 80% of the cumulative average time at the previous unit.
c. as production doubles, the incremental time for a unit is 80% of the time at the previous
doubling point.
d. as production doubles, the cumulative average time is 80% of the time at the previous doubling
point.
b 31. In which company is the learning effect probably most important?
a. A canner of orange juice.
b. A manufacturer of airplanes.
c. A highly automated chemical manufacturer.
d. A manufacturer of nails.
a 32. Which of the following is NOT a reason why some JIT operations do not use standards?
a. Standards are often set too tight for JIT operations.
b. Using standards can stifle continuous improvement.
c. Standards focus on cost centers, not on the entire manufacturing operation.
d. All of the above are reasons.
d 33. The role of activity-based costing in standard costs is to
a. determine the standard material content of products.
b. find value-adding activities.
c. determine the variable overhead rate per direct labor hour.
d. identify drivers of overhead costs.
a 34. A company that uses activity-based costing to develop standard costs
a. will usually have more than one variable overhead component in its standard costs.
b. cannot compute variable overhead efficiency variances.
c. will have less information about the profitability of individual products.
d. all of the above.
d 35. Advanced manufacturers
a. are especially concerned with material price variances.
b. almost always seek out the least expensive vendors.
c. use more materials than conventional manufacturers.
d. are generally more concerned with quality and delivery than with price.
b 36. For a company whose variable overhead relates to direct labor, the variable overhead efficiency
103

variance
a. results from efficient or inefficient use of variable overhead elements.
b. results from efficient or inefficient use of direct labor.
c. is always the same as the direct labor efficiency variance.
d. is more like a budget variance than a use variance.
b 37. Acme Company produced 500 units with a $50 unfavorable labor rate variance. The labor use
variance was $180 favorable. Actual labor cost was $17,870. The standard wage rate was $9. Actual
hours were
a. 1,520
b. 1,980
c. 2,000
d. 2,020
a 38. Crunch Company expects a 90% learning curve. The first batch of a new product required 1,000
hours. The total time for the first four batches should be
a. 3,240 hours.
b. 3,600 hours.
c. 4,000 hours.
d. some other number of hours.
c 39. Crunch Company expects a 90% learning curve. The first batch of a new product required 10
hours. The first four batches should take an average of
a. 10 hours.
b. 9 hours.
c. 8.1 hours.
d. some other number of hours.
c 40. Acme has a standard of 15 parts of component X costing $1.50 each. Acme purchased 14,910
units of X for $21,950. Acme generated a $415 favorable price variance and a $3,735 favorable
quantity variance. If there were no changes in the component inventory, how many units of finished
product were produced?
a. 994 units
b. 1,000 units
c. 1,160 units
d. some other number
b 41. Acme has a standard price of $6 per pound for materials. July's results showed an unfavorable
material price variance of $44 and a favorable quantity variance of $228. If 1,066 pounds were used
in production, what was the standard quantity allowed for materials?
a. 1,066
b. 1,104
c. 1,294
d. some other number
c 42. Genco paid $78,800 to direct labor for the production of 1,500 units. Standards allow 2 labor
hours per unit at a rate of $25.00 per hour. Actual hours totaled 2,900. The direct labor rate variance
was
a. $2,050 favorable
b. $3,800 favorable
c. $6,300 unfavorable
d. some other number
a 43. Genco paid $78,800 to direct labor for the production of 1,500 units. Standards allow 2 labor
hours per unit at a rate of $25.00 per hour. Actual hours totaled 2,900. The direct labor efficiency
variance was
a. $2,500 favorable
b. $3,800 favorable
c. $6,300 unfavorable
d. some other number
c 44. Danner had a $550 favorable direct labor rate variance and a $720 unfavorable efficiency
variance. Danner paid $6,650 for 800 hours of labor. What was the standard direct labor wage rate?
a. $8.10
b. $8.31
c. $9.00
d. some other number
104

c 45. Jeter's Company had a $510 unfavorable direct labor rate variance and a $1,000 favorable
efficiency variance. Jeter's standard payroll was $11,200 at a standard wage of $10 per hour. What
was the actual direct labor wage rate?
a. $9.56
b. $10.00
c. $10.50
d. some other number
a 46. Chippewa paid $32,225 to direct labor for the production of 1,700 units. Standards allow 3 labor
hours per unit at a rate of $6.50 per hour. Actual hours totaled 5,150. The direct labor rate variance
was
a. $1,250 favorable
b. $925 favorable
c. $325 favorable
d. $325 unfavorable
d 47. Chippewa paid $32,225 to direct labor for the production of 1,700 units. Standards allow 3 labor
hours per unit at a rate of $6.50 per hour. Actual hours totaled 5,150. The direct labor efficiency
variance was
a. $1,250 favorable
b. $925 favorable
c. $325 favorable
d. $325 unfavorable
a 48. Chetek Company has standard variable costs as follows:
Materials, 3 pounds at $4.00 per pound $12.00
Labor, 2 hours at $10.00 per hour
20.00
Variable overhead, $7.50 per labor hour
15.00
$47.00
During September, Chetek produced 5,000 units, using 9,640 labor hours at a total wage of
$94,670 and incurring $78,600 in variable overhead. The variable overhead budget variance is
a. $6,300 unfavorable
b. $3,600 unfavorable
c. $2,700 favorable
d. some other number
c 49. Barron Company has standard variable costs as follows:
Materials, 3 pounds at $4.00 per pound $12.00
Labor, 2 hours at $10.00 per hour
20.00
Variable overhead, $7.50 per labor hour
15.00
$47.00
During September, Barron produced 5,000 units, using 9,640 labor hours at a total wage of
$94,670 and incurring $78,600 in variable overhead. The variable overhead efficiency variance is
a. $6,300 unfavorable
b. $3,600 unfavorable
c. $2,700 favorable
d. $3,300 favorable
c 50.
Silver Bow manufactured the first batch of product in 100 hours. The second batch took an
additional 60 hours. What percent learning occurred?
a. 100%
b. 90%
c. 80%
d. Cannot be determined with the information given.
True-False
T 1. Standard costs are per-unit expressions of flexible budget allowances based on output.
F 2. The value of b, the exponent in the learning curve formula, is the learning rate.
T 3. The use of ideal standards could have undesirable effects on output.
F 4. Learning curves can only be used for a one-of-a-kind special order product.
F 5. So long as total actual costs approximate total budgeted costs there is no need for managerial
concern or action.
105

T 6. It is not always possible to separate the variable and fixed components of actual overhead cost.
F 7. When an unfavorable variance occurs, there is some action that some manager can take to
correct the event or circumstance that gave rise to the variance.
F 8. Standard costs are devices for measuring effectiveness but not efficiency.
F 9. "Ideal standards" are those most likely to be met under most conditions.
T 10. The labor efficiency variance excludes the effects of laborers being paid more or than the
standard labor rate.
Problem
1. The data below relate to a product of Salois Company.
Standard costs:
Materials, 2 pounds at $6 per pound
$12per unit
Labor, 3 hours at $15 per hour $45per unit
Variable overhead at $8 per labor hour $24per unit
Budgeted fixed production costs
$140,000 per year
Budgeted production for the year 4,000 units
Actual results were:
Production 3,700 units
Material purchases, 8,000 pounds $ 46,400
Labor, 10,360 hours
$160,580
Variable overhead incurred $ 84,700
Fixed overhead incurred
$137,500
Material used in production 7,300 pounds
For each variance, determine the amount and circle the correct direction,
F = favorable, U = unfavorable
a.

Material price variance. F

b.

Material use variance. F

c.

Direct labor rate variance. F

d.

Direct labor efficiency variance.F

e.

Variable overhead budget variance. F

f.

Variable overhead efficiency variance. F

g.

Fixed overhead budget variance.

U
U

U
U

SOLUTION:
a.

MPV

$1,600 F

$46,400 - ($6 x 8,000)

b.

MUV

$600 F

c.

DLRV $5,180 U

d.

DLEV $11,100 F

($15 x 10,360) - ($45 x 3,700)

e.

VOHBV $1,820 U

$84,700 - ($8 x 10,360)

f.

VOHEV $5,920 F

($8 x 10,360) - ($24 x 3,700)

g.

FOHBV $2,500 F

$137,500 - $140,000

($6 x 7,300) - ($12 x 3,700)


$160,580 - ($15 x 10,360)

2. The data below relate to a product of Conroy Company.


Standard costs:
Materials, 3 pounds at $4 per pound

$12per unit
106

Labor, 5 hours at $12 per hour $60per unit


Variable overhead at $7 per labor hour $35per unit
Budgeted fixed production costs
$150,000 per year
Budgeted production for the year 5,000 units
Actual results were:
Production 5,200 units
Material purchases, 15,000 pounds $ 63,220
Labor, 24,860 hours
$301,620
Variable overhead incurred $ 168,600
Fixed overhead incurred
$152,760
Material used in production 15,300 pounds
For each variance, determine the amount and circle the correct direction,
F = favorable, U = unfavorable
a.

Material price variance. F

b.

Material use variance. F

c.

Direct labor rate variance. F

d.

Direct labor efficiency variance.F

e.

Variable overhead budget variance. F

f.

Variable overhead efficiency variance. F

g.

Fixed overhead budget variance.

U
U

U
U

SOLUTION:
a.

MPV

$3,220 U

$63,220 - ($4 x 15,000)

b.

MUV

$1,200 F

($4 x 15,300) - ($12 x 5,200)

c.

DLRV $3,300 U

$301,620 - ($12 x 24,860)

d.

DLEV $13,680 F

($12 x 24,860) - ($60 x 5,200)

e.

VOHBV $5,420 F

$168,600 - ($7 x 24,860)

f.

VOHEV $7,980 F

($7 x 24,860) - ($35 x 5,200)

g.

FOHBV $2,760 U

$152,760 - $150,000

3.

Anne's Arbors has the following budget and actual results for May:
Budget Actual
----------Unit production 9,000 9,600
Direct labor hours 11,250 11,550
Materials used, feet15,750 16,100

Standard labor rate is $12 per hour; standard material price is $4.50 per foot. Actual wages were
$140,250; actual material purchases were 17,500 pounds for $77,160.
For each variance, determine the amount and circle the correct direction,
F = favorable, U = unfavorable
a.

Material price variance. F

b.

Material use variance. F

c.

Direct labor rate variance. F

d.

Direct labor efficiency variance.F

U
U
107

SOLUTION:
a. MPV:
$1,590 F

$77,160 - ($4.50 x 17,500)

b.

MUV:

$3,150 F

$4.50 x (16,100 - [1.75 x 9,600])

c.

DLRV:

$1,650 U

$140,250 - ($12 x 11,550)

d.

DLEV:

$5,400 F

$12 x (11,550 - [1.25 x 9,600])

4.

North Company has the following budget and actual results for July:
Budget Actual
----------Unit production 10,000 8,400
Direct labor hours 12,000 11,860
Materials used, feet16,000 16,750

Standard labor rate is $14 per hour; standard material price is $7.50 per foot. Actual wages were
$168,750; actual material purchases were 18,800 pounds for $149,825.
For each variance, determine the amount and circle the correct direction,
F = favorable, U = unfavorable
a.

Material price variance. F

b.

Material use variance. F

c.

Direct labor rate variance. F

d.

Direct labor efficiency variance.F

U
U

SOLUTION:
a.

MPV:

$ 8,825 U

$149,825 - ($7.50 x 18,800)

b.

MUV:

$24,825 U

$7.50 x (16,750 - [1.6 x 8,400])

c.

DLRV:

$ 2,710 U

$168,750 - ($14 x 11,860)

d.

DLEV:

$24,920 U

$14 x (11,860 - [1.2 x 8,400])

5.

The data below relate to a product of Acme Company.

Standard costs:
Materials, 5 yards at $3 per pound $15per unit
Labor, 3 hours at $14 per hour $42per unit
Variable overhead at $10 per labor hour
$30per unit
Budgeted fixed production costs
$175,000 per year
Budgeted production for the year 7,700 units
Actual results were:
Production 6,300 units
Material purchases, 31,700 yards $ 80,890
Labor, 17,660 hours
$252,330
Variable overhead incurred $ 178,300
Fixed overhead incurred
$172,200
Material used in production 31,600 yards
For each variance, determine the amount and circle the correct direction,
F = favorable, U = unfavorable
a.

Material price variance. F

b.

Material use variance. F

c.

Direct labor rate variance. F

U
108

d.

Direct labor efficiency variance.F

e.

Variable overhead budget variance. F

f.

Variable overhead efficiency variance. F

U
U

SOLUTION:
a.

MPV

$14,210 F

$80,890 - ($3 x 31,700)

b.

MUV

$ 300 U

c.

DLRV

$ 5,090 U

$252,330 - ($14 x 17,660)

d.

DLEV

$17,360 F

($14 x 17,660) - ($42 x 6,300)

e.

VOHBV $ 1,700 U

f.

VOHEV $11,160 U

($3 x 31,600) - ($15 x 6,300)

$178,300 - ($10 x 17,660)


($9 x 17,660) - ($27 x 6,300)

6. Toimi Inc. had the following variances for the most recent month:
Materials Price Variance
$3,500 U
Materials Usage Variance
$ 720 F
Direct Labor Rate Variance
$5,770 F
Direct Labor Efficiency Variance $6,980 U
Other information included: actual wages paid $72,310; materials purchased $130,760; standards
per unit were 2 labor hours at $5 per hour, 3 pounds at $6 per pound. There were no changes in
materials inventories.
a.

Find the units produced.

b.

Find the standard labor hours.

c.

Find the actual labor hours.

d.

Find the standard quantity of materials allowed.

e.

Find the actual quantity of materials used.

SOLUTION:
a.

7,110 units ($72,310 + 5,770 - 6,980) / (2 x $5)

b.

14,220 SH $71,100 / $5

c.

15,616 AH $78,080 / $5

d.

21,330 lbs ($130,760 - 3,500 + 720) / $6

e.

21,210 lbs $127,260 / $6

7.

Ralph Inc. had the following variances for the most recent month:
Direct Labor Rate Variance
$14,560 U
Direct Labor Efficiency Variance
$ 3,660 U
Variable Overhead Spending Variance
$12,320 F

Other information included: actual wages paid $105,560; materials purchased $124,860;
standards per unit were 2 labor hours at $5 per hour and variable overhead at $6 per hour.
a.

Find the units produced.

b.

Find the standard labor hours.

c.

Find the actual labor hours.


109

d.

Find the variable overhead efficiency variance.

Find the actual variable overhead.


SOLUTION:
a.

8,734 units ($105,560 - 14,560 - 3,660) / (2 x $5)

b.

17,468 SH 8,734 x 2

c.

18,200 AH ($105,560 - 14,560) / $5

d.

$4,392 U

(18,200 - 17,468) x 6

e.

$96,880

(18,200 x $6) - $12,320

8. Gros Ventre Company expects a learning rate of 80%.


expected to take 500 direct labor hours.

The first batch of a new product is

a. Compute the cumulative average time for the first four batches.
b.Compute the total time for the first four batches.

SOLUTION:
a.
b.

9.

320 [500 hours x 80% x 80%]


1,280
Output (X) Average time (Y)
Total time (XY)
1
500
500
2
400 (500 x 80%)
800 (2 x 400)
4
320 (400 x 80%)
1,280 (4 x 320)
Benco Inc. has the following results for December when production was 8,000 units:
Materials purchased
18,000 pounds $213,520
Materials used
17,750 pounds
Direct labor
27,050 hours $436,544

Per unit standards are 2.5 pounds of materials at $12.00 per pound and 3.5 hours at $16 per
hour.
For each variance, determine the amount and circle the correct direction,
F = favorable, U = unfavorable
a.

Material price variance. F

b.

Material use variance. F

c.

Direct labor rate variance. F

d.

Direct labor efficiency variance.F

U
U

SOLUTION:
a.

MPV

$ 2,480 F

$213,520 - ($12 x 18,000)

b.

MUV

$27,000 F

($12 x 17,750) - ($12 x 2.5 x 8,000)

c.

DLRV

$ 3,744 U

$436,544 - ($16 x 27,050)

d.

DLEV

$15,200 F

($16 x 27,050) - ($16 x 3.5 x 8,000)

10. Cascade Company expects a learning rate of 90%. The first batch of a new product is expected to
take 200 direct labor hours.
110

a. Compute the cumulative average time for the first eight batches.
b.Compute the total time for the first eight batches.
SOLUTION:
a.
b.

145.8 (200 hours x 90% x 90% x 90%)


1,166.4
Output (X) Average time (Y)
Total time (XY)
1
200
200
2
180 (200 x 90%)
360 (2 x 180)
4
162 (180 x 90%)
648 (4 x 162)
8
145.8 (162 x 90%)
1,166.4 (8 x 145.8)

Multiple Choice

CHAPTER 12: INTRODUCTION TO PRODUCT COSTING

b 1. The three major components of manufacturing cost are


a. materials, work in process, and finished goods.
b. materials, labor, and manufacturing overhead.
c. materials, labor, and finished goods.
d. materials, labor, and production costs.
a 2. A predetermined overhead rate CANNOT be used
a. if a company does not budget its overhead costs.
b. by a company that uses job-order costing.
c. in a multiple-product company.
d. by a highly automated company where labor is a minor part of product cost.
a 3. Assigning overhead to jobs using a predetermined overhead rate is called
a. applying.
b. budgeting.
c. product costing.
d. job-order costing.
d 4. Predetermined overhead rates are based on activity measured by
a. number of jobs.
b. units of sales.
c. units of production.
d. units of an input factor.
b 5. Absorption costing is required
a. for financial accounting purposes only.
b. for financial accounting and tax purposes.
c. for financial and managerial accounting purposes.
d. for managerial accounting purposes only.
a 6. Both actual and normal costing
a. include material, labor, and overhead in product cost.
b. require predetermined overhead rates.
c. are likely to result in over- or underabsorption of overhead.
d. will show a favorable volume variance if production is greater than budgeted.
b 7. An advantage of normal costing over actual costing is that
a. the company can compute the exact cost of a unit of product.
b. unit costs are not affected by monthly fluctuations in production activity.
c. control over fixed costs is improved.
d. the company need not forecast the level of productive activity for the year.
d 8. An unfavorable volume variance signifies that
a. cost control was poor.
b. sales were less than budgeted.
c. production was less than sales.
d. production was less than the level used to set the fixed overhead application rate.
111

b 9. The principal reason for using more than one rate to apply overhead is
a. to keep the individual rates low.
b. that overhead costs are driven by more than one activity.
c. that such rates recognize the seasonal nature of some costs.
d. to simplify recordkeeping.
a 10. Activity-based overhead rates are more useful than a single plant-wide rate if
a. overhead costs are driven by several activities.
b. direct labor cost varies significantly from department to department.
c. all products require about the same amounts of all activities.
d. manufacturing overhead costs are nearly all fixed.
c 11. Actual costing and normal costing differ in treating
a. materials cost.
b. direct labor cost.
c. overhead cost.
d. all of the above.
d 12. Which company is LEAST likely to use job-order costing?
a. Furniture maker.
b. Printer.
c. Construction company.
d. Flour maker.
d 13. A cost pool is
a. all costs of a production department.
b. the material and labor cost used on a particular job.
c. overapplied or underapplied overhead costs.
d. a group of overhead costs driven by the same activity.
c 14. Each group of overhead costs should be applied based on
a. direct labor hours or cost.
b. units produced.
c. whatever activity drives those specific overhead costs.
d. machine time.
d 15. A POOR reason to use activity-based overhead rates is that
a. some departments are labor-intensive, some are machine-intensive.
b. significant amounts of overhead are driven by different factors.
c. rates calculated for some departments are much higher than for other departments.
d. all jobs require about the same amounts of cost-driving activities.
a 16. XYZ had an $8,000 unfavorable volume variance, a $11,500 unfavorable variable overhead
spending variance, and $1,500 total underapplied overhead. The fixed overhead budget variance was
a. $18,000 favorable.
b. $21,000 favorable.
c. $17,500 unfavorable.
d. $21,000 unfavorable.
d 17. Machine hours used to set the predetermined overhead rate were 25,000, actual hours were
24,000, and overhead applied was $60,000. Budgeted overhead for the year was
a. $57,600.
b. $59,000.
c. $60,000.
d. $62,500.
c 18. The three inventory accounts in a manufacturing company are
a. Materials, Labor, and Manufacturing Overhead.
b. Materials, Labor, and Finished Goods.
c. Materials, Work in Process, and Finished Goods.
d. Materials, Finished Goods, and Inventory Sold.
a 19. A predetermined overhead rate is calculated using
a. budgeted overhead cost and budgeted activity.
b. actual overhead cost and actual activity.
c. budgeted overhead cost and budgeted direct labor hours.
d. budgeted overhead cost and budgeted direct labor cost.
d 20. Under normal costing, income can be
112

a. lower than under actual costing.


b. higher than under actual costing.
c. the same as under actual costing.
d. any of the above.
a 21. The numerator in computing a predetermined overhead rate is
a. budgeted manufacturing overhead cost.
b. actual manufacturing overhead cost.
c. budgeted activity.
d. fixed manufacturing overhead.
c 22. The denominator in calculating a predetermined overhead rate is
a. budgeted manufacturing overhead cost.
b. actual manufacturing overhead cost.
c. budgeted activity.
d. actual activity.
c 23. In a factory operated largely by robots, the best basis for applying overhead is probably
a. direct labor hours.
b. direct labor cost.
c. machine hours.
d. raw material use.
a 24. Spooner applies overhead based on direct labor cost. It had budgeted manufacturing overhead
of $50,000 and budgeted direct labor of $25,000. Actual overhead was $52,500, actual labor cost
was $27,000. Overhead was
a. overapplied by $1,500.
b. overapplied by $2,000.
c. overapplied by $2,500.
d. underapplied by $2,000.
c 25. Hayward applies overhead at $5 per machine hour. During March it worked 10,000 hours and
overapplied overhead by $3,000. Actual overhead was
a. $53,000.
b. $50,000.
c. $47,000.
d. none of the above.
b 26. Aurora applies overhead at $9 per direct labor hour of which $4 is variable overhead. Budgeted
direct labor hours were 80,000. Budgeted fixed overhead was
a. $320,000
b. $400,000.
c. $720,000.
d. none of the above.
a 27. Which is a good reason to use separate overhead rates?
a. Some departments are labor-intensive, some are machine-intensive.
b. Labor rates vary considerably among departments.
c. The resulting overhead rates are all about the same.
d. All jobs require about the same percentage of time in all departments.
c 28. In contrast to a company that uses a single overhead rate, one that uses activity-based costing
a. will have higher product costs than one using a single overhead rate.
b. cannot compute budget variances.
c. will incur additional costs for recordkeeping.
d. must have a preponderance of fixed overhead costs.
a 29. Which of the following is a sign of poor cost control?
a. A high unfavorable budget variance.
b. A high unfavorable volume variance.
c. High underapplied overhead.
d. High overapplied overhead.

c 30. Hoyt Company applies overhead at $4 per direct labor hour. In March Hoyt incurred overhead of
$96,000. Underapplied overhead was $4,000. How many direct labor hours did Hoyt work?
a. 25,000
b. 24,000
113

c. 23,000
d. 22,000
d 31. A company using activity-based overhead rates
a. will usually have higher budget variances than one using a single rate.
b. will usually have higher volume variances than one using a single rate.
c. cannot compute fixed and variable components of overhead cost.
d. should have better information for planning and control than one using a single rate.
a 32. Daly had a $9,000 favorable volume variance, a $7,500 unfavorable variable overhead spending
variance, and $6,000 total overapplied overhead. The fixed overhead budget variance was
a. $4,500 favorable.
b. $8,000 favorable.
c. $4,500 unfavorable.
d. $8,000 unfavorable.
d 33. Acme had a $6,000 favorable fixed overhead budget variance, a $2,500 unfavorable variable
overhead spending variance, and $1,000 total overapplied overhead. The volume variance was
a. $4,500 overapplied.
b. $4,500 underapplied.
c. $2,500 overapplied.
d. $2,500 underapplied.
b 34. Waldorf had a $10,000 unfavorable fixed overhead budget variance, a $6,000 unfavorable
variable overhead spending variance, and a $2,000 favorable volume variance. The total overhead
was
a. $14,000 overapplied.
b. $14,000 underapplied.
c. $18,000 overapplied.
d. $18,000 underapplied.
a 35. Bacon had a $18,000 unfavorable volume variance, a $5,000 unfavorable fixed overhead
budget variance, and $12,000 total underapplied overhead. The variable overhead spending variance
was
a. $11,000 favorable.
b. $1,000 favorable.
c. $11,000 unfavorable.
d. $23,000 unfavorable.
d 36. Gonzalez Company uses the equation $520,000 + $2 per direct labor hour to budget
manufacturing overhead. Gonzalez has budgeted 150,000 direct labor hours for the year. Actual
results were 150,000 direct labor hours and $817,500 total manufacturing overhead. The total
overhead applied for the year is
a. $300,000.
b. $520,000.
c. $817,500.
d. $820,000.
a 37. Gonzalez Company uses the equation $520,000 + $2 per direct labor hour to budget
manufacturing overhead. Gonzalez has budgeted 150,000 direct labor hours for the year. Actual
results were 150,000 direct labor hours and $817,500 total manufacturing overhead. The total
overhead variance for the year is
a. $2,500 favorable.
b. $12,500 favorable.
c. $2,500 unfavorable.
d. some other number.
c 38. Bonds Company uses the equation $300,000 + $1.75 per direct labor hour to budget
manufacturing overhead. Bonds has budgeted 125,000 direct labor hours for the year. Actual results
were 110,000 direct labor hours, $297,000 fixed overhead, and $194,500 variable overhead. The total
overhead variance for the year is
a. $2,000.
b. $3,000.
c. $47,000.
d. $48,000.
a 39. Bonds Company uses the equation $300,000 + $1.75 per direct labor hour to budget
manufacturing overhead. Bonds has budgeted 125,000 direct labor hours for the year. Actual results
were 110,000 direct labor hours, $297,000 fixed overhead, and $194,500 variable overhead. The
114

variable overhead spending variance for the year is


a. $2,000.
b. $3,000.
c. $47,000.
d. $48,000.
b 40. Bonds Company uses the equation $300,000 + $1.75 per direct labor hour to budget
manufacturing overhead. Bonds has budgeted 125,000 direct labor hours for the year. Actual results
were 110,000 direct labor hours, $297,000 fixed overhead, and $194,500 variable overhead. The
fixed overhead budget variance for the year is
a. $2,000.
b. $3,000.
c. $47,000.
d. $48,000.
d 41. Bonds Company uses the equation $300,000 + $1.75 per direct labor hour to budget
manufacturing overhead. Bonds has budgeted 125,000 direct labor hours for the year. Actual results
were 110,000 direct labor hours, $297,000 fixed overhead, and $194,500 variable overhead. The
fixed overhead volume variance for the year is
a. $2,000.
b. $3,000.
c. $47,000.
d. $48,000.
a 42. Machine hours used to set the predetermined overhead rate were 80,000, actual hours were
90,000, and overhead applied was $117,000. Budgeted overhead for the year was
a. $104,000.
b. $117,000.
c. $131,625.
d. some other number.
a 43. Cooke Company uses the equation $450,000 + $1.50 per direct labor hour to budget
manufacturing overhead. Cooke has budgeted 150,000 direct labor hours for the year. Actual results
were 156,000 direct labor hours and $697,500 total manufacturing overhead. The total overhead
variance for the year is
a. $4,500 favorable.
b. $18,000 favorable.
c. $4,500 unfavorable.
d. $18,000 unfavorable.
b 44. Antaya Company uses the equation $375,000 + $1.20 per direct labor hour to budget
manufacturing overhead. Antaya has budgeted 75,000 direct labor hours for the year. Actual results
were 81,000 direct labor hours, $388,000 fixed overhead, and $98,600 variable overhead. The total
overhead variance for the year is
a. $2,700.
b. $10,700.
c. $22,000.
d. $30,000.
a 45. Antaya Company uses the equation $375,000 + $1.20 per direct labor hour to budget
manufacturing overhead. Antaya has budgeted 75,000 direct labor hours for the year. Actual results
were 81,000 direct labor hours, $388,000 fixed overhead, and $98,600 variable overhead. The
variable overhead spending variance for the year is
a. $2,700.
b. $10,700.
c. $22,000.
d. $30,000.
c 46. Antaya Company uses the equation $375,000 + $1.20 per direct labor hour to budget
manufacturing overhead. Antaya has budgeted 75,000 direct labor hours for the year. Actual results
were 81,000 direct labor hours, $388,000 fixed overhead, and $98,600 variable overhead. The fixed
overhead budget variance for the year is
a. $2,700.
b. $10,700.
c. $22,000.
d. $30,000.
d 47. Antaya Company uses the equation $375,000 + $1.20 per direct labor hour to budget
manufacturing overhead. Antaya has budgeted 75,000 direct labor hours for the year. Actual results
115

were 81,000 direct labor hours, $388,000 fixed overhead, and $98,600 variable overhead. The fixed
overhead volume variance for the year is
a. $1,400.
b. $13,000.
c. $15,600.
d. $30,000.
a 48. Machine hours used to set the predetermined overhead rate were 68,000, actual hours were
64,000, and budgeted overhead was $142,800. Overhead applied for the year was
a. $134,400.
b. $136,500.
c. $142,800.
d. $151,725.
a 49. Rhoda had a $2,000 favorable volume variance, a $7,000 unfavorable variable overhead
spending variance, and $3,000 total underapplied overhead. The fixed overhead budget variance was
a. $1,000 favorable.
b. $8,000 favorable.
c. $2,000 unfavorable.
d. $8,000 unfavorable.
b 50. Katrina Inc. had a $30,000 favorable fixed overhead budget variance, a $44,000 unfavorable
variable overhead spending variance, and $44,000 total underapplied overhead. The volume variance
was
a. $30,000 overapplied.
b. $30,000 underapplied.
c. $58,000 overapplied.
d. $58,000 underapplied.
True-False
T 1. A major advantage of normal costing over actual costing is that it smoothes out fluctuations in
unit costs.
F 2. Normal costing incomes are less than actual costing incomes.
T 3. The term overapplied overhead is not used with actual costing.
T 4. A seasonal business using normal costing expects high overapplied and underapplied overhead
during individual months of the year.
F 5. Underapplied overhead indicates inefficient operations.
F 6. Activity-based overhead rates give higher costs than does a single, plant-wide rate.
F 7. A cost pool consists of all of the costs of a particular department.
T 8. Absorption costing is required by GAAP for external financial reporting.
F 9. Variable costing and full costing are the same thing.
T 10. Overhead variances do not exist when actual costing is used.
Problems
1.

Gagne Company uses the following equation to budget manufacturing overhead.


Manufacturing overhead = $600,000 + $2 per direct labor hour

Gagne has budgeted 300,000 direct labor hours for the year. Actual results were 320,000 direct
labor hours and $1,249,000 total manufacturing overhead.
a. Find the predetermined overhead rate.
b.Find total overhead applied for the year.
c. Compute (overapplied underapplied) overhead and circle the correct direction.
d.Find the overhead budget variance and state whether favorable or unfavorable.
116

e. Compute the volume variance and state whether favorable or unfavorable


SOLUTION:
a. $4 ($600,000/300,000 + $2 variable)
b. $1,280,000 ($4 x 320,000)
c. $31,000 overapplied ($1,280,000 - $1,249,000)
d. $9,000 unfavorable {$1,249,000 - [($2 x 320,000) + $600,000]}
e. $40,000 favorable ($1,240,000 budgeted - $1,280,000 applied)
2. Gomez Company uses job-order costing. Data are as follows:
Jobs in
Jobs in Ending
Totals Jobs Sold Ending WIP Finished Goods
---------- ---------- ---------- -------------Materials $1,600,000 $1,450,000 $ 50,000
$100,000
Direct labor $1,250,000 $ 900,000 $100,000
$250,000
Gomez applies overhead to jobs at $0.80 per direct labor dollar. Total overhead cost incurred was
$950,000. There were no beginning inventories.
a. What is cost of goods sold using normal costing?
b.Find the ending inventory of work in process using normal costing.
c. Find the amount of overhead (overapplied underapplied) and circle the correct direction.
d.Gomez treats overapplied or underapplied overhead as an adjustment to cost of goods sold.
Compute total cost of goods sold.
SOLUTION:
a. $3,070,000 [$1,450,000 + $900,000 + ($0.80 x $900,000)]
b. $230,000 [$50,000 + $100,000 + ($0.80 x $100,000)]
c. $50,000 overapplied [($1,250,000 x $0.80) - $950,000]
d. $3,020,000 ($3,070,000 normal minus $50,000 overapplied OH)
3.

Antigo Company uses job-order costing. Data related to March are as follows:
Job A
Job B
Job C
---------------Material cost
$3,900
$4,700
$5,200
Direct labor cost $4,000
$6,000
$4,000
Machine hours
300
500
300

Antigo applies overhead to jobs at $8 per machine hour. Total overhead cost incurred in March
was $8,400. There were no beginning inventories. Job A was incomplete at the end of March, Job B
was sold for $28,000, and Job C was in finished goods inventory. Selling and administrative expenses
were $2,100.
a. Compute the ending inventory of work in process.
b.Compute the ending inventory of finished goods.
c. Overhead was (overapplied underapplied) by? Circle the correct direction.
d.Compute the normal cost of goods sold.
e. Prepare an income statement showing underapplied or overapplied overhead as an adjustment to
normal cost of sales.
SOLUTION:
117

a.

$10,300 (Cost of job A, below)

b.

$11,600 (Cost of job C, below)


Job A
Job B
Job C
----------------Material cost
$ 3,900
$ 4,700
$ 5,200
Direct labor cost
4,000
6,000
4,000
Applied overhead at $8
2,400
4,000
2,400
------------------Total costs
$10,300
$14,700
$11,600
=======
=======
=======

c.

$400 overapplied

Actual overhead
$8,400
Applied overhead ($2,400 + $4,000 + $2,400)
-----Overapplied overhead
$ 400
======
d. $14,700 (Cost of job B, see above)

8,600

e.

4.

Sales
$28,000
Normal cost of sales
$14,700
Less overapplied overhead
400
14,300
------------Gross margin
$13,700
Selling and administrative expenses
2,100
------Income
$11,600
=======
Bruno Company uses the following equation to budget manufacturing overhead.
Manufacturing overhead = $200,000 + $3 per direct labor hour

Bruno has budgeted 100,000 direct labor hours for the year. Actual results were 90,000 direct
labor hours and $457,000 total manufacturing overhead.
a. Find the predetermined overhead rate.
b.Find total overhead applied for the year.
c. Compute (overapplied underapplied) overhead and circle the correct direction.
d.Find the overhead budget variance and state whether favorable or unfavorable.
e. Compute the volume variance and state whether favorable or unfavorable
SOLUTION:
a. $5 ($200,000/100,000 + $3 variable)
b. $450,000 ($5 x 90,000)
c. $7,000 underapplied ($450,000 - $457,000)
d. $13,000 favorable {$457,000 - [($3 x 90,000) + $200,000]}
e. $20,000 unfavorable ($470,000 budgeted - $450,000 applied)
5. Hurley Company uses job-order costing. Data are as follows:
Jobs in
Jobs in Ending
Totals
Jobs Sold Ending WIP Finished Goods
---------- ---------- ---------- -------------Materials $2,050,000 $1,500,000 $200,000
$350,000
Direct labor $2,250,000 $1,650,000 $160,000
$440,000
Hurley applies overhead to jobs at $0.60 per direct labor dollar. Total overhead cost incurred was
$1,460,000. There were no beginning inventories.
118

a. What is cost of goods sold using normal costing?


b.Find the ending inventory of work in process using normal costing.
c. Find the amount of overhead (overapplied underapplied) and circle the correct direction.
d.Hurley treats overapplied or underapplied overhead as an adjustment to cost of goods sold.
Compute total cost of goods sold.
SOLUTION:
a. $4,140,000 [$1,500,000 + $1,650,000 + ($0.60 x $1,650,000)]
b. $456,000 [$200,000 + $160,000 + ($0.60 x $160,000)]
c. $110,000 underapplied [($2,250,000 x $0.60) - $1,460,000]
d. $4,250,000 ($4,140,000 normal plus $110,000 underapplied OH)
6.

Acme Company uses job-order costing. Data related to August are as follows:
Job A
Job B
Job C
---------------Material cost
$3,900
$4,700
$5,400
Direct labor cost $4,000
$7,000
$4,000
Machine hours
400
700
500

Acme applies overhead to jobs at $10.00 per machine hour. Total overhead cost incurred in
August was $16,700. There were no beginning inventories. Job A was incomplete at the end of
August, Job B was sold for $34,000, and Job C was in finished goods inventory. Selling and
administrative expenses were $3,500.
a. Compute the ending inventory of work in process.
b.Compute the ending inventory of finished goods.
c. Overhead was (overapplied underapplied) by? Circle the correct direction.
d.Compute the normal cost of goods sold.
e. Prepare an income statement showing underapplied or overapplied overhead as an adjustment to
normal cost of sales.
SOLUTION:
a.

$11,900 (Cost of job A, below)

b.

$14,400 (Cost of job C, below)


Job A
Job B
Job C
----------------Material cost
$ 3,900
$ 4,700
$ 5,400
Direct labor cost
4,000
7,000
4,000
Applied overhead at $10 4,000
7,000
4,000
-----------------Total costs
$11,900
$18,700
$14,400
=======
=======
=======

c.

$700 underapplied
Actual overhead
$16,700
Applied overhead ($4,000 + $7,000 + $4,000)
------Underapplied overhead
$ 700
=======

d.

16,000

$18,700 (Cost of job B, see above)

e.
119

Sales
$34,000
Normal cost of sales
$18,700
Plus underapplied overhead
700
19,400
------------Gross margin
$14,600
Selling and administrative expenses
3,500
------Income
$11,100
=======
7. Darlington Company uses job-order costing. Data are as follows:
Jobs in
Jobs in Ending
Totals
Jobs Sold Ending WIP Finished Goods
---------- ---------- ---------- -------------Materials $3,200,000 $2,700,000 $200,000
$300,000
Direct labor $1,500,000 $1,000,000 $100,000
$400,000
Darlington uses actual costing to assign overhead based on direct labor. Total overhead cost
incurred was $2,700,000. There were no beginning inventories.
a. What is cost of goods sold?
b.Find the ending inventory of work in process.
c. Find the ending finished goods inventory value.
SOLUTION:
a. $5,500,000 [$2,700,000 + $1,000,000 + ($1,000,000/$1,500,000 x
$2,700,000)]
b. $480,000 [$200,000 + $100,000 + ($100,000/$1,500,000 x $2,700,000)]
c. $1,420,000 [$300,000 + $400,000 + ($400,000/$1,500,000 x $2,700,000)]
8.

Beloit Company uses job-order costing. Data related to September are as follows:
Job A
Job B
Job C
---------------Material cost
$4,400
$3,400
$4,200
Direct labor cost $3,000
$6,000
$2,000

Beloit uses actual costing to apply overhead to jobs based on direct labor cost. Total overhead
cost incurred in September was $14,300. There were no beginning inventories. Job A was incomplete
at the end of September, Job B was sold for $22,000, and Job C was in finished goods inventory.
Selling and administrative expenses were $3,800.
a. Compute the ending inventory of work in process.
b.Compute the ending inventory of finished goods.
c. Overhead was (overapplied underapplied) by? Circle the correct direction.
d.Compute cost of goods sold.
e. Prepare an income statement showing underapplied or overapplied overhead as an adjustment to
cost of sales.
SOLUTION:
a.

$11,300 (Cost of job A, below)

b.

$8,800 (Cost of job C, below)


Job A
Job B
Job C
----------------Material cost
$4,400
$ 3,400
$4,200
Direct labor cost
3,000
6,000
2,000
Applied overhead
3,900
7,800
2,600
120

Total costs
c.

-----------------$11,300
$17,200
$8,800
=======
=======
======

neither; actual overhead is applied


Actual overhead
$14,300
Applied overhead ($3,900 + $7,800 + $2,600)
-----Underapplied overhead
$
0
=======

d.
e.

14,300

$17,200 (Cost of job B, see above)


Sales
Cost of sales

$22,000
17,200
------Gross margin
$ 4,800
Selling and administrative expenses
3,800
------Income
$ 1,000
=======

9. Ashland Company uses job-order costing. Data are as follows:


Jobs in
Jobs in Ending
Totals
Jobs Sold Ending WIP Finished Goods
---------- ---------- ---------- -------------Materials $1,450,000 $800,000 $300,000
$350,000
Direct labor $1,650,000 $850,000 $260,000
$540,000
Ashland applies overhead to jobs at $0.70 per direct labor dollar. Total overhead cost incurred was
$1,070,000. There were no beginning inventories.
a. What is cost of goods sold using normal costing?
b.Find the ending inventory of work in process using normal costing.
c. Find the amount of overhead (overapplied underapplied) and circle the correct direction.
d.Ashland treats overapplied or underapplied overhead as an adjustment to cost of goods sold.
Compute total cost of goods sold.
SOLUTION:
a. $2,245,000 [$800,000 + $850,000 + ($0.70 x $850,000)]
b. $742,000 [$300,000 + $260,000 + ($0.70 x $260,000)]
c. $85,000 overapplied [($1,650,000 x $0.70) - $1,070,000]
d. $2,160,000 ($2,245,000 normal less $85,000 overapplied OH)
10. Hayes Company uses job-order costing. Data related to May are as follows:
Job A
Job B
Job C
---------------Material cost
$3,900
$5,700
$4,400
Direct labor cost $2,000
$4,000
$3,000
Machine hours
1,000
700
1,400
Hayes applies overhead to jobs at $8 per machine hour. Total overhead cost incurred in May was
$24,650. There were no beginning inventories. Job A was incomplete at the end of May, Job B was
sold for $30,000, and Job C was in finished goods inventory. Selling and administrative expenses were
$3,900.
a. Compute the ending inventory of work in process.
b.Compute the ending inventory of finished goods.
121

c. Overhead was (overapplied underapplied) by? Circle the correct direction.


d.Compute the normal cost of goods sold.
e. Prepare an income statement showing underapplied or overapplied overhead as an adjustment to
normal cost of sales.
SOLUTION:
a.

$13,900 (Cost of job A, below)

b.

$18,600 (Cost of job C, below)


Job A
Job B
Job C
----------------Material cost
$3,900
$ 5,700
$4,400
Direct labor cost
2,000
4,000
3,000
Applied overhead at $8
8,000
5,600
11,200
-----------------Total costs
$13,900
$15,300
$18,600
=======
=======
=======

c.

$150 overapplied
Actual overhead
$24,650
Applied overhead ($8,000 + $5,600 + $11,200)
------Overapplied overhead
$ 150
=======

d.

24,800

$15,300 (Cost of job B, see above)

e.
Sales
$30,000
Normal cost of sales
$15,300
Less overapplied overhead
150
15,150
------------Gross margin
$14,850
Selling and administrative expenses
3,900
------Income
$10,950
=======
CHAPTER 13:
STANDARD COSTING, VARIABLE COSTING, AND THROUGHPUT
COSTING
Multiple Choice
a 1. Which of the following is NOT a type of absorption costing?
a. Direct costing.
b. Actual costing.
c. Normal costing.
d. None of the above.
b 2. Variable costing is UNACCEPTABLE for
a. managerial accounting.
b. financial accounting.
c. transfer pricing.
d. reporting by product lines for internal purposes.
d 3. A criticism of variable costing for managerial accounting purposes is that it
a. is not acceptable for product line segmented reporting.
b. does not reflect cost-volume-profit relationships.
c. overstates inventories.
d. might encourage managers to emphasize the short term at the expense of the long term.
c 4. Normal costing and standard costing differ in that
a. the two systems can show different overhead budget variances.
b. only normal costing can be used with absorption costing.
c. the two systems show different volume variances if standard hours do not equal actual hours.
122

d. normal costing is less appropriate for multiproduct firms.


d 5. Variable costing and absorption costing will show the same incomes when there are no
a. beginning inventories.
b. ending inventories.
c. variable costs.
d. beginning and ending inventories.
c 6. ABC had the same activity in 20X3 as in 20X2 except that production was higher in 20X3 than in
20X2. ABC will show
a. higher income in 20X3 than in 20X2.
b. the same income in both years.
c. the same income in both years under variable costing.
d. the same income in both years under absorption costing.
b 7. The use of variable costing requires knowing
a. the contribution margin and break-even point for each product.
b. the variable and fixed components of production cost.
c. controllable and noncontrollable components of all costs.
d. the number of units of each product produced during the period.
d 8. Which measure of activity is likely to give the LOWEST standard fixed cost per unit?
a. Actual activity.
b. Normal capacity.
c. Budgeted activity.
d. Practical capacity.
c 9. Which item is NOT used to compute the fixed overhead volume variance?
a. Standard fixed cost per unit.
b. Budgeted fixed overhead.
c. Actual fixed overhead.
d. Actual quantity produced.
b 10. Which variance is LEAST relevant for control purposes?
a. Material use variance.
b. Fixed overhead volume variance.
c. Fixed overhead budget variance.
d. Labor efficiency variance.
a 11. A company that sets a standard fixed cost based on practical capacity
a. should expect unfavorable volume variances.
b. will set its selling prices too low.
c. has a higher cost per unit than a company using normal activity to set the standard.
d. usually overapplies its fixed costs.
a 12. A predetermined overhead rate for fixed costs is unlike a standard fixed cost per unit in that a
predetermined overhead rate is
a. based on an input factor like direct labor hours and a standard cost per unit is based on a unit
of output.
b. based on practical capacity and a standard fixed cost can be based on any level of activity.
c. used with variable costing while a standard fixed cost is used with absorption costing.
d. likely to be higher than a standard fixed cost per unit.
b 13. ABC had $400,000 budgeted fixed overhead costs and based its standard on normal activity of
40,000 units. Actual fixed overhead costs were $430,000, actual production was 36,000 units, and
sales were 30,000 units. The volume variance was
a. $30,000.
b. $40,000.
c. $70,000.
d. $77,777.
a 14. Advocates of variable costing for internal reporting purposes do NOT rely on which of the
following points?
a. The matching concept.
b. Price-volume relationships.
c. Absorption costing does not include selling and administrative expenses as part of
inventoriable cost.
d. Production influences income under absorption costing.
123

d 15. Calculating income under variable costing does NOT require knowing
a. unit sales.
b. unit variable manufacturing costs.
c. selling price.
d. unit production.
a 16. Inventoriable costs under absorption costing include
a. both fixed and variable production costs.
b. only variable production costs.
c. all production costs plus variable selling and administrative costs.
d. all production costs plus all selling and administrative costs.
b 17. Inventoriable costs under variable costing include
a. fixed and variable production costs.
b. variable production costs.
c. all production costs plus variable selling and administrative costs.
d. all production costs plus all selling and administrative costs.
d 18. Absorption costing and variable costing differ in that
a. income is lower under variable costing.
b. variable costing treats selling costs as period costs.
c. variable costing treats all variable costs as product costs.
d. inventory cost is higher under absorption costing.
c 19. Absorption costing differs from variable costing in that
a. standards can be used with absorption costing, but not with variable costing.
b. absorption costing inventories are more correctly valued.
c. production influences income under absorption costing, but not under variable costing.
d. companies using absorption costing have lower fixed costs.
a 20. Which method gives the lowest inventory cost per unit?
a. Variable costing.
b. Absorption costing using normal activity to set the standard fixed cost.
c. Absorption costing using practical capacity to set the standard fixed cost.
d. Actual absorption costing.
b 21. Which costs are treated differently under absorption costing and variable costing?
a. Variable manufacturing costs.
b. Fixed manufacturing costs.
c. Variable selling and administrative expenses.
d. Fixed selling and administrative expenses.
a 22. ABC Company had 15,000 units in ending inventory. The total cost of those units under variable
costing is
a. less than it is under absorption costing.
b. the same as it is under absorption costing.
c. more than it is under absorption costing.
d. any of the above.
b 23. York Company had $200,000 income using absorption costing. York has no variable
manufacturing costs. Beginning inventory was $15,000 and ending inventory was $22,000. Income
under variable costing would have been
a. $178,000.
b. $193,000.
c. $200,000.
d. $207,000.
c 24. An unfavorable volume variance means that
a. cost control was probably poor.
b. absorption costing income is lower than variable costing income.
c. actual output was less than the level used to set the standard fixed cost.
d. actual output was more than the level used to set the standard fixed cost.
d 25. Which variance CANNOT arise under variable costing?
a. variable overhead budget variance.
b. variable overhead efficiency variance.
c. fixed overhead budget variance.
d. fixed overhead volume variance.
124

a 26. Standard costing differs from normal costing in the treatment of


a. materials, direct labor, and overhead.
b. materials and direct labor.
c. direct labor and overhead.
d. overhead.
d 27. Normal costing differs from actual costing in treating
a. materials, direct labor, and overhead.
b. materials and direct labor.
c. direct labor and overhead.
d. overhead.
c 28. As compared to normal costing, standard costing can yield
a. different volume variances and budget variances.
b. different budget variances.
c. different volume variances.
d. none of the above.
c 29. Under variable costing there can be no
a. fixed overhead variances.
b. fixed overhead budget variance.
c. fixed overhead volume variance.
d. no fixed overhead.
c 30. ABC had the same activity in 20X4 as in 20X3 except that production was lower in 20X4 than in
20X3. ABC will show
a. lower income in 20X4 than in 20X3.
b. the same income in both years.
c. the same income in both years under variable costing.
d. the same income in both years under absorption costing.
a 31. Rounder Industries manufactures a single product. Variable production costs are $20 and fixed
production costs are $300,000. Rounder uses a normal activity of 20,000 units to set its standard
costs. Rounder began the year with no inventory, produced 22,000 units, and sold 21,000 units.
Ending inventory under variable costing would be
a. $20,000.
b. $30,000.
c. $35,000.
d. cannot be determined without further information.
c 32. Rounder Industries manufactures a single product. Variable production costs are $20 and fixed
production costs are $300,000. Rounder uses a normal activity of 20,000 units to set its standard
costs. Rounder began the year with no inventory, produced 22,000 units, and sold 21,000 units.
Ending inventory under absorption costing would be
a. $20,000.
b. $30,000.
c. $35,000.
d. cannot be determined without further information.
a 33. Rounder Industries manufactures a single product. Variable production costs are $20 and fixed
production costs are $300,000. Rounder uses a normal activity of 20,000 units to set its standard
costs. Rounder began the year with no inventory, produced 22,000 units, and sold 21,000 units. The
volume variance under variable costing would be
a. $0.
b. $20,000.
c. $30,000.
d. some other number.
c 34. Rounder Industries manufactures a single product. Variable production costs are $20 and fixed
production costs are $300,000. Rounder uses a normal activity of 20,000 units to set its standard
costs. Rounder began the year with no inventory, produced 22,000 units, and sold 21,000 units. The
volume variance under absorption costing would be
a. $0.
b. $20,000.
c. $30,000.
d. some other number.
b 35. Rounder Industries manufactures a single product. Variable production costs are $20 and fixed
production costs are $300,000. Rounder uses a normal activity of 20,000 units to set its standard
125

costs. Rounder began the year with no inventory, produced 22,000 units, and sold 21,000 units. The
standard cost of goods sold under variable costing would be
a. $400,000.
b. $420,000.
c. $735,000.
d. some other number.
c 36. Rounder Industries manufactures a single product. Variable production costs are $20 and fixed
production costs are $300,000. Rounder uses a normal activity of 20,000 units to set its standard
costs. Rounder began the year with no inventory, produced 22,000 units, and sold 21,000 units. The
standard cost of goods sold under absorption costing would be
a. $400,000.
b. $420,000.
c. $735,000.
d. some other number.
c 37. Alpha Company has a standard fixed cost of $10 per unit. At an actual production of 16,000
units an unfavorable volume variance of $20,000 resulted. What were total budgeted fixed costs?
a. $140,000
b. $160,000
c. $180,000
d. Cannot be determined without further information.
a 38. Beta Company has a standard fixed cost of $10 per unit using a normal capacity of 11,000
units. An unfavorable volume variance of $12,000 resulted. What was the volume produced?
a. 9,800
b. 11,000
c. 12,200
d. Cannot be determined without further information.
a 39. Gamma Corporation has total budgeted fixed costs of $150,000. Actual production was 8,000
units; normal capacity is 7,500 units. What was the volume variance?
a. $10,000 favorable
b. $15,000 favorable
c. $15,000 unfavorable
d. $10,000 unfavorable
b 40. Eastern Co. has total budgeted fixed costs of $150,000. Actual production of 39,000 units
resulted in a $6,000 favorable volume variance. What normal capacity was used to determine the
fixed overhead rate?
a. 33,000
b. 37,500
c. 40,560
d. Cannot be determined without further information.
a 41. Western Company has a standard fixed cost of $8 per unit. At an actual production of 8,000
units a favorable volume variance of $12,000 resulted. What were total budgeted fixed costs?
a. $52,000
b. $64,000
c. $76,000
d. Cannot be determined without further information.
d 42. Monona Corporation has total budgeted fixed costs of $64,000. Actual production was 15,000
units; normal capacity is 16,000 units. What was the volume variance?
a. $4,000 favorable
b. $4,267 favorable
c. $4,267 unfavorable
d. $4,000 unfavorable
b 43. Madison Industries manufactures a single product using standard costing. Variable production
costs are $26 and fixed production costs are $250,000. Madison uses a normal activity of 12,500 units
to set its standard costs. Madison began the year with 1,000 units in inventory, produced 11,000
units, and sold 11,500 units. Ending inventory under variable costing would be
a. $10,000.
b. $13,000.
c. $23,000.
d. cannot be determined without further information.
c 44. Madison Industries manufactures a single product using standard costing. Variable production
126

costs are $26 and fixed production costs are $250,000. Madison uses a normal activity of 12,500 units
to set its standard costs. Madison began the year with 1,000 units in inventory, produced 11,000
units, and sold 11,500 units. Ending inventory under absorption costing would be
a. $10,000.
b. $13,000.
c. $23,000.
d. cannot be determined without further information.
d 45. Madison Industries manufactures a single product using standard costing. Variable production
costs are $26 and fixed production costs are $250,000. Madison uses a normal activity of 12,500 units
to set its standard costs. Madison began the year with 1,000 units in inventory, produced 11,000
units, and sold 11,500 units. The volume variance under variable costing would be
a. $10,000.
b. $20,000.
c. $30,000.
d. some other number.
c 46. Madison Industries manufactures a single product using standard costing. Variable production
costs are $26 and fixed production costs are $250,000. Madison uses a normal activity of 12,500 units
to set its standard costs. Madison began the year with 1,000 units in inventory, produced 11,000
units, and sold 11,500 units. The volume variance under absorption costing would be
a. $10,000.
b. $20,000.
c. $30,000.
d. some other number.
b 47. Madison Industries manufactures a single product using standard costing. Variable production
costs are $26 and fixed production costs are $250,000. Madison uses a normal activity of 12,500 units
to set its standard costs. Madison began the year with 1,000 units in inventory, produced 11,000
units, and sold 11,500 units. The standard cost of goods sold under variable costing would be
a. $230,000.
b. $299,000.
c. $506,000.
d. $529,000.
c 48.
Sigma Company has a standard fixed cost of $18 per unit using a normal capacity of 9,000
units. A favorable volume variance of $18,000 resulted. What was the volume produced?
a. 8,000
b. 9,000
c. 10,000
d. Cannot be determined without further information.
c 49.
Western Co. has total budgeted fixed costs of $72,000. Actual production of 5,500 units
resulted in a $6,000 unfavorable volume variance. What normal capacity was used to determine the
fixed overhead rate?
a. 5,000
b. 5,500
c. 6,000
d. Cannot be determined without further information.
d 50. Madison Industries manufactures a single product using standard costing. Variable production
costs are $26 and fixed production costs are $250,000. Madison uses a normal activity of 12,500 units
to set its standard costs. Madison began the year with 1,000 units in inventory, produced 11,000
units, and sold 11,500 units. The standard cost of goods sold under absorption costing would be
a. $230,000.
b. $299,000.
c. $506,000.
d. $529,000.
True-False
F 1. Absorption costing incomes are always higher than variable costing incomes.
F 2. Income under standard variable costing is not influenced by the total amount of fixed
manufacturing costs.
T 3. A multiproduct company using standard absorption costing calculates standard fixed costs for
each product using a standard fixed overhead rate based on an input factor such as direct labor
hours.
127

T 4. A major difference between standard costing and normal costing is that one uses actual hours to
apply overhead and the other uses standard hours.
T 5. Proponents of variable costing for external reporting argue that while fixed production costs
benefit production as a whole, they do not benefit any particular unit of product.
T 6. A company using absorption costing can increase its income by increasing production without
increasing sales.
F 7. A company using variable costing can increase its income by increasing production without
increasing sales.
F 8. Variable costing must be used for internal reporting.
F 9. According to GAAP, absorption costing must be used for internal reporting.
T 10. According to GAAP, absorption costing must be used for external financial reporting.
Problems
1. Whitehall Company sells a single product for $25. It had no beginning inventories. Operating data
follow.
Sales, 27,000 units
Normal capacity
Production costs:
Variable per unit
Fixed production
Selling and administrative expenses:
Variable per unit sold
Fixed selling
Number of units produced

$675,000
30,000 units
$13
$150,000
$2
$20,000
32,500 units

Assume the actual costs were as budgeted.


a. Find contribution margin per unit.
b.Compute the ending inventory under standard variable costing.
c. Compute the income under standard variable costing.
Assume standard absorption costing using normal capacity as the basis for computing the
standard fixed cost per unit. Compute
d.Standard gross profit per unit.
e. Ending inventory.
f. Volume variance.
g.Income.
SOLUTION:
a.

$10 ($25 - $13 -$2)

b.

$71,500 [$13 x ending inventory of 5,500 units (32,500 - 27,000)]

c.

$100,000 [(27,000 x $10) - ($150,000 + $20,000)]

d.

$7 [$25 - $13- ($150,000/30,000)]

e.

$99,000 [5,500 x ($13 + $5)]

f.

$12,500 F [(32,500 - 30,000) x $5 standard fixed cost per unit]

g.

$127,500 [(27,000 x $7) + $12,500 - (27,000 x $2) - $20,000]

2. Lund Company sells a single product for $25. It had no beginning inventories. Operating data
128

follow.
Sales, 55,000 units
Normal capacity
Production costs:
Variable per unit
Fixed production
Selling and administrative expenses:
Variable per unit sold
Fixed selling
Number of units produced

$1,375,000
60,000 units
$13
$300,000
$2
$40,000
66,000 units

Assume the actual costs were as budgeted.


a. Compute income under standard variable costing.
b. Compute income under standard absorption costing.
SOLUTION:
a.

$210,000 [55,000 x ($25 - 13 - 2) - ($300,000 + $40,000)]

$265,000 {$1,375,000 - 55,000 x [$13 + ($300,000/60,000)] - 55,000 x $2


- $40,000 + $30,000 volume variance}
3. Maiden Rock Company sells a single product for $25. It had no beginning inventories. Operating
data follow.
Sales, 20,000 units
Normal capacity
Production costs:
Variable per unit
Fixed production
Selling and administrative expenses:
Variable per unit sold
Fixed selling
Number of units produced

$500,000
30,000 units
$13
$150,000
$2
$20,000
32,500 units

Assume the actual costs were as budgeted.


a. Find Maiden Rocks income under standard variable costing.
b.Find Maiden Rocks income under standard absorption costing.
SOLUTION:
a.
b.

$30,000 [20,000 x ($25 - 13 - 2) - ($150,000 + $20,000)]


$92,500 {$500,000 - 20,000 x [$13 + [$150,000/30,000)] - 20,000 x $2
- $20,000 + $12,500 volume variance}

4. Genco Inc. makes a single product that sells for $50. The standard variable manufacturing cost is
$32.50 and the standard fixed manufacturing cost is $7.50, based on producing 20,000 units. During
the year Genco produced 22,000 units and sold 21,000 units. Actual fixed manufacturing costs were
$157,000; actual variable manufacturing costs were $735,000. Selling and administrative expenses,
all fixed, were $75,000. There were no beginning inventories.
a. Prepare a standard absorption costing income statement.
b.Prepare a standard variable costing income statement.
SOLUTION:
a.

Sales (21,000 x $50)


$1,050,000
Cost of Goods Sold (21,000 x $40) $840,000
Variances:
Variable Spending $20,000 Un
Fixed Spending
7,000 Un
129

Volume
(15,000) F 12,000
Adjusted Cost of Goods Sold
852,000
Gross Profit
$198,000
Selling & Administrative
75,000
Net Income
$123,000
b.

Sales
$1,050,000
Variable Costs (21,000 x $32.50) $682,500
Variable Spending Variance
20,000 Un
Adjusted Variable Cost of Goods Sold
702,500
Contribution Margin
$347,500
Fixed Costs:
Manufacturing
$157,000
Selling & Administrative
75,000
232,000
Net Income
$115,500

5.

Brahms Corp. has the following data:


Normal capacity
25,000
Practical capacity
30,000
Budgeted production
20,000
Actual production
22,000
Actual sales ($25 per unit)
21,000
Standard variable production cost per unit
$15
Budgeted fixed production costs
$120,000

There were no variable cost variances for the year. Fixed costs incurred were equal to the
budgeted amount. There were no beginning inventories and no selling or administrative expenses.
a. Compute the absorption costing income if fixed costs per unit are determined using normal
capacity.
b.Compute the absorption costing income if fixed costs per unit are determined using practical
capacity.
c. Compute the absorption costing income if fixed costs per unit are determined using budgeted
production.
d.Compute the variable costing income.
SOLUTION:
a.

$94,800

[$525,000 - (21,000 x $19.80) - $14,400 volume variance]

Volume variance $14,400 = $120,000/25,000 x 22,000 - $120,000


b.

$94,000

[$525,000 - (21,000 x $19) - $32,000 volume variance]

Volume variance $32,000 = $120,000/30,000 x 22,000 - $120,000


c.

$96,000

[$525,000 - (21,000 x $21) + $12,000 volume variance]

Volume variance $12,000 = $120,000/20,000 x 22,000 - $120,000


d.

$90,000

[$525,000 - (21,000 x $15) - $120,000]

6. Cumberland Company sells a single product for $30. It had no beginning inventories. Operating
data follow.
Sales, 12,000 units
Normal capacity
Production costs:
Variable per unit
Fixed production
Selling and administrative expenses:
Variable per unit sold
Fixed selling
Number of units produced

$360,000
20,000 units
$15
$75,000
$5
$25,000
13,000 units
130

Assume the actual costs were as budgeted.


a. Find contribution margin per unit.
b.Compute the ending inventory under standard variable costing.
c. Compute the income under standard variable costing.
Assume standard absorption costing using normal capacity as the basis for computing the standard
fixed cost per unit. Compute
d.Standard gross profit per unit.
e. Ending inventory.
f. Volume variance.
g.Income.
SOLUTION:
a.

$10 ($30 - $15 - $5)

b.

$15,000 [$15 x ending inventory of 1,000 units (13,000 - 12,000)]

c.

$20,000 [(12,000 x $10) - ($75,000 + $25,000)]

d.

$11.25 [$30 - $15 - ($75,000/20,000)]

e.

$18,750 [1,000 x ($15 + $3.75)]

f.

$26,250 U [(20,000 - 13,000) x $3.75 standard fixed cost per unit]

g.

$23,750 [(12,000 x $11.25) - $26,250 - (12,000 x $5) - $25,000]

7. Acme Company sells a single product for $30. It had no beginning inventories. Operating data
follow.
Sales, 12,000 units
Normal capacity
Production costs:
Variable per unit
Fixed production
Selling and administrative expenses:
Variable per unit sold
Fixed selling
Number of units produced

$360,000
15,000 units
$17
$75,000
$5
$25,000
13,500 units

Assume the actual costs were as budgeted.


a. Compute income under standard variable costing.
b. Compute income under standard absorption costing.
SOLUTION:
a.

$(4,000) [(12,000 x ($30 - $17 - $5) - ($75,000 + $25,000)]

$3,500 {$360,000 - 12,000 x [$17 + ($75,000/15,000)] - 12,000 x $5


- $25,000 - $7,500 volume variance}
8. Carlson Company sells a single product for $30. It had no beginning inventories. Operating data
follow.
Sales, 12,500 units
Normal capacity
Production costs:
Variable per unit
Fixed production

$375,000
15,000 units
$17
$75,000
131

Selling and administrative expenses:


Variable per unit sold
Fixed selling
Number of units produced

$5
$25,000
13,000 units

Assume the actual costs were as budgeted.


a. Find Carlsons income under standard variable costing.
b.Find Carlsons income under standard absorption costing.
SOLUTION:
a.
b.

$ -0- [12,500 x ($30 - $17 - $5) - ($75,000 + $25,000)]


$2,500 {$375,000 12,500 x [$17 + ($75,000/15,000)] 12,500 x $5
- $25,000 - $10,000 volume variance}

9. Bach Inc. makes a single product that sells for $40. The standard variable manufacturing cost is
$22 and the standard fixed manufacturing cost is $8, based on producing 30,000 units. During the
year Bach produced 28,000 units and sold 26,000 units. Actual fixed manufacturing costs were
$235,000; actual variable manufacturing costs were $595,000. Selling and administrative expenses
were $95,000. There were no beginning inventories.
a. Prepare a standard absorption costing income statement.
b.Prepare a standard variable costing income statement.
SOLUTION:
a.

Sales (26,000 x $40)


$1,040,000
Cost of Goods Sold (26,000 x $30) $780,000
Variances:
Variable Spending $(21,000) F
Fixed Spending
( 5,000) F
Volume
16,000 Un (10,000)
Adjusted Cost of Goods Sold
770,000
Gross Profit
$270,000
Selling & Administrative
95,000
Net Income
$175,000

b.

Sales
$1,040,000
Variable Costs (26,000 x $22)
$572,000
Variable Spending Variance
(21,000) F
Adjusted Variable Cost of Goods Sold
551,000
Contribution Margin
$489,000
Fixed Costs:
Manufacturing
235,000
Selling & Administrative
95,000
330,000
Net Income
$159,000

10. Hayden Corp. has the following data:


Normal capacity
40,000
Practical capacity
45,000
Budgeted production
30,000
Actual production
35,000
Actual sales ($20 per unit)
32,000
Standard variable production cost per unit
$12
Budgeted fixed production costs
$135,000
There were no variable cost variances for the year. Fixed costs incurred were equal to the
budgeted amount. There were no beginning inventories and no selling or administrative expenses.
a. Compute the absorption costing income if fixed costs per unit are determined using normal
capacity.
b.Compute the absorption costing income if fixed costs per unit are determined using practical
capacity.
132

c. Compute the absorption costing income if fixed costs per unit are determined using budgeted
production.
d.Compute the variable costing income.
SOLUTION:
a.

$131,125

[$640,000 - (32,000 x $15.375) - $16,875 volume variance]

Volume variance $16,875 = $135,000/40,000 x 35,000 - $135,000


b.

$130,000

[$640,000 - (32,000 x $15) - $30,000 volume variance]

Volume variance $30,000 = $135,000/30,000 x 35,000 - $135,000


c.

$134,500

[$640,000 - (32,000 x $16.50) + $22,500 volume variance]

Volume variance $22,500 = $135,000/30,000 x 35,000 - $135,000


d.

$121,000

[$640,000 - (32,000 x $12) - $135,000]

CHAPTER 14: PROCESS COSTING AND THE COST ACCOUNTING CYCLE


Multiple Choice
c 1. ABC Company made the following journal entry.
Work in Process Inventory
$200,000
Direct Labor
$188,000
Direct Labor Rate Variance
12,000
From this entry we can tell that ABC uses
a. job-order costing.
b. process costing.
c. standard costing.
d. normal costing.
d 2. CDE Company made the following journal entry.
Finished Goods Inventory
$250,000
Work in Process Inventory
$250,000
From this entry we can tell that CDE uses
a. job-order costing.
b. process costing.
c. standard costing.
d. any of the above.
b 3. Which of the following is NOT relevant in determining weighted-average unit cost in process
costing?
a. Cost of beginning inventory.
b. Equivalent unit production in beginning inventory.
c. Equivalent unit production in ending inventory.
d. Units completed.
d 4. Standard process costing does NOT require information about
a. units completed during the period.
b. equivalent unit production in ending inventory.
c. standard cost per unit.
d. actual unit cost for the period.
d 5. A company that uses job-order costing
a. cannot use standard costs.
b. accumulates costs by department.
c. probably makes a single product.
d. does not have to calculate equivalent production.
c 6. Which company is most likely to use job-order costing?
a. A brewery.
b. An automobile manufacturer.
c. A bridge builder.
d. A button manufacturer.
133

c 7. Which company is most likely to use process costing?


a. A manufacturer of nuclear reactors.
b. A construction contractor.
c. A cannery.
d. A textbook publisher.
a 8. Fixed production costs are inventoriable only if a company
a. uses absorption costing.
b. uses standard costing.
c. produces a single product.
d. receives permission from the Internal Revenue Service.
d 9. Which cost accumulation method is most likely to be used by a company that mass produces
similar products?
a. Actual costing.
b. Normal costing.
c. Job-order costing.
d. Standard costing.
c 10. Standard costing can be used in
a. only job-order costing systems.
b. only process costing systems.
c. either job-order or process systems.
d. either manufacturing or retailing firms.
a 11. Which of the following is the same whether the company uses standard process costing or
actual process costing?
a. Equivalent production.
b. Cost of goods transferred from work in process to finished goods.
c. Net income for the period.
d. Cost per unit of ending inventory of work in process.
c 12. It is usually necessary to calculate equivalent unit production for
a. materials.
b. conversion costs.
c. materials and conversion costs.
d. materials, conversion costs, and overhead.
b 13. If a company uses actual process costing, the amount transferred from Work in Process
Inventory to Finished Goods Inventory is the cost of
a. equivalent unit production for the period.
b. units completed during the period.
c. units completed and sold during the period.
d. all units worked on during the period.
b 14. If a company uses standard process costing, the amount transferred from Work in Process
Inventory to Finished Goods Inventory is the
a. standard cost of equivalent unit production for the period.
b. standard cost of units completed during the period.
c. actual cost of units completed and sold during the period.
d. actual cost of all units worked on during the period.
a 15. Under standard costing, the amount of direct labor cost charged (debited) to Work in Process
Inventory is
a. standard labor hours at standard rates.
b. standard labor hours at actual rates.
c. actual labor hours at actual rates.
d. actual direct labor cost incurred.
d 16. A company that uses standard costing
a. must make only one product.
b. always has a volume variance unless normal capacity and practical capacity are the same.
c. shows higher incomes than it would if it used actual costing.
d. shows the same per-unit cost of inventory each month.
c 17. The numerator of weighted-average unit cost calculations is
a. current period cost.
b. cost of beginning inventory.
c. current period cost plus cost of beginning inventory.
134

d. cost of goods sold.


a 18. The numerator of the FIFO unit cost calculation is
a. current period cost.
b. cost of beginning inventory.
c. current period cost plus cost of beginning inventory.
d. cost of goods sold.
b 19. FIFO equivalent unit production (EUP) is 6,200 units. EUP in ending inventory is 300, in
beginning inventory it is 125. Weighted-average EUP is
a. 6,500.
b. 6,325.
c. 6,025.
d. 5,900.
a 20. Weighted-average EUP is 4,100 units. Cost incurred during the period are $11,250, and the
beginning inventory was $2,150. Unit cost is
a. $3.268.
b. $2.744.
c. $2.220
d. $0.524.
a 21. Weighted-average EUP is 11,400 units. Beginning inventory was 1,000 units 60% complete,
ending inventory is 2,000 units 20% complete. The number of units completed is
a. 11,000.
b. 10,800.
c. 10,400.
d. 9,400.
c 22. Algoma completed 10,000 units, had beginning inventory of 2,500 units 40% complete, and
ending inventory of 1,000 units 20% complete. Weighted-average EUP was
a. 9,200.
b. 10,000.
c. 10,200.
d. 11,000.
d 23. Which formula gives weighted-average equivalent unit production? (UC = units completed, BI =
equivalent units in beginning inventory, EI = equivalent units in ending inventory)
a. UC + BI + EI.
b. UC + BI - EI.
c. UC + EI - BI.
d. UC + EI.
c 24. Which formula gives FIFO equivalent unit production? (UC = units completed, BI = equivalent
units in beginning inventory, EI = equivalent units in ending inventory)
a. UC + BI + EI.
b. UC + BI - EI.
c. UC + EI - BI.
d. UC + EI.
c 25. The entry to apply overhead in a job-order system is
a. debit Cost of Goods Sold, credit Manufacturing Overhead.
b. debit Finished Goods Inventory, credit Work-in-Process Inventory.
c. debit Work-in-Process Inventory, credit Manufacturing Overhead.
d. debit Work-in-Process Inventory, credit Direct Labor.
a 26. Conversion costs are
a. labor and overhead costs.
b. materials and labor costs.
c. all fixed manufacturing costs.
d. all manufacturing costs.
a 27. Which item is NOT relevant in determining FIFO unit cost?
a. Cost of beginning inventory.
b. Equivalent unit production in beginning inventory.
c. Equivalent unit production in ending inventory.
d. Units completed.
c 28. Weighted-average equivalent production is always
a. less than the number of units completed.
b. equal to the number of units completed.
135

c. equal to or greater than the number of units completed.


d. any of the above.
d 29. FIFO equivalent production can be
a. less than the number of units completed.
b. equal to the number of units completed.
c. equal to or greater than the number of units completed.
d. any of the above.
a 30. Which item is NOT relevant in determining FIFO equivalent unit production?
a. Cost of beginning inventory.
b. Equivalent unit production in beginning inventory.
c. Equivalent unit production in ending inventory.
d. Units completed.
b 31. The FIFO method of calculating equivalent production and unit costs
a. is less likely to be accurate than the weighted-average method.
b. is more useful for control purposes than the weighted-average method.
c. cannot be used unless a company also uses standard costing.
d. eliminates the need to calculate separate equivalent-production numbers for each element of
manufacturing cost.
a 32. Backflushing, or backflush costing
a. requires significantly less recordkeeping than other methods.
b. can be used by any company.
c. ignores inventories.
d. does not distinguish between materials and conversion costs.
c 33. Scooter Corp had no beginning inventories, finished 40,000 units, and sold 36,000 units. There
were no ending inventories of materials or work in process. Materials purchased and used were
$225,000; direct labor and overhead were $170,000. Ending inventory would be valued at
a. $17,000.
b. $22,500.
c. $39,500.
d. some other number.
b 34. Scooter Corp had no beginning inventories, finished 40,000 units, and sold 36,000 units. There
were no ending inventories of materials or work in process. Materials purchased and used were
$225,000; direct labor and overhead were $170,000. Cost of goods sold would be valued at
a. $39,500.
b. $355,500.
c. $395,000.
d. some other number.
b 35. Dewey Company had a beginning inventory of 3,000 units 35% complete, and an ending
inventory of 2,500 units 20% complete. If 17,500 units were completed, weighted-average EUP is
a. 17,500.
b. 18,000.
c. 18,550.
d. 20,000.
b 36. Dewey Company had a beginning inventory of 3,000 units 35% complete, and an ending
inventory of 2,500 units 20% complete. If 17,500 units were completed, FIFO EUP is
a. 17,500.
b. 16,950.
c. 16,050.
d. 15,050.
a 37. Cheatem has a weighted-average EUP of 30,000 units. Beginning inventory was 4,000 units
40% complete; ending inventory was 5,000 units 60% complete. The number of units completed is
a. 27,000.
b. 29,000.
c. 30,000.
d. 31,000.
b 38. Cheatem has a weighted-average EUP of 30,000 units. Beginning inventory was 4,000 units
40% complete; ending inventory was 5,000 units 60% complete. FIFO EUP is
a. 25,400.
b. 28,400.
136

c. 30,000.
d. 31,000.
d 39. Howe has a FIFO EUP of 46,580 units. Beginning inventory of 6,500 units was 80% complete;
the ending inventory of 2,800 units was 60% complete. How many units were completed during the
period?
a. 39,700
b. 44,900
c. 46,200
d. 50,100
c 40. Howe has a FIFO EUP of 46,580 units. Beginning inventory of 6,500 units was 80% complete; the
ending inventory of 2,800 units was 60% complete. Weighted-average EUP is
a. 46,580.
b. 47,880.
c. 51,780.
d. some other number.
b 41. Sosa Inc. had $3,000 in beginning work in process and incurred an additional $28,500 during
the period. If weighted-average EUP was 10,000 units, unit cost would be
a. $2.85.
b. $3.15.
c. $9.50.
d. some other number.
a 42. Granger Co. had $3,000 in beginning work in process and incurred an additional $28,500 during
the period. If FIFO EUP was 10,000 units, unit cost would be
a. $2.85.
b. $3.15.
c. $9.50.
d. some other number.
b 43. Field Company had a beginning inventory of 2,000 units 40% complete, ending inventory of
1,500 units 70% complete, and transferred out 23,500 units. Weighted-average unit costs were $1.15
for materials, $0.75 for conversion costs. All materials are added at the start of the process. The cost
of finished units transferred to finished goods is
a. $28,750.
b. $44,650.
c. $47,500.
d. $52,250.
b 44. Field Company had a beginning inventory of 4,000 units 40% complete, ending inventory of
3,000 units 70% complete, and transferred out 47,000 units. Weighted-average unit costs were $1.15
for materials, $0.75 for conversion costs. All materials are added at the start of the process. The cost
of ending inventory is
a. $5,700.
b. $5,025
c. $3,990.
d. some other number.
b 45. Garden Co. had a beginning inventory of 3,000 units 60% complete, ending inventory of 3,000
units 80% complete, and transferred out 27,500 units. FIFO unit costs were $2.15 for materials, $1.25
for conversion costs. All materials are added at the start of the process. Beginning inventory cost
$9,400. The cost of finished units transferred out is
a. $69,875.
b. $92,900.
c. $93,500.
d. $103,700.
b 46. Garden Co. had a beginning inventory of 3,000 units 60% complete, ending inventory of 3,000
units 80% complete, and transferred out 27,500 units. FIFO unit costs were $2.15 for materials, $1.25
for conversion costs. All materials are added at the start of the process. Beginning inventory cost
$9,400. The cost of ending inventory is
a. $8,160.
b. $9,450.
c. $10,200.
d. $17,000.
d 47. Woods Run has a weighted-average EUP of 49,750 units. Beginning inventory of 4,500 units was
137

60% complete; the ending inventory of 4,800 units was 60% complete. The units completed during
the period is
a. 49,750.
b. 44,950.
c. 47,050.
d. 46,870.
b 48. Woods Run has a weighted-average EUP of 49,750 units. Beginning inventory of 4,500 units was
60% complete; the ending inventory of 4,800 units was 60% complete. Conversion costs in beginning
inventory were $1,960; conversion costs added during the period were $40,825. Conversion costs per
unit are
a. $0.82.
b. $0.86.
c. $0.70.
d. cannot be determined with the information given.
a 49. Grover Co. had a beginning inventory of 1,750 units 70% complete, ending inventory of 3,000
units 20% complete, and transferred out 24,500 units. Weighted-average unit costs were $2.15 for
materials, $1.75 for conversion costs. All materials are added at the start of the process. The cost of
finished units transferred to finished goods is
a. $95,550.
b. $102,375.
c. $107,250.
d. $114,075.
c 50. Grover Co. had a beginning inventory of 1,750 units 70% complete, ending inventory of 3,000
units 20% complete, and transferred out 24,500 units. Weighted-average unit costs were $2.15 for
materials, $1.75 for conversion costs. All materials are added at the start of the process. The cost of
ending inventory is
a. $2,340.
b. $6,450.
c. $7,500.
d. $11,700.
True-False
T 1. To calculate weighted-average equivalent production you do not need to know the number of
units in the beginning inventory.
F 2. Equivalent production calculated using FIFO is higher than equivalent production calculated
using weighted average.
T 3. Departmental overhead rates can be used by both job-order and process costing firms.
F 4. A multiproduct company cannot use standard costing.
T 5. Cost of Goods Sold and inventory accounts have debit balances.
F 6. Variance accounts have only credit balances.
F 7. Backflush costing eliminates the need for journal entries.
F 8. Backflush costing uses two inventory accounts: raw materials and a combined work in
process/finished goods.
T 9. If a company has no inventories, the weighted-average approach and the FIFO approach will
result in the same income.
F. 10. Although weighted average and FIFO may give different values for inventory, the resulting
income will always be the same.
Problems
1.

Clater uses weighted-average process costing. It had the following results in July.
Beginning inventory, 60% complete
2,000 units
Units completed
10,000 units
Units in ending inventory, 40% complete
1,000 units
Cost of beginning inventory
$21,000
138

Current period production costs

$166,200

a. Compute the equivalent unit production for July.


b.Compute the unit cost for July.
c. Compute the ending inventory of work in process.
d.Compute the cost transferred to finished goods.
SOLUTION:
a.

10,400 [10,000 + (1,000 x 40%)]

b.

$18 [($21,000 + $166,200)/10,400]

c.

$7,200 (1,000 x 40% x $18)

d.

$180,000 (10,000 x $18)

2.

Nikel Company uses FIFO process costing. Data are as follows:


Beginning inventory 40% complete
5,000 units
Units completed during period
100,000 units
Ending inventory 70% complete
9,000 units

The cost of the beginning inventory was $2,900 and current period production costs were
$166,880.
a. Compute equivalent production.
b.Compute the unit cost.
c. Compute the cost of the ending inventory of work in process.
d.Compute the cost of goods completed and transferred to finished goods inventory.
SOLUTION:
a.

104,300 [100,000 + (9,000 x 70%) - (5,000 x 40%)]

b.

$1.60

c.

$10,080, (9,000 x 70% x $1.60)

d.

$159,700

($166,880/104,300)

Beginning inventory
$ 2,900
Finish beginning inventory (5,000 x 60% x $1.60)
4,800
Units started and completed during period (95,000 x $1.60) 152,000
------Total
$159,700
========
3. The following data are available for 20X4 for Scottso, which uses weighted-average process
costing.
Beginning inventory (40% complete)
Units started during 20X4
Units completed during 20X4
Ending inventory (70% complete)
Costs of inventory at beginning of 20X4
Production costs incurred during 20X4

3,000 units
52,000 units
50,000 units
5,000 units
$2,750
$83,920

a. Compute equivalent production for 20X4.


b.Compute the unit cost for 20X4 to the nearest cent.
c. Compute the cost of the ending inventory of work in process.
d.Compute the cost of goods completed and transferred to finished goods.
139

e. Scottso now uses FIFO. Compute ending inventory of work in process.


SOLUTION
a. 53,500 [50,000 + (5,000 x 70%)]
b.

$1.62 [($2,750 + $83,920)/53,500]

c.

$5,670 [(5,000 x 70%) x $1.62]

d.

$81,000 (50,000 x $1.62)

e.

$5,616 ($1.6046 x 5,000 x 70%)


$83,920
-------------------------------------- = $1.6046
50,000 + (5,000 x 70%) - (3,000 x 40%)

4. Debra's Pottery Studios uses weighted-average process costing. It had the following results in
June.
Beginning inventory, 30% complete
12,000 units
Units completed
30,000 units
Units in ending inventory, 60% complete
9,000 units
Cost of beginning inventory
$45,000
Current period production costs
$379,800
a. Compute equivalent unit production for June.
b.Compute the unit cost for June.
c. Compute the ending inventory of work in process.
d.Compute the cost transferred to finished goods.
SOLUTION:
a.

35,400 [30,000 + (9,000 x 60%)]

b.

$12 [($45,000 + $379,800)/35,400]

c.

$64,800 (9,000 x 60% x $12)

$360,000 (30,000 x $12)


5. Dubois Corp. has a just-in-time manufacturing system and maintains no ending materials or work
in process inventory balances. Dubois uses backflush costing and had the following data for March.
Beginning inventories
none
Units finished
90,000
Units sold
88,000
Materials purchased and used
$375,000
Direct labor and manufacturing overhead
$525,000
a. Prepare journal entries to reflect the March activity.
b.Compute the ending finished goods inventory balance.
c. Compute the cost of goods sold.
SOLUTION:
a.

Materials and In-Process Inventory $375,000


Accounts Payable
$375,000
Conversion Costs
$525,000
Assorted accounts
$525,000
Cost of Goods Sold
$880,000
Finished Goods
$ 20,000
Materials and In-Process Inventory
$375,000
140

Conversion Costs
b.

$20,000

c.

$880,000

$525,000

[($375,000 + $525,000)/90,000 x 2,000]


[($375,000 + $525,000)/90,000 x 88,000]

6. Wheeler Inc. sold 125,000 units of product during the year. Variable cost per unit was $5,
standard fixed manufacturing cost per unit was $8, and selling and administrative costs were
$425,000. All costs were incurred as budgeted. Income was $175,000 after a favorable volume
variance of $100,000. There were no changes in inventory during the year.
a. Determine the selling price of a unit of product.
b.Determine the volume used to set the standard fixed manufacturing cost per unit.
c. Determine the budgeted amount of fixed manufacturing cost.
SOLUTION:
a.

$17

{[($8 + $5) x 125,000 + $425,000 + $175,000 - $100,000]/125,000}

b.

112,500 units [125,000 - ($100,000/$8)]

c.

$900,000

7.

DJH Company uses FIFO process costing. Data are as follows:

[(125,000 x $8) - $100,000]

Beginning inventory 20% complete


15,000 units
Units completed during period
200,000 units
Ending inventory 30% complete
19,000 units
The cost of the beginning inventory was $3,180 and current period production costs were
$222,970.
a. Compute equivalent production.
b.Compute the unit cost.
c. Compute the cost of the ending inventory of work in process.
d.Compute the cost of goods completed and transferred to finished goods inventory.
SOLUTION:
a.

202,700 [200,000 + (19,000 x 30%) - (15,000 x 20%)]

b.

$1.10

d.

$219,880

($222,970/202,700)c. $6,270 (19,000 x 30% x $1.10)

Beginning inventory
$ 3,180
Finish beginning inventory (15,000 x 80% x $1.10)
13,200
Units started and completed (185,000 x $1.10)
203,500
------Total
$219,880
========
8. The following data are available for 20X2 for Hunter, Inc., which uses weighted-average process
costing.
Beginning inventory (30% complete)
Units started during 20X2
Units completed during 20X2
Ending inventory (60% complete)
Costs of inventory at beginning of 20X2
Production costs incurred during 20X2

3,000 units
56,600 units
55,000 units
4,500 units
$1,458
$71,244

a. Compute equivalent production for 20X2.


b.Compute the unit cost for 20X2 to the nearest cent.
141

c. Compute the cost of the ending inventory of work in process.


d.Compute the cost of goods completed and transferred to finished goods.
SOLUTION:
a.

57,700 [55,000 + (4,500 x 60%)]

b.

$1.26 [($1,458 + $71,244)/57,700]

c.

$3,402 [(4,500 x 60%) x $1.26]

$69,300 (55,000 x $1.26)


9.

Molitor Inc. has the following data for July:


Gallons in beginning inventory
13,000 gallons
Gallons completed in July
52,500 gallons
Gallons in ending inventory
7,500 gallons
Percentage complete:
Materials
100 %
Conversion costs
45 %
Costs
Materials
Conversion Costs
Beginning Inventory $ 56,300
$ 33,600
Incurred during July $321,700
$256,950
Molitor uses the weighted-average method of costing.

a. Compute the equivalent units of production for materials and for conversion costs for the month
of July.
b.Compute the unit costs for each cost factor.
c. Compute the cost of finished gallons for July.
d.Compute the cost of ending work in process.
SOLUTION:
a.

Materials: 60,000 [52,500 + (7,500 x 100%)]


Conversion: 55,875 [52,500 + (7,500 x 45%)]

b.

Materials: $6.30 [($56,300 + $321,700)/60,000]


Conversion: $5.20 [($33,600 + $256,950)/55,875]

c.

$603,750 [($6.30 + 5.20) x 52,500]

d.

$64,800 Materials: 7,500 x 100% x $6.30 = $47,250


Conversion: 7,500 x 45% x $5.20 = 17,550
------$64,800
=======
10. Yount Inc. has the following data for September:
Gallons in beginning inventory
8,000 gallons
Percentage complete:
Materials
100 %
Conversion costs
60 %
Gallons completed in September
72,500 gallons
Gallons in ending inventory
10,000 gallons
Percentage complete:
Materials
100 %
Conversion costs
75 %
Costs
Materials
Conversion Costs
Beginning Inventory
$109,730
$ 38,950
Incurred during September $968,500
$661,760
142

Yount uses the FIFO method of costing.


a. Compute the equivalent units of production for materials and for conversion costs for the month
of September.
b.Compute the unit costs for each cost factor.
c. Compute the cost of finished gallons for September.
d.Compute the cost of ending work in process.
SOLUTION:
a.

Materials: 74,500 [72,500 + (10,000 x 100%) - (8,000 x 100%)]


Conversion: 75,200 [72,500 + (10,000 x 75%) - (8,000 x 60%)]

b.

Materials: $13.00 ($968,500/74,500)


Conversion: $ 8.80 ($661,760/75,200)

c.

$1,582,940
Beginning inventory ($109,730 + 38,950)
Finish beginning inventory (8,000 x 40% x $8.80)
Units started and completed (64,500 x $21.80)
--------Total
$1,582,940
==========
$196,000
Materials: 10,000 x 100% x $13.00 =
Conversion: 10,000 x 75% x $8.80 =
-------$196,000
========

d.

143

$148,680
28,160
1,406,100

$130,000
66,000

You might also like